BRS upper limb, Gray's Anatomy Review - Upper Limb

¡Supera tus tareas y exámenes ahora con Quizwiz!

16. A patient with Bennett's fracture (a fracture of the base of the fi rst metacarpal bone) experiences an impaired thumb movement. Which of the following intrinsic muscles of the thumb is most likely injured? (A) Abductor pollicis brevis (B) Flexor pollicis brevis (superfi cial head) (C) Opponens pollicis (D) Adductor pollicis (E) Flexor pollicis brevis (deep head)

The answer is C. The opponens pollicis inserts on the fi rst metacarpal. All other intrinsic muscles of the thumb, including the abductor pollicis brevis, the fl exor pollicis brevis, and the adductor pollicis muscles, insert on the proximal phalanges.

1 A 45-year-old woman is being examined as a candidate for cosmetic breast surgery. The surgeon notes that both of her breasts sag considerably. Which structure(s) has most likely become stretched to result in this condition? A. Scarpa's fascia B. Pectoralis major muscle C. Pectoralis minor muscle D. Suspensory (Cooper's) ligaments E. Serratus anterior muscle

1 D. The suspensory ligaments of the breast, also known as Cooper's ligaments, are fibrous bands that run from the dermis of the skin to the deep layer of superficial fascia and are primary supports for the breasts against gravity. Ptosis of the breast is usually due to the stretching of these ligaments and can be repaired with plastic surgery. Scarpa's fascia is the deep membranous layer of superficial fascia of the anterior abdominal wall. The pectoralis major and pectoralis minor are muscles that move the upper limb and lie deep to the breast but do not provide any direct support structure to the breast. The serratus anterior muscle is involved in the movements of the scapula. GAS 131, 139; N 179; McM 179

10 Laboratory studies in the outpatient clinic on a 24-year-old woman included assessment of circulating blood chemistry. Which of the following arteries is most likely at risk during venipuncture at the cubital fossa? A. Brachial B. Common interosseous C. Ulnar D. Anterior interosseous E. Radial

10 A. The three chief contents of the cubital fossa are the biceps brachii tendon, brachial artery, and median nerve (lateral to medial). The common and anterior interosseous arteries arise distal to the cubital fossa; the ulnar and radial arteries are the result of the bifurcation of the brachial artery distal to the cubital fossa. GAS 768; N 434; McM 151

10. A 35-year-old man walks in with a stab wound to the most medial side of the proximal portion of the cubital fossa. Which of the following structures would most likely be damaged? (A) Biceps brachii tendon (B) Radial nerve (C) Brachial artery (D) Radial recurrent artery (E) Median nerve

10. The answer is E. The contents of the cubital fossa from medial to lateral side are the median nerve, the brachial artery, the biceps brachii tendon, and the radial nerve. Thus, the median nerve is damaged. The radial recurrent artery ascends medial to the radial nerve.

100 Physical examination reveals weakness of medial deviation of the wrist (adduction), loss of sensation on the medial side of the hand, and clawing of the fingers. Where is the most likely place of injury? A. Compression of a nerve passing between the humeral and ulnar heads of origin of flexor carpi ulnaris B. Compression of a nerve passing at Guyon's canal between the pisiform bone and flexor retinaculum C. Compression of a nerve passing through the carpal tunnel D. Compression of a nerve passing between the ulnar and radial heads of origin of flexor digitorum superficialis E. Compression of a nerve passing deep to brachioradialis muscle

100 A. The ulnar nerve enters the forearm by passing between the two heads of the flexor carpi ulnaris and descends between and innervates the flexor carpi ulnaris (for medial wrist deviation) and flexor digitorum profundus (medial half) muscles. Injuring the ulnar nerve results in claw hand. It enters the hand superficial to the flexor retinaculum and lateral to the pisiform bone, where it is vulnerable to damage. The ulnar nerve also enters Guyon's canal, but damage to it here would not present with the aforementioned symptoms. The median nerve enters the carpal tunnel and the radial nerve passes deep to the brachioradialis. GAS 777, 784; N 464; McM 145

101 A 22-year-old pregnant woman was admitted urgently to the hospital after her baby had begun to appear at the introitus. The baby had presented in the breech position, and it had been necessary to exert considerable traction to complete the delivery. The newborn is shown in Fig. 6-7. Which of the following structures was most likely injured by the trauma of childbirth? A. Radial nerve B. Upper trunk of the brachial plexus C. Lower trunk of the brachial plexus D. Median, ulnar, and radial nerves E. Upper and lower trunks of the brachial plexus

101 B. During a breech delivery as described here, downward traction is applied to the shoulders and upper limbs as the baby is forcibly extracted from the birth canal. This exerts traction on the upper cord of the brachial plexus, often causing a traction injury from which the baby can often recover. If the ventral rami of C5 and C6 are avulsed from the spinal cord, the injury is permanent. GAS 738, 747; N 416; McM 31

102 A 17-year-old female student of martial arts entered the emergency department with a complaint of pain in her hand. Patient history reveals that she had been breaking concrete blocks with her hand. Examination reveals that the patient has weak abduction and adduction of her fingers but has no difficulty in flexing them. The patient also has decreased sensation over the palmar surfaces of the fourth and fifth digits. Which of the following best describes the nature of her injury? A. Compression of the median nerve in the carpal tunnel B. Fracture of the triquetrum, with injury to the dorsal ulnar nerve C. Dislocation of a bone in the proximal row of the carpus D. Fracture of the shaft of the fifth metacarpal E. Injury of the ulnar nerve in Guyon's canal

102 E. Striking the concrete blocks with the medial side of her hand has injured the ulnar nerve in Guyon's canal. This is the triangular tunnel formed by the pisiform bone medially, the flexor retinaculum dorsally, and the deep fascia of the wrist ventrally. This injury would result in loss of sensation to the medial palm and the palmar surface of the medial one and a half digits and motor loss of the hypothenar muscles, the interossei, and the medial two lumbricals. The median nerve is not involved because the thenar muscles and lateral palmar sensations are intact. The dorsal ulnar nerve arises proximal to the wrist, thus it would not be lost. Carpal bone dislocation is unlikely. If the lunate bone were dislocated, it would not cause compression of the ulnar nerve at the wrist. There is no indication of fifth metacarpal fracture, the so-called boxer's fracture. GAS 784, 814; N 449; McM 159

103 A 10-year-old boy suffered a dog bite that entered the common flexor synovial sheath of his forearm. He was admitted to the hospital, where the wound was cleaned and dressed and he was treated further with rabies antiserum. Two days later the boy had an elevated temperature and his palm and one digit were obviously swollen, causing him to cry with pain. Into which of the digits could the infection spread most easily, following the anatomy of the typical common flexor sheath? A. First B. Second C. Third D. Fourth E. Fifth

103 E. The common flexor sheath encloses the long flexor tendons of the fingers in the carpal tunnel and proximal palm. This sheath is usually continuous with the flexor sheath of the little (fifth) finger, which continues within the palm, having no connection with sheaths of the other digits, which do not extend into the digits. GAS 800-802; N 448; McM 158

104 While sharpening his knife, a 23-year-old male soldier accidentally punctured the ventral side of the fifth digit at the base of the distal phalanx. The wound became infected, and within a few days the infection had spread into the palm, within the sheath of the flexor digitorum profundus tendons. If the infection were left untreated, into which of the following spaces could it most likely spread? A. Central compartment B. Hypothenar compartment C. Midpalmar space D. Thenar compartment E. Thenar space

104 C. The infectious agent was introduced into the synovial sheath of the long tendons of the little (fifth) finger. Proximally, this sheath runs through the midpalmar space, and inflammatory processes typically rupture into this space unless aggressively treated with the appropriate antibiotics. GAS 800-801; N 448; McM 158

105 A 36-year-old man is admitted to the emergency department with a dull ache in his shoulder and axilla (Fig. 6-8). During physical examination the pain worsens by activity, and, conversely, rest and elevation relieve the pain. History reveals that the patient was hospitalized 2 days ago and a central venous line was used. What is the most likely diagnosis? A. Axillary-subclavian vein thrombosis B. Compression of C5 to C8 spinal nerve C. Disc herniation of C4 to C8 D. Impingement syndrome E. Injury to radial, ulnar, and median nerves

105 A. Axillary-subclavian vein thrombosis is becoming much more common in recent years because of the extensive use of catheters in cancer patients and other chronic medical conditions. Effort-induced thrombosis is seen with strenuous use of the dominant arm with hyperabduction and external rotation of the arm or backward and downward rotation of the shoulder as in playing cricket, volleyball, or baseball, or chopping wood. Because the symptoms of subclavian stenosis are fairly dramatic, most patients present promptly, usually within 24 hours. They complain of a dull ache in the shoulder and axilla, the pain worsened by activity. Conversely, rest and elevation often relieve the pain. Patients with catheter-associated axillary-subclavian deep vein thrombosis report similar symptoms at the arm or shoulder on the side with the indwelling catheter. GAS 759; N 420; McM 206

106 A 22-year-old woman had suffered a severe knife wound to the upper lateral portion of her pectoral region, with entry of the knife at the deltopectoral groove. Pressure applied to the wound had prevented further profuse bleeding. In the emergency department, vascular clamps were applied to the axillary artery, proximal and distal to the site of injury, which had occurred between the second and third parts of the axillary artery. The vascular surgeon knew there was time to repair the wound of the artery because of the rich collateral pathway provided by the anastomoses between which of the following arteries? A. Transverse cervical and suprascapular B. Posterior circumflex humeral and profunda brachii C. Suprascapular and circumflex scapular D. Supreme (superior) thoracic and thoracoacromial E. Lateral thoracic and suprascapular

106 C. The injury is at the second part of the axillary artery. The suprascapular artery is a branch of the thyrocervical trunk off the subclavian artery, proximal to the axillary artery. The subscapular artery is the major branch of the third part of the axillary artery, giving off the thoracodorsal and the circumflex scapular arteries. In this case blood would be flowing from the circumflex scapular artery in a retrograde direction into the axillary artery, supplying blood distal to the injury. GAS 733-735; N 420; McM 134

107 In a penetrating wound to the forearm of a 24-year-old man, the median nerve is injured at the entrance of the nerve into the forearm. Which of the following would most likely be apparent when the patient's hand is relaxed? A. The MCP and IP joints of the second and third digits of the hand will be in a condition of extension. B. The third and fourth digits will be held in a slightly flexed position. C. The thumb will be flexed and slightly abducted. D. The first, second, and third digits will be held in a slightly flexed position. E. The MCP and IP joints of the second and third digits of the hand will be in a condition of flexion.

107 A. This proximal injury to the median nerve would paralyze all of the long flexors of the digits, except for the muscle that flexes the distal interphalangeal joints of digits 4 and 5, thereby swinging the "balance of power" to the muscles that extend the digits, all of which are innervated by the radial nerve. The intrinsic hand muscles can aid in flexion of the metacarpophalangeal joints, and are innervated by the ulnar nerve. However, they are of insufficient size to compensate for the extensor forces exerted on fingers. GAS 742-746; N 463; McM 148

108 A 55-year-old male firefighter is admitted to the hospital after blunt trauma to his right axilla. Examination reveals winging of the scapula and partial paralysis of the right side of the diaphragm. Which of the following parts of the brachial plexus have been injured? A. Cords B. Divisions C. Ventral rami D. Terminal branches E. Trunks

108 C. The winged scapula results from a lesion of the long thoracic nerve, which supplies the serratus anterior muscle. This muscle is responsible for rotating the scapula upward, which occurs during abduction of the arm above the horizontal. The long thoracic nerve arises from the ventral rami of C5 to C7 of the brachial plexus. The diaphragm is supplied by the phrenic nerve, which comes from the ventral rami of C3 to C5 (mnemonic: C3, 4 and 5 keep the diaphragm alive). GAS 727; N 413; McM 129

109 A 69-year-old man has numbness and pain in the middle three digits of his right hand at night. He retired 9 years ago after working as a carpenter for 30 years. He has atrophy of the thenar eminence (see Fig. 6-3). Which of the following conditions will be the most likely cause of this atrophy? A. Compression of the median nerve in the carpal tunnel B. Formation of the osteophytes that compress the ulnar nerve at the ulnar condyle C. Hypertrophy of the triceps brachii muscle compressing the brachial plexus D. Osteoarthritis of the cervical spine E. Repeated trauma to the ulnar nerve

109 A. The median nerve supplies sensory innervation to the thumb, index, and middle fingers as well as to the lateral half of the ring finger. The median nerve also provides motor innervation to muscles of the thenar eminence. Compression of the median nerve in the carpal tunnel explains these deficits in conjunction with normal functioning of the flexor compartment of the forearm. The ulnar nerve is not implicated in these symptoms. Compression of the brachial plexus could not be attributed to pressure from hypertrophy of the triceps brachii muscle, it is located distal to the plexus. In addition, symptoms would include several upper limb deficits rather than the focal symptoms described in this instance. Osteoarthritis of the cervical spine would also lead to increasing complexity of symptoms. GAS 745, 817; N 463; McM 159

11 A 22-year-old man is diagnosed with metastatic malignant melanoma of the skin over the xiphoid process. Which lymph nodes receive most of the lymph from this area and are therefore most likely to be involved in metastasis of the tumor? A. Deep inguinal B. Vertical group of superficial inguinal C. Horizontal group of superficial inguinal D. Axillary E. Deep and superficial inguinal

11 D. Lymph from the skin of the anterior chest wall primarily drains to the axillary lymph nodes. GAS 748; N 412; McM 179

110 A 54-year-old woman presents with pain in her right wrist that resulted when she fell forcefully on her outstretched hand. Radiographic studies indicate an anterior dislocation of a carpal bone of the proximal row (see Fig. 6-4). Which of the following bones is most commonly dislocated? A. Capitate B. Lunate C. Scaphoid D. Pisiform E. Triquetrum

110 B. The lunate bone is the most commonly dislocated carpal bone. Displacement is almost always anteriorly. Dislocation of the lunate bone can precipitate the signs associated typically with carpal tunnel syndrome. GAS 793-795; N 443; McM 122

111 A 32-year-old man who is an expert target shooter reports pain in his right upper limb and slight tingling and numbness of all digits of the ipsilateral hand. However, the tingling and numbness of the fourth and fifth digits is the most severe. The man states that the problem usually occurs when he is firing his gun with his hand overhead. Radiographic studies reveal the presence of a cervical rib and accessory scalene musculature. Which of the following structures is most likely being compressed? A. Axillary artery B. Upper trunk of brachial plexus C. Subclavian artery D. Lower trunk of brachial plexus E. Brachiocephalic artery and lower trunk of brachial plexus

111 D. A cervical rib (usually found at C7) may cause thoracic outlet syndrome, which is a condition characterized by weak muscle tone in the hand and loss of radial pulse when the upper limb is abducted above the shoulder. The mechanism of injury with the gun being fired overhead suggests a lower trunk injury to the brachial plexus. The axillary artery supplies the shoulder muscles, and there is no loss of function to these muscles. The upper trunk of the brachial plexus also supplies innervation to the shoulder muscles, which are unaffected based on the patient's presenting abnormalities. The subclavian artery is located anterior to the brachial plexus until the plexus separates into cords as it passes under the clavicle. The brachiocephalic artery and lower trunk of the brachial plexus is only partially correct; the brachiocephalic artery is not directly associated with the brachial plexus due to its location at the midline of the body behind the sternum. GAS 150; N 183, 416; McM 129

112 A 23-year-old woman arrives at the emergency department with a swollen, painful forearm. An MRI examination reveals a compartment syndrome originating at the interosseous membrane between the radius and ulna. Which of the following type of joint will most likely be affected? A. Synarthrosis B. Symphysis C. Synchondrosis D. Trochoid E. Ginglymus

112 A. A synarthrosis joint is a fibrous connection that allows minimal to no movement. In this case, virtually no movement is allowed by the interosseous membrane joint between the radius and ulna. Symphysis joints are permanent fibrocartilaginous fusions between two bones; pubic symphysis is an example. Synchondrosis is a temporary joint made of cartilage that transitions to bone typically after growth completes (i.e., epiphyseal plate). Trochoid joints are pivot joints, and the humeral-radial portion of the elbow joint is an example. Ginglymus joints are hinge joints located at the interphalangeal junctions in the hand and foot (PIPs and DIPs). GAS 18-20, 774-775; N 425; McM 146

113 While working out with weights, a 28-year-old woman experiences a severe pain in her chest. The pain is referred to the anterior chest wall, radiating to the mandible and her left arm. The woman felt dizzy and after 10 minutes she collapsed and was unconscious. A physician happened to be near the woman and immediately tried to feel her radial pulse. The radial artery lies between two tendons near the wrist, which are useful landmarks. Which of the following is the correct pair of tendons? A. Flexor carpi radialis and palmaris longus B. Flexor carpi radialis and brachioradialis C. Brachioradialis and flexor pollicis longus D. Flexor pollicis longus and flexor digitorum superficialis E. Flexor pollicis longus and flexor digitorum profundus

113 B. The radial pulse is best located on the forearm (antebrachium) just proximal to the wrist joint. At this point the radial artery travels on the distal radius between the flexor carpi radialis and brachioradialis tendons. The palmaris longus tendon travels more medially to the radial artery and above the flexor retinaculum. The flexor pollicis longus tendon is a deeper structure in the antebrachium and is also located medially to the radial artery. GAS 827; N 432; McM 161

114 A 59-year-old woman is admitted to the hospital in a state of shock. During physical examination, several lacerations are noted in her forearm and her radial pulse is absent. Where is the most typical place to identify the radial artery immediately after crossing the radiocarpal joint? A. Between the two heads of the first dorsal interosseous muscle B. In the anatomic snuffbox C. Below the tendon of the flexor pollicis longus D. Between the first and second dorsal interossei muscles E. Between the first dorsal interosseous muscle and the adductor pollicis longus

114 B. The radial artery enters the palm through the anatomic snuffbox. The artery then moves on to pierce through the two heads of the first dorsal interosseous muscle and enter the deep aspect of the palm. The flexor pollicis longus tendon runs on the palmar aspect of the hand and the radial artery runs on the dorsal aspect of the hand before entering the deep aspect of the palm, and therefore the radial artery does not run below this tendon. The radial artery does not run between the first and second interosseous muscle and therefore cannot be used as a landmark to identify the artery. Finally, the artery does not run between the first dorsal interosseous muscle and the adductor pollicis longus. GAS 800, 812; N 454; McM 161

115 A 69-year-old woman visits the outpatient clinic with a complaint of numbness and tingling of her hand for the past 3 months. Physical examination reveals she has numbness and pain in the lateral three digits of her right hand that are relieved by vigorous shaking of the wrist. In addition, the abductor pollicis brevis, opponens pollicis, and the first two lumbrical muscles are weakened. Sensation was decreased over the lateral palm and the volar aspect of the first three digits. Which of the following nerves is most likely compressed? A. Ulnar B. Radial C. Recurrent median D. Median E. Posterior interosseous

115 D. The median nerve provides innervation to most of the muscles in the flexor compartment of the forearm; cutaneous innervation of the second, third, and fourth digits and palmar and dorsum aspects of the hand; and innervation of four intrinsic hand muscles: first and second lumbricals, abductor pollicis brevis, opponens pollicis, and flexor pollicis brevis. The thenar compartment contains the abductor pollicis brevis, opponens pollicis, and flexor pollicis brevis muscles, and these muscles are innervated by the recurrent branch of the median nerve. The patient has weakening of the first two lumbricals and not simply the thenar muscles, so the median nerve is most likely to be compressed. Another indication that the median nerve is compressed is the vigorous shaking of the wrist. Because the median nerve traverses the carpal tunnel, carpal tunnel compression could lead to this action on part of the patient. The ulnar nerve provides innervation for part of the flexor digitorum profundus and flexor carpi ulnaris muscles. These muscles are not weakened in this patient. The radial nerve provides cutaneous supply to the dorsum of the hand and forearm as well as extensor muscles of the forearm. The posterior interosseous nerve is a branch of the radial nerve and provides innervation of the extensor muscles in the forearm. GAS 745, 784; N 463; McM 157

116 A 32-year-old man is admitted to the emergency department after a severe car crash. Radiographic examination reveals multiple fractures of his right upper limb. A surgical procedure is performed and metallic plates are attached to various bony fragments to restore the anatomy. Five months postoperatively the patient visits the outpatient clinic. Upon physical examination the patient can abduct his arm and extend the forearm, and the sensation of the forearm and hand is intact; however, hand grasp is very weak, and he cannot extend his wrist against gravity. Which of the following nerves was most likely injured during the surgical procedure? A. Posterior cord of the brachial plexus B. Radial nerve at the distal third of the humerus C. Radial and ulnar D. Radial, ulnar, and median E. Radial and musculocutaneous

116 B. The patient can extend his forearm, which suggests that the triceps brachii muscle is not weakened. Supination appears to be weak along with hand grasp and wrist drop. This would indicate that part of the radial nerve has been lost below the innervation of the triceps brachii and above the branches to the supinator and extensors in the forearm. However, sensation on the forearm and hand is intact, indicating that the superficial branch of the radial nerve is intact. The superficial branch of the radial nerve separates from the deep radial nerve at the distal third of the humerus. The posterior cord of the brachial plexus is responsible for providing innervation of the axially and radial nerves. This patient does have some radial nerve innervation and no loss of axillary nerve function. The patient does not have weakened adduction of the wrist, indicating that the ulnar nerve is not injured. If both the radial and musculocutaneous nerves are injured, supination would not be possible as the supinator and biceps brachii muscles provide supination of the forearm. GAS 761-763, 785; N 465; McM 143

117 A 52-year-old man is admitted to the emergency department after falling on wet pavement. Radiographic examination reveals fracture of the radius. An MRI study reveals a hematoma between the fractured radius and supinator muscle. Upon physical examination the patient has weakened abduction of the thumb and extension of the metacarpophalangeal joints of the fingers. Which of the following nerves is most likely affected? A. Anterior interosseous B. Posterior interosseous C. Radial nerve D. Deep branch of ulnar nerve E. Median nerve

117 B. The posterior interosseous nerve is an extension of the deep branch of the radial nerve after it emerges distal to the supinator. It is responsible for innervation of several muscles in the extensor compartment of the posterior aspect of the forearm, including extension of the metacarpophalangeal joints. The deep radial nerve courses laterally around the radius and passes between the two heads of the supinator muscle and is thus likely to be compressed by a hematoma between the fractured radius and the supinator muscle. Though the radial nerve gives rise to the posterior interosseous nerve, this answer choice is too general and would not indicate the precise injured branch of the radial nerve. Both the deep branch of the ulnar nerve and the median nerve traverse the medial and anteromedial aspect of the arm, respectively. These nerves primarily supply the flexor compartment of the arm. The anterior interosseous nerve is a branch of the median nerve and supplies the flexor digitorum profundus, flexor pollicis longus, and the pronator quadratus muscles (GAS Fig. 7-90). GAS 785, 792; N 466; McM 152

118 A 34-year-old woman is admitted to the emergency department after a car crash. Radiographic studies show marked edema and hematoma of the arm, but there are no fractures. During physical examination the patient presents with inability to abduct her arm without first establishing lateral momentum of the limb, and inability to flex the elbow and shoulder. Which of the following portions of the brachial plexus is most likely injured? A. Superior trunk B. Middle trunk C. Inferior trunk D. Lateral cord E. Medial cord

118 A. The superior trunk of the brachial plexus includes C5 and C6, which give rise to the suprascapular nerve, which innervates the supraspinatus muscle. The supraspinatus muscle is the primary muscle involved in abduction of the arm from 0 to 15 degrees. The deltoid muscle, supplied primarily by C5, abducts the arm from 15 to 90 degrees. The middle trunk is just C7 and has nothing to do with the muscle involved in initial abduction of the arm. The inferior trunk is C8-T1 and does not supply the supraspinatus muscle; therefore, it is not the right answer. The cords are distal to the branching of the supraspinatus muscle; therefore, neither lateral cord nor medial cord is the correct answer. GAS 738, 747; N 416; McM 28

119 A 22-year-old man is admitted to the hospital after a car collision. Radiographic examination reveals an oblique fracture of his humerus. Upon physical examination the patient is unable to extend his forearm. The damaged nerve was most likely composed of fibers from which of the following spinal levels? A. C5, C6 B. C5, C6, C7 C. C5, C6, C7, C8, T1 D. C6, C7, C8, T1 E. C7, C8, T1

119 C. The radial nerve acts to extend the forearm at the elbow. This nerve is derived from all the ventral rami of the brachial plexus C5 to T1. None of the other answers includes all the ventral rami and are therefore incorrect. GAS 745-746; N 416; McM 96

12 A 49-year-old woman who had suffered a myocardial infarction must undergo a bypass graft procedure using the internal thoracic artery. Which vessels will most likely continue to supply blood to the anterior part of the upper intercostal spaces? A. Musculophrenic B. Superior epigastric C. Posterior intercostal D. Lateral thoracic E. Thoracodorsal

12 C. The anterior intercostal arteries are twelve small arteries, two in each of the upper six intercostal spaces at the upper and lower borders. The upper artery lying in each space anastomoses with the posterior intercostal arteries, whereas the lower one usually joins the collateral branch of the posterior intercostal artery. The musculophrenic artery is a terminal branch of the internal thoracic artery (also known as the internal mammary artery), and it supplies the pericardium, diaphragm, and muscles of the abdominal wall. It anastomoses with the deep circumflex iliac artery. The superior epigastric artery is the other terminal branch of the internal thoracic artery, and it supplies the diaphragm, peritoneum, and the anterior abdominal wall and anastomoses with the inferior epigastric artery. The lateral thoracic artery runs along the lateral border of the pectoralis minor muscle and supplies the pectoralis major, pectoralis minor, and serratus anterior. The thoracodorsal artery accompanies the thoracodorsal nerve in supplying the latissimus dorsi muscle and lateral thoracic wall. GAS 155; N 188; McM 183

12. An automobile body shop worker has his middle fi nger crushed while working on a transmission. Which of the following muscles is most likely to retain function? (A) Extensor digitorum (B) Flexor digitorum profundus (C) Palmar interosseous (D) Dorsal interosseous (E) Lumbrical

12. The answer is C. The extensor digitorum, fl exor digitorum profundus, dorsal interosseous, and lumbrical muscles are attached to the middle digit, but no palmar interosseous muscle is attached to the middle digit.

120 A 56-year-old woman is admitted to the hospital after a severe car crash. A large portion of her chest wall needed to be surgically removed and replaced with a musculo-osseous scapular graft involving the medial border of the scapula. Which of the following arteries will most likely recompensate the blood supply to the entire scapula? A. Suprascapular B. Dorsal scapular artery C. Posterior circumflex humeral artery D. Lateral thoracic E. Supreme thoracic artery

120 A. The suprascapular artery arises as a major branch of the thyrocervical trunk from the subclavian artery. It has rich anastomoses with the circumflex scapular artery and could provide essential blood supply to the scapula. The dorsal scapular artery would be lost with the graft. None of the other vessels listed is in position to provide adequate supply to the scapula. GAS 720; N 414; McM 31

121 A 56-year-old woman visits the emergency department after falling on a wet pavement. Radiographic examination reveals osteoporosis and a Colles' fracture. Which of the following carpal bones are often fractured or dislocated with a Colles' fracture? A. Triquetrum and scaphoid B. Triquetrum and lunate C. Scaphoid and lunate D. Triquetrum, lunate, and scaphoid E. Triquetrum and pisiform

121 C. The scaphoid and lunate carpal bones have a direct articulation with the radius, which is fractured in a Colles' fracture; therefore, they would most likely be disrupted or fractured. The other carpal bones listed do not have direct contact with the radius and have a more distal location; therefore, they would not be as likely to be injured with a Colles' fracture. GAS 774; N 439; McM 123

122 A 3-year-old girl is admitted to the emergency department with severe pain. History taking reveals that the girl was violently lifted by her raised arm by her mother to prevent the girl from walking in front of a moving car. Which of the following is most likely the cause of the pain? A. Compression of the median nerve B. Separation of the head of the radius from its articulation with the trochlea of the humerus C. Separation of the head of the radius from its articulation with the ulna and the capitulum of the humerus D. Separation of the ulna from its articulation with the trochlea of the humerus E. Stretching of the radial nerve as it passes behind the medial epicondyle of the humerus

122 C. This type of dislocation is common in children and results when the radius is dislocated and slips out from the anular ligament, which holds it in place, articulating with the ulna and the capitulum of the humerus. In adults the anular ligament has a good "grip" at the radial neck, but in young children the radial head is not fully developed, leading to an indistinct neck. Compression of the median nerve is not likely due to its medial position in the cubital fossa. The radius does not articulate with the trochlea of the humerus; the ulna articulates at this position. The ulna is not likely to be dislocated because it is more stable than the radius, which has only the anular ligament for its support. The radial nerve does not pass behind the medial epicondyle; rather, the ulnar nerve does this, so this is not the correct answer. GAS 766-768, 775; N 424; McM 146

123 A 61-year-old man was hit by a cricket bat in the midhumeral region of his left arm. Physical examination reveals normal elbow motion; however, he could not extend his wrist or his metacarpophalangeal joints and he reported a loss of sensation on a small area of skin on the dorsum of the hand proximal to the first two digits. Radiographic examination reveals a hairline fracture of the shaft of the humerus just distal to its midpoint. Which of the following nerves is most likely injured? A. Median B. Ulnar C. Radial D. Musculocutaneous E. Axillary

123 C. Injury to the radial nerve can be caused by a blow to the midhumeral region because the nerve winds around the shaft of the humerus. The symptoms described include the loss of wrist and finger extension and a loss of sensation in an area of skin supplied by the radial nerve. GAS 763; N 465; McM 144

124 A 34-year-old man is admitted to the hospital after a car collision. Radiographic examination reveals a fracture at his wrist. Physical examination reveals paralysis of the muscles that act to extend the interphalangeal joints (Fig. 6-9). Which of the following nerves is most likely injured? A. Ulnar B. Recurrent branch of median C. Radial D. Musculocutaneous E. Anterior interosseous

124 A. The ulnar nerve innervates the dorsal and palmar interossei, which act to abduct and adduct the fingers and assist the lumbricals in their actions of flexing the metacarpophalangeal joints and extending the interphalangeal joints. The recurrent branch of the median nerve innervates the thenar muscle group that functions in the movement of the thumb. The radial and musculocutaneous nerves do not innervate any muscles in the hand. The anterior interosseous innervates the flexor pollicis longus and the pronator quadratus. GAS 808-809; N 464; McM 159

125 A 45-year-old woman is admitted to the hospital with neck pain. An MRI examination reveals a herniated disc in the cervical region. Physical examination reveals weak triceps brachii muscle. Which of the following spinal nerves is most likely injured? A. C5 B. C6 C. C7 D. C8 E. T1

125 C. The triceps brachii muscle is innervated by the radial nerve (primarily C7), which comes off C5 to T1 spinal nerves. Because the patient's only motor deficit involves the triceps brachii muscles, one can rule out C5 and C6, which supply fibers to the axillary, musculocutaneous, and upper subscapular nerves. Damage to either of these ventral rami would result in additional motor deficits of the shoulder and flexor compartment of the arm. One can also rule out C8-T1 because these ventral rami form the medial pectoral nerve and the medial brachial and antebrachial cutaneous nerves. Damage to these ventral rami would result in loss of pectoral muscle function and cutaneous sensation over the medial surface of the upper limb. GAS 745, 761; N 416; McM 96

126 A 34-year-old woman is admitted to the hospital after a car collision. Physical examination reveals a mallet finger. Which of the following conditions is expected to be present during radiographic examination? A. A lesion of the ulnar nerve at the distal flexor crease of the wrist B. A separation of the extension expansion over the middle interphalangeal joint C. Compression of the deep ulnar nerve by dislocation of the lunate bone D. Avulsion fracture of the dorsum of the distal phalanx E. Fracture of the fourth or fifth metacarpal bone

126 D. Mallet finger, also known as baseball finger, is a deformity in which the finger is permanently flexed at the distal interphalangeal joint due to avulsion of the insertion of the extensor tendon at the distal phalanx. GAS 802; N 443; McM 165

127 A 42-year-old woman is admitted to the hospital with injury to the upper (superior) trunk of the brachial plexus. The diagnosis is Erb-Duchenne palsy. Which of the following conditions is expected to be present during physical examination? A. Winged scapula B. Inability to laterally rotate the arm C. Paralysis of intrinsic muscles of the hand D. Paraesthesia in the medial aspect of the arm E. Loss of sensation in the dorsum of the hand

127 B. Injury to the superior trunk of the brachial plexus can damage nerve fibers going to the suprascapular, axillary, and musculocutaneous nerves. Damage to the suprascapular and axillary nerves causes impaired abduction and lateral rotation of the arm. Damage to the musculocutaneous nerve causes impaired flexion of the forearm. A winged scapula would be caused by damage to the long thoracic nerve. The long thoracic nerve is formed from spinal cord levels C5, C6, and C7, so the serratus anterior muscle would be weakened from the damage to C5 and C6, but the muscle would not be completely paralyzed. The intrinsic muscles of the hand are innervated by the ulnar nerve, which would most likely remain intact. Paraesthesia in the medial aspect of the arm would be caused by damage to the medial brachial cutaneous nerve (C8-T1; inferior trunk). Loss of sensation on the dorsum of the hand would be caused by damage to either the ulnar or radial nerves (C6 to T1). GAS 738, 747; N 416; McM 31

128 A 41-year-old woman is admitted to the hospital after a car crash. Radiographic examination reveals a transverse fracture of the radius proximal to the attachment of the pronator teres muscle. The proximal portion of the radius is deviated laterally. Which of the following muscles will most likely be responsible for this deviation? A. Pronator teres B. Pronator quadratus C. Brachialis D. Supinator E. Brachioradialis

128 D. The supinator muscle attaches to the radius proximally and when fractured would cause a lateral deviation. The pronator teres muscle originates on the medial epicondyle and coronoid process of the ulna and inserts onto the middle of the lateral side of the radius, pulling the radius medially below the fracture. The pronator quadratus muscle originates on the anterior surface of the distal ulna and inserts on the anterior surface of the distal radius, pulling the radius medially. The brachioradialis muscle originates on the lateral supracondylar ridge of the humerus and inserts at the base of the radial styloid process, far below the fracture. The brachialis muscle originates in the lower anterior surface of the humerus and inserts in the coronoid process and ulnar tuberosity, hence not causing an action on the radius. GAS 777, 787-788; N 426; McM 121

129 A 45-year-old woman is bitten by a dog on the lateral side of her hand. Two days later the woman develops fever and swollen lymph nodes. Which of the following group of lymphatics will most likely be involved? A. Central B. Humeral C. Pectoral D. Subscapular E. Parasternal

129 A. Lymph from the lateral side of the hand drains directly into humeral (epitrochlear) nodes then to the central (axillary) nodes. Pectoral nodes receive lymph mainly from the anterior thoracic wall, including most of the breast. Subscapular nodes receive lymph from the posterior aspect of the thoracic wall and scapular region. Parasternal nodes receive lymph from the lower medial quadrant of the breast (GAS Fig. 7-57). GAS 748; N 403; McM 364

13 A 22-year-old woman is admitted to the emergency department in an unconscious state. The nurse takes a radial pulse to determine the heart rate of the patient. This pulse is felt lateral to which tendon? A. Palmaris longus B. Flexor pollicis longus C. Flexor digitorum profundus D. Flexor carpi radialis E. Flexor digitorum superficialis

13 D. The location for palpation of the radial pulse is lateral to the tendon of the flexor carpi radialis, where the radial artery can be compressed against the distal radius. The radial pulse can also be felt in the anatomic snuffbox between the tendons of the extensor pollicis brevis and extensor pollicis longus muscles, where the radial artery can be compressed against the scaphoid. GAS 782, 827; N 432; McM 150

13. A 14-year-old boy falls on his outstretched hand and has a fracture of the scaphoid bone. The fracture is most likely accompanied by a rupture of which of the following arteries? (A) Brachial artery (B) Ulnar artery (C) Deep palmar arterial arch (D) Radial artery (E) Princeps pollicis artery

13. The answer is D. The scaphoid bone forms the fl oor of the anatomic snuffbox, through which the radial artery passes to enter the palm. The radial artery divides into the princeps pollicis artery and the deep palmar arch.

130 A 25-year-old woman is admitted to the emergency department after a car collision. Radiographic examination reveals a fracture at the spiral groove of the humerus. A cast is placed, and 3 days later she complains of severe pain over the length of her arm. During physical examination the arm appears swollen, pale, and cool. Radial pulse is absent, and any movement of the arm causes severe pain. Which of the following conditions will most likely characterize the findings of the physical examination? A. Venous thrombosis B. Thoracic outlet syndrome C. Compartment syndrome D. Raynaud's disease E. Injury of the radial nerve

130 C. Compartment syndrome is characterized by increased pressure within a confined space by a fascial compartment, which impairs blood supply, resulting in paleness and loss of pulses distal to the compartment. Venous thrombosis would not cause pain but could cause death from a pulmonary embolism if a thrombus (clot) broke free and became lodged in the pulmonary trunk. Thoracic outlet syndrome affects nerves in the brachial plexus and the subclavian artery and blood vessels between the neck and the axilla, far above the cast. Raynaud's disease affects blood flow to the limbs when they are exposed to temperature changes or stress. The fracture at the radial groove probably resulted in a radial nerve injury but would not be responsible for these symptoms. GAS 590, 763; N 432; McM 143

131 A 22-year-old woman is admitted to the hospital after falling from a tree. Radiographic examination reveals fractured pisiform and hamate bones. Which of the following nerves will most likely be injured? A. Median B. Recurrent median C. Radial D. Anterior interosseous E. Deep ulnar

131 E. The deep branch of the ulnar nerve arises at the level of the pisiform bone and passes between the pisiform and the hook of the hamate; hence the deep branch of the ulnar nerve is most likely to be injured in this patient. The median nerve enters the forearm between the humeral and ulnar heads of the pronator teres muscle then becomes superficial near the wrist. The recurrent branch of the median nerve branches off after the median nerve enters the palm through the carpal tunnel. The radial nerve divides into superficial and deep branches when it enters the cubital fossa. GAS 814; N 452; McM 162

132 A 43-year-old man visits the outpatient clinic with a painful shoulder. Physical examination reveals a painful arc syndrome due to supraspinatus tendinopathy. Which of the following conditions are expected to be present during physical examination as the patient abducts his arm? A. Painful abduction 0 to 15 degrees B. Painful abduction 0 to 140 degrees C. Painful abduction 70 to 140 degrees D. Painful abduction 15 to 140 degrees E. Painful abduction 40 to 140 degrees

132 A. The supraspinatus initiates abduction of the arm during the first 15 degrees of abduction; palpation of the tendon during this phase would result in pain from a tendinopathy of the supraspinatus. GAS 717; N 411; McM 132

133 A 54-year-old woman is admitted to the hospital after falling from a tree with an outstretched hand. Radiographic examination reveals a wrist dislocation. Which of the following carpal bones will most likely be involved? A. Scaphoid-lunate B. Trapezoid-trapezium C. Hamate-lunate D. Pisiform-triquetrum E. Hamate-capitate

133 A. The hallmark fracture caused by a fall on an outstretched hand is a scaphoid-lunate fracture; the scaphoid and lunate are the two wrist bones most proximal to the styloid process of the radius. All the other wrist bones are less likely to be affected by this injury. GAS 793-794, 797; N 439; McM 122

134 A 62-year-old man is admitted to the emergency department after falling on wet pavement. Radiographic examination reveals a carpometacarpal fracture at the base of the thumb. What is the term applied to the described fracture? A. Colles' fracture B. Scaphoid fracture C. Bennett's fracture D. Smith's fracture E. Boxer's fracture

134 C. Bennett's fracture is a carpometacarpal fracture at the base of the thumb. Smith's fracture is also called a reverse Colles' fracture and is caused when the distal fragment of the radius angles forward. Colles' fracture is also called "silver fork deformity" because the distal fragment is displaced posteriorly. Boxer's fractures of the necks of metacarpal bones are fractures to the fingers. A scaphoid fracture would be indicated by pain in the anatomical snuffbox. GAS 793-796; N 439; McM 122

135 A 23-year-old woman is participating in a dry skislope competition. The woman is admitted to the emergency department after falling and catching her thumb in the matting. Radiographic and physical examinations reveal rupture of the ulnar collateral ligament of the metacarpophalangeal joint of the thumb. The thumb is extremely painful and an injection of lidocaine is given. What is the most likely diagnosis in this case? A. Gamekeeper's thumb B. Scaphoid fracture C. Bennett's fracture D. Smith's fracture E. Boxer's fracture

135 A. Interestingly, "gamekeeper's thumb" was a term coined to describe an injury common among Scottish gamekeepers who, it is said, killed small animals such as rabbits by breaking their necks between the ground and the gamekeeper's thumb and index finger. The resulting valgus force on the abducted metacarpophalangeal (MCP) joint caused injury to the ulnar collateral ligament. Today this injury is more commonly seen in skiers who land awkwardly with their hand braced on a ski pole, causing the valgus force on the thumb, as seen in this patient. Whereas the term "skier's thumb" is sometimes used, "gamekeeper's thumb" is still in common usage. Bennett's fracture is a fracture at the base of the metacarpal of the thumb. Scaphoid fracture occurs after a fall on an outstretched hand, involving the scaphoid and lunate bone. Colles' fracture is also called silver fork deformity because the distal fragment of the radius is displaced posteriorly. Boxer's fracture is a fracture of the necks of the second and third (and sometimes the fifth) metacarpals. Smith's fracture is also called a reverse Colles' fracture and is caused when the distal radius is fractured and the distal radial fragment is angled forward. GAS 793-796; N 442; McM 163

136 A 54-year-old woman is found unconscious in her car. She is admitted to the hospital, and during physical examination her biceps brachii reflex is absent. What is the spinal level of the afferent component of this reflex? A. C5 B. C6 C. C7 D. C8 E. T1

136 B. The biceps brachii reflex is elicited by tapping on the tendon of the biceps near its insertion on the radius. The biceps brachii reflex involves C5 and C6 spinal nerves. C5 provides the motor component; C6 the afferent side of the reflex arc. GAS 731-732; N 417; McM 148

137 A 54-year-old woman is found unconscious in her bed. She is admitted to the hospital, and during physical examination she has absence of her brachioradialis reflex. The ventral ramus of which spinal nerve is responsible for this reflex? A. C5 B. C6 C. C7 D. C8 E. T1

137 B. The brachioradialis reflex is elicited by tapping the tendon of the brachioradialis muscle. The reflex involves spinal nerves C5, C6, and C7. The major contribution is from C6. GAS 785-787; N 432; McM 150

138 A 55-year-old woman is admitted to the emergency department after a car crash. Physical examination reveals severe pain in the flexor muscles of the forearm, fixed flexion position of the finger, and swelling, cyanosis, and anesthesia of the fingers. Which of the following is the most likely diagnosis? A. Colles' fracture B. Scaphoid fracture C. Bennett's fracture D. Volkmann's ischemic contracture E. Boxer's fracture

138 D. Volkmann's contracture is a flexion deformity of the fingers and sometimes the wrist from an ischemic necrosis of the forearm flexor muscles. Bennett's fracture is a fracture at the base of the metacarpal of the thumb. Scaphoid fracture occurs after a fall on an outstretched hand and involves the scaphoid and lunate bones. Colles' fracture is also called silver fork deformity because the distal fragment of the radius is displaced posteriorly. Boxer's fracture is a fracture of the necks of the second and third (and sometimes the fifth) metacarpals. Smith's fracture is also called a reverse Colles' fracture and is caused when the distal radius is fractured, with the radial fragment angled forward. GAS 774; N 432; McM 150

139 A 62-year-old man visits the outpatient clinic with pain after falling on his outstretched hand. Radiographic examination reveals a fracture of the pisiform bone and hematoma of the surrounding area. Which of the following nerves will most likely be affected? A. Ulnar B. Radial C. Median D. Deep ulnar E. Deep radial

139 D. The ulnar nerve enters the forearm by passing between the two heads of the flexor carpi ulnaris and descends between and innervates the flexor carpi ulnaris and flexor digitorum profundus (medial half) muscles. It enters the hand superficial to the flexor retinaculum and lateral to the pisiform bone, where it is vulnerable to damage and provides the deep ulnar branch. The deep branch of the radial nerve arises proximally in the forearm. GAS 784; N 464; McM 149

14 A 45-year-old man is admitted to the hospital after accidentally walking through a plate glass door in a bar while intoxicated. Physical examination shows multiple lacerations to the upper limb, with inability to flex the distal interphalangeal joints of the fourth and fifth digits. Which of the following muscles is most likely affected? A. Flexor digitorum profundus B. Flexor digitorum superficialis C. Lumbricals D. Flexor digitorum profundus and flexor digitorum superficialis E. Interossei

14 A. The flexor digitorum profundus muscle is dually innervated by the ulnar nerve to the medial two fingers and the median nerve for the long and index fingers. Because of the superficial course of the ulnar nerve, it is vulnerable to laceration. Such an injury would result in an inability to flex the distal interphalangeal joints of the fourth and fifth digits because the flexor digitorum profundus muscle is the only muscle that flexes this joint. The flexor digitorum superficialis muscle is innervated by the median nerve only, and the course of this nerve runs too deep to be usually affected by lacerations. The lumbricals function to flex the MP joints and assist in extending the IP joints. The interossei adduct and abduct the fingers. GAS 781; N 433; McM 160

14. A 12-year-old boy walks in; he fell out of a tree and fractured the upper portion of his humerus. Which of the following nerves are intimately related to the humerus and are most likely to be injured by such a fracture? (A) Axillary and musculocutaneous (B) Radial and ulnar (C) Radial and axillary (D) Median and musculocutaneous (E) Median and ulnar

14. The answer is C. The axillary nerve passes posteriorly around the surgical neck of the humerus, and the radial nerve lies in the radial groove of the middle of the shaft of the humerus. The ulnar nerve passes behind the medial epicondyle, and the median nerve is vulnerable to injury by supracondylar fracture of the humerus, but these nerves lie close to or in contact with the lower portion of the humerus. The musculocutaneous nerve is not in direct contact with the humerus.

140 A 32-year-old woman visits the outpatient clinic after injuring her elbow falling from her bicycle. Physical examination reveals a "benediction attitude" of the hand with the index and long fingers extended and the ring and little fingers flexed. Which of the following is the most likely diagnosis? A. Injury to median and radial nerves B. Injury to median nerve C. Injury to radial and ulnar nerves D. Injury to ulnar nerve E. Injury to median ulnar and radial nerves

140 B. "Benediction attitude" of the hand with the index and long fingers straight and the ring and little fingers flexed is caused by an injury to the median nerve. The long flexors of the digits are supplied by the median nerve; the unopposed radial nerve and deep ulnar nerve supply the extensors of the digits 1-3, causing them to be in the extended position. Digits 4 and 5 are slightly flexed because the flexors of the proximal interphalangeal joints are supplied by the ulnar nerve. GAS 784, 417; N 463; McM 157

141 A 54-year-old man is admitted to the emergency department with severe chest pain. Electrocardiographic evaluation reveals a myocardial infarction. Due to the severity of the infarction, a coronary artery bypass surgery using a radial artery graft is proposed. Which of the following tests should be performed during physical examination before the bypass graft operation? A. Allen test B. Triceps reflex C. Tinel test D. Brachioradialis reflex E. Biceps reflex

141 A. The Allen test involves compression of the radial and ulnar arteries at the wrist with the fingers flexed tightly to move the blood out the palm. Pressure is then released on the radial and ulnar arteries successively to determine the degree of supply to the hand by either vessel and the patency of the anastomoses between them. The usefulness of the radial artery for bypass can thereby be assessed. If the the palm does not flush with blood when the radial artery is released, then the ulnar artery is not sufficient to supply the hand if the radial artery is harvested for a graft. The other tests have nothing to do with the patency of the radial artery. GAS 814; N 435; McM 160

142 A 34-year-old man visits the outpatient clinic with a painful upper limb after a fall onto a concrete floor. Physical examination reveals that the patient has weak abduction and adduction of his fingers but has no difficulty in flexing them. The patient also has decreased sensation over the palmar surface of the fourth and fifth fingers. Which of the following diagnoses is most likely? A. Compression of the median nerve in the carpal tunnel B. Injury of the radial nerve from fractured humerus in the radial tuberosity C. Compression of the median nerve as it passes between the two heads of the pronator teres D. Compression of the radial nerve from the supinator E. Injury of the ulnar nerve by a fractured pisiform

142 E. The ulnar nerve enters the hand superficial to the flexor retinaculum and lateral to the pisiform bone and innervates all the interossei via the deep branch. These muscles are responsible for adduction and abduction of the fingers. Flexion of the fingers is spared because the flexor digitorum superficialis and most of the flexor digitorum profundus are innervated by the median nerve, which is unaffected by this injury. Had the median nerve been compressed in the carpal tunnel, one would have difficulty with motion of the thumb as a result of a lack of innervation of the thenar muscles. An injury of the radial nerve in the arm results in extension deficit in the forearm and hand. GAS 814; N 452; McM 158

143 A 65-year-old man is admitted to the emergency department after falling on his outstretched hand. The patient complains of severe right shoulder pain. Upon physical examination, the patient holds his arm externally rotated and slightly abducted. There is also flattening and sensory loss over the right deltoid muscle. Which of the following is the most likely diagnosis? A. Anterior dislocation of the humerus B. Acromioclavicular joint subluxation C. Clavicular fracture D. Spiral fracture of the humeral midshaft E. Rotator cuff tear

143 A. The glenohumeral joint is an extremely mobile joint with a wide range of movement. Anterior dislocation is the most common. Anterior dislocations of the humerus usually follow injuries where abnormal force is applied to the shoulder while the arm is extended, abducted, and externally rotated. When the head of the humerus is displaced anteriorly and inferiorly, there is flattening of the deltoid prominence (due to the increased weight of the humerus pulling on the muscle), protrusion of the acromion, and anterior axillary fullness (due to the movement of the humeral head into this location). The most commonly injured nerve is the axillary nerve, which innervates the teres minor and deltoid and also provides cutaneous supply to the posterior arm and the skin overlying the deltoid muscle. Acromioclavicular joint subluxation typically results from a blow to the tip of the shoulder when the arm is at the side and slightly adducted. It produces swelling and superior displacement of the clavicle. It is not associated with specific major nerve injuries or sensory deficits. The clavicle is a commonly fractured bone typically after direct trauma. Most fractures occur in the middle third of the clavicle. There is local swelling and tenderness but rarely any neurovascular damage. A spiral humerus midshaft fracture may result from a fall on an outstretched hand. The radial nerve is commonly fractured as it runs in the radial groove. Rotator cuff tears usually occur when there is some degenerative injury to the tendons. The rotator cuff is made up of the subscapularis, supraspinatus, infraspinatus, and teres minor and tendons. GAS 707; N 422, 424; McM 136

144 A 4-year-old boy is brought to the emergency department after falling while holding hands and walking with his two parents. The boy cannot move his right upper extremity because any movement produces pain, and he holds it at his side with his elbow extended and forearm pronated. There are no visible hematomas or swelling. Which of the following structures is most likely injured in this patient? A. Anular ligament B. Biceps brachii tendon C. Interosseous membrane D. Radial collateral ligament E. Ulnar collateral ligament

144 A. The patient is experiencing radial head subluxation ("nursemaid's elbow"), the most common elbow injury in children. The injury often results from a sharp pull on the hand while the forearm is pronated and the elbow is extended. The underdevelopment of the radial head and the laxity of the anular ligament allows for the radial head to sublux (partially dislocate) from this cuff of tissue. This condition is extremely painful but can be easily treated with supination and compression of the elbow joint. Although it is uncommon for muscle tendons to rupture, the most common is the tendon of the long head of the biceps brachii. It produces a characteristic deformity when flexing the elbow: an extremely prominent bulge of unattached muscle belly called the "Popeye sign." The interosseous membrane is an expansive sheet of connective tissue that connects the radius and ulna at their midsection. It serves as an attachment site for the muscles of the forearm. The radial collateral ligament lies on the lateral side of the elbow joint reinforcing the radiohumeral joint. The ulnar collateral ligament lies on the medial side of the elbow joint reinforcing the ulnohumeral joint. GAS 764, 766; N 424; McM 146

145 An emergency department physician examines a patient who fell from a motorcycle and injured his shoulder. The clinician notices a loss of the normal contour of the shoulder and an abnormal-appearing depression below the acromion. Which of the following injuries did the patient most likely sustain? A. Avulsion of the coronoid process B. Dislocated shoulder joint C. Fracture of the midshaft of the humerus D. Fracture of the surgical neck of the humerus E. Laceration of the axillary branch of the posterior cord

145 B. The glenohumeral joint is an extremely mobile joint with a wide range of motion. Anterior dislocation of the humerus is most common and usually associated with an isolated traumatic incident. When the head of the humerus is displaced anteriorly and inferiorly, flattening of the deltoid prominence occurs, leading to loss of the normal contour of the humerus. There is protrusion of the acromion, and the slope of the shoulder lateral to the acromion is depressed and has a "dented" appearance. Avulsion of the coronoid process of the ulna usually occurs with elbow hyperextension, which affects the shoulder joint. A fracture of the midshaft of the humerus damages the radial nerve. Although a fracture to the surgical neck of the humerus and a laceration to the axillary part of the posterior cord affect the axillary nerve, which innervates the deltoid muscle, there will not be any depression beneath the acromion in either case. GAS 707; N 422, 424; McM 136

146 A 22-year-old woman who is in training to become a phlebotomist is performing venipuncture on another student. She places the needle into the median cubital vein but is unable to withdraw blood. She quickly realizes that she passed the needle completely through the vein. Which of the following structures located deep to the median cubital vein has acted as a barrier and has prevented her from puncturing an artery? A. Flexor retinaculum B. Pronator teres muscle C. Bicipital aponeurosis D. Brachioradialis muscle E. Biceps brachii tendon

146 C. The median cubital vein is a superficial vein that lies on the biceps brachii aponeurosis. The biceps brachii aponeurosis, also known as lacertus fibrosus, is a flat sheet of connective tissue that fans out from the medial side of the biceps brachii tendon to blend with the deep fascia of the biceps brachii muscle. It reinforces the cubital fossa and protects the brachial artery, which runs beneath it. GAS 768-769; N 403; McM 148

147 A 21-year-old painter sustains a laceration on the anterior surface of his left wrist just distal to the skin fold crease. When he arrives at the emergency department, the physician extends the patient's wrist to determine the depth of the laceration and observes a broad, glistening white structure deep to the superficial fascia. The patient has no numbness or tingling of any of the fingers and is able to discriminate sharp/dull sensation in all of the fingers and palm of the hand. There is no loss of motion in any of the fingers or the hand, and grip strength is normal. Which structure is the physician most likely observing? A. Flexor retinaculum B. Flexor carpi ulnaris tendon C. Palmar skin D. Flexor digitorum superficialis tendons E. Flexor digitorum profundus tendons

147 A. The flexor retinaculum is a thick connective tissue ligament that spans the space between the medial and lateral sides of the base of the carpal tunnel. It protects and stabilizes the tendons that run beneath it. Damage to the flexor carpi ulnaris tendon, flexor digitorum superficialis tendons, and flexor digitorum profundus tendons result in functional losses in the hand. The palmar skin is loose connective tissue and does not have a shiny, glistening appearance (GAS Fig. 7-103). GAS 798; N 449; McM 158

148 While using a wood-carving gouge, a 34-year-old woman lacerates the proximal aspect of her palm from the base of the thumb across to the pisiform bone. Neurological examination reveals pronounced weakness in opposition of the thumb, with intact sensation in the hand. Which of the following injuries best accounts for her findings? A. Injury of the median nerve in the carpal tunnel B. Injury of the superficial palmar branch of the median nerve C. Injury of the recurrent and superficial branch of the median nerve D. Injury of the recurrent median nerve at the wrist E. Injury of the radial and ulnar nerves

148 D. The recurrent branch of the median nerve usually originates from the lateral side of the median nerve at the distal margin of the flexor retinaculum. It innervates the three thenar muscles: the opponens pollicis, flexor pollicis brevis, and abductor pollicis brevis muscles. Injury of the median nerve in the carpal tunnel, as well as injury of the recurrent and superficial branch of the median nerve, causes both sensory and motor deficits. Injury of the superficial palmar branch of the median nerve results in loss of sensation only of the palm. Injury to the radial and ulnar nerves results in a greater number of sensory and motor deficits in the distribution of theses nerves (GAS Fig. 7-103). GAS 817; N 463; McM 157

149 A 22-year-old football player suffered a wrist injury after falling on his outstretched hand. When the anatomical snuffbox is exposed in surgery, an artery is visualized crossing the fractured bone that provides a floor for this space. Which of the following arteries was most likely visualized? A. Ulnar B. Radial C. Anterior interosseous D. Posterior interosseous E. Deep palmar arch

149 B. The radial artery enters the anatomical snuffbox as it passes to the posterior aspect of the hand to pass between the two heads of the 1st dorsal interosseous muscle. The ulnar artery continues anteriorly and enters the hand on the palmar surface. The anterior and posterior interosseous arteries are found anteriorly and posteriorly, respectively, on the interosseous membrane, which is located between the radius and ulna. The deep palmar arch is an anastomosis on the palmar surface of the hand that is formed by the radial artery and the deep branch of the ulna artery and lies on the anterior surface of the hand. GAS 782, 800, 810-815; N 454; McM 161

15 A 24-year-old man is admitted with a wound to the palm of his hand. He cannot touch the pad of his index finger with his thumb but can grip a sheet of paper between all fingers and has no loss of sensation on the skin of his hand. Which of the following nerves has most likely been injured? A. Deep branch of ulnar B. Anterior interosseous C. Median D. Recurrent branch of median E. Deep branch of radial

15 D. The recurrent branch of the median nerve is motor to the muscles of the thenar eminence, which is an elevation caused by the abductor pollicis brevis, flexor pollicis brevis, and opponens pollicis muscles. If the opponens pollicis is paralyzed, one cannot oppose the pad of the thumb to the pads of the other digits because this is the only muscle that can oppose the thumb by moving the first metacarpal on the trapezium. The recurrent branch does not have a cutaneous distribution. Holding a piece of paper between the fingers is a simple test of adduction of the fingers. These movements are controlled by the deep branch of the ulnar nerve, which is not injured in this patient. GAS 817; N 460; McM 159

15. A man injures his wrist on broken glass. Which of the following structures entering the palm superfi cial to the fl exor retinaculum may be damaged? (A) Ulnar nerve and median nerve (B) Median nerve and fl exor digitorum profundus (C) Median nerve and fl exor pollicis longus (D) Ulnar artery and ulnar nerve (E) Ulnar nerve and fl exor digitorum superfi cialis

15. The answer is D. Structures entering the palm superfi cial to the fl exor retinaculum include the ulnar nerve, ulnar artery, palmaris longus tendon, and palmar cutaneous branch of the median nerve. The median nerve, the fl exor pollicis longus, and the fl exor digitorum superfi cialis and profundus run deep to the fl exor retinaculum.

150 A 36-year-old man is brought to the emergency department because of a deep knife wound on the medial side of his distal forearm. He is unable to hold a piece of paper between his fingers and has lost sensation from the fifth digit and the medial side of the fourth digit. Which of the following nerves is most likely injured? A. Axillary B. Median C. Musculocutaneous D. Radial E. Ulnar

150 E. The ulnar nerve is responsible for cutaneous innervation to the medial one and a half digits and motor innervation to most of the intrinsic muscles of the hand including the interossei. The interossei muscles are responsible for adduction of the digits, which is the action that would be used to hold a piece of paper between the fingers. The median nerve supplies cutaneous innervation to the lateral three and a half fingers and the thenar eminence and lateral two lumbricals. These muscles function to oppose the thumb and flex the MP joints, respectively. The musculocutaneous nerve is responsible for innervation of the anterior compartment of the arm, and muscular nerve fibers of this nerve would not be damaged by a wound in the distal forearm. The radial nerve supplies the dorsum of the hand, with sensation and extension function of the forearm muscles, and damage will not lead to this array of symptoms (GAS Fig. 7-109). GAS 814, 815; N 464; McM 159

151 A 28-year-old telephone company worker falls off a street pole during a telephone line repair and lands directly on his right shoulder. Plain radiographs reveal a vertical fracture through the entire length of the floor of the intertubercular sulcus of the right humerus. The muscle that is most likely affected by the fracture is innervated by a nerve that is composed of which of the following nerve roots? A. C3 and C4 B. C6 to C8 C. C4 and C5 D. C2 to C4 E. C5 to C7

151 B. The muscle that attaches into the intertubercular sulcus of the humerus is the latissimus dorsi. Nerve supply is via the thoracodorsal nerve, which is a branch of the posterior cord and is made up of roots C6-8. Nerves C2, C3 and C4 are not part of the brachial plexus but of the cervical plexus and will supply the "strap" muscles. Nerves C4 and C5 are the main contributions to the phrenic nerve, and C5 does not contribute to the formation of the thoracodorsal nerve. GAS 728; N 416; McM 115

152 A 21-year-old woman who is an athlete dislocated her glenohumeral joint while playing soccer and the shoulder was reduced in the emergency department. However, after 1 week the physician noted that the woman had lost strength when she attempted internal rotation of her arm at the shoulder. This finding was most likely caused by a tear in which of the following muscles? A. Infraspinatus B. Pectoralis minor C. Subscapularis D. Supraspinatus E. Teres minor

152 C. Anterior dislocation of the humerus may damage the nerves located in the axilla or cause tears in the rotator cuff muscles. Internal rotation is the primary function of subscapularis muscle; with this being the only action impaired it is the most likely damaged muscle, probably as a result of injury to the upper and/or lower subscapular nerves that innervate this muscle. The infraspinatus and trees minor muscles are external rotators, and the supraspinatus muscle is the abductor of the arm from 0 to 15 degrees. The pectoralis major is a flexor, adductor, and medial rotator and would not likely be damaged during a shoulder dislocation. GAS 712; N 411; McM 136

153 A 29-year-old man presents with difficulty with fine motor control in his hand. A few weeks ago he fell from a ladder; as he was falling he reached out and grabbed a limb of a tree. Examination reveals a deficit in his ability to abduct and adduct his digits and inability to oppose his thumb on his right hand. Which of the following was most likely injured? A. Lower trunk of the brachial plexus B. Median nerve C. Musculocutaneous nerve D. Ulnar nerve E. Upper trunk of the brachial plexus

153 A. The nerve responsible for innervation of the interosseus muscles that are weakened in this patient is the deep branch of the ulnar nerve. Innervation of the muscles responsible for opposition of the thumb is via the recurrent branch of the median nerve. Both of these nerves are formed by the C8 and T1 ventral rami, which combine to form the inferior trunk of the brachial plexus. Damage to either the median or ulnar nerves would not produce both of these symptoms. Median nerve damage would involve all of the flexors of the wrist except the flexor carpi ulnaris and most digits except for the interphalangeal joints of the 4th and 5th fingers. It will also result in loss of function of the thumb entirely. Ulnar nerve damage will result in weakness of the medial half of flexor digitorum profundus (4th and 5th interphalangeal joint flexion), as well as the intrinsic muscles of the hand except for the lateral two lumbricals. GAS 814; N 416; McM 157

154 An emergency department physician evaluates a 28-year-old man who injured his hand in a knife fight. The physician notes that the ring and little fingers cannot be extended at the interphalangeal joints, and the patient cannot spread the fingers of his injured hand. Weakness of which of the following muscles is the major reason for the loss of interphalangeal extension of the medial two fingers? A. Dorsal interosseus muscles B. Extensor digitorum C. Lumbrical muscles D. Palmar interosseus muscles E. Extensor digiti minimi

154 C. The patient likely has damage to the ulnar nerve, which affected both the interossei and medial two lumbricals. The lumbricals extend the interphalangeal joints of the ring and little fingers, while the interossei are responsible for abduction and adduction of the digits. The dorsal interossei are responsible for abduction, while the palmar interossei are responsible for adduction of the digits. The extensor digit minimi is responsible for extension of the little finger only and if damaged will not affect the ring finger. If the extensor digitorum were damaged it would lead to weakness of all four digits, not only the ring and little fingers. GAS 814; N 488; McM 159

155 After a fall on her outstretched arm, a 72-year-old woman presents with elbow pain. Physical examination reveals a palpable defect over her biceps brachii tendon. Elbow flexion causes pain but does not limit active movement. Radiographs do not show fractures or dislocations. She is diagnosed with a biceps brachii tendon rupture. Which of the following muscles most likely allow the patient to continue to flex her elbow? A. Brachialis and brachioradialis B. Flexor carpi ulnaris and flexor carpi radialis C. Flexor digitorum superficialis and flexor digitorum profundus D. Pronator teres and supinator E. Triceps brachii and coracobrachialis

155 A. Flexion of the elbow is achieved by contraction of the biceps brachii, brachialis, and brachioradialis muscles. The brachialis muscle is the major flexor of the elbow joint and together with the brachioradialis will continue to achieve flexion if the biceps brachii is damaged. The flexor carpi ulnaris and radialis produce flexion of the wrist, and the flexor digitorum superficialis and profundus produce flexion of the digits at the metacarpophalangeal and interphalangeal joints, respectively. The pronator teres and supinator are responsible for pronation and supination, respectively. The coracobrachialis does not cross the elbow joint and acts only on the shoulder, while the triceps brachii is the elbow extensor. GAS 755; N 417; McM 150

156 A 16-year-old girl is brought to the emergency department after attempting suicide by cutting her wrist. The deepest part of the wound is between the tendons of the flexor carpi radialis and the flexor digitorum superficialis. This patient is most likely to have a deficit of which of the following? A. Adduction and abduction of the fingers B. Extension of the index finger C. Flexion of the ring and little finger D. Sensation over the base of the little finger E. Opposition of the thumb and slightly weakened flexion of the second and third digits

156 E. Opposition, a complex movement, begins with the thumb in the extended position and initially involves abduction and medial rotation of the first metacarpal. This is produced by the action of the opponens pollicis muscle at the carpometacarpal joint by the flexor pollicis brevis muscle and then by flexion at the metacarpophalangeal joint. The opponens pollicis and flexor pollicis brevis muscles are supplied by the recurrent branch of the median nerve (C8, T1). The median nerve is the principal nerve of the anterior compartment of the forearm and the thenar muscles of the hand. It passes through the carpal tunnel with the tendons of the flexor digitorum profundus, flexor digitorum superficialis, and flexor pollicis longus to supply the thenar muscles of the hand. Abduction and adduction of the fingers is done by the palmar and dorsal interossei muscles, which are supplied by the median nerve. The extensor indicis extends the index finger. The ring and little fingers are flexed by the medial two tendons of the flexor digitorum superficialis (supplied by the median nerve) and the medial two tendons of the flexor digitorum profundus (supplied by the ulnar nerve). GAS 817; N 452; McM 159

157 A 36-year-old man presents to the emergency department with pain and tenderness in his right wrist after a fall on his outstretched hand two days ago. On examination there is tenderness on the lateral side of the wrist, just proximal to the base of the first metacarpal. What is the most likely diagnosis? A. Fracture of the first metacarpal B. Fracture of the trapezium C. Tenosynovitis of thumb extensors D. Fracture of the scaphoid E. First carpometacarpal joint arthritis

157 D. The scaphoid is the most frequently fractured carpal bone. Fracture often results from a fall on the palm when the hand is abducted, the fracture occurring across the narrow part ("waist") of the scaphoid. Pain occurs primarily on the lateral side of the wrist, especially during dorsiflexion and abduction of the hand. If the only blood supply to the scaphoid enters the bone distally, avascular necrosis (pathological death of bone resulting from inadequate blood supply) of the proximal fragment of the scaphoid may occur and produce degenerative joint disease of the wrist. Thumb metacarpal fractures are usually caused by an axial blow directed against the partially flexed metacarpal. Tenosynovitis is an infection of the digital synovial sheaths. Symptoms of tenosynovitis include pain, swelling, and difficulty moving the particular joint where the inflammation occurs. Carpometacarpal joint arthritis is a degenerative joint disease affecting the first carpometacarpal joint. GAS 79; N 488; McM 159

158 A 55-year-old man is admitted to hospital after blunt trauma at the junction of his neck and shoulder on the right side. Examination reveals winging of the scapula and partial paralysis of the right side of the diaphragm. Which part of the brachial plexus has been injured? A. Cords B. Divisions C. Ventral rami D. Terminal branches E. Trunks

158 C. The long thoracic nerve arises from the upper three ventral rami to the brachial plexus (C5 to C7) and supplies the serratus anterior, which protracts the scapula. The diaphragm is innervated by the phrenic nerve, which also arises from ventral rami (C3-C5). GAS 727, 741; N 413; McM 140

159 A 55-year-old right-handed woman presents to the clinic with a 1-week history of right elbow pain. The pain started after a long game of competitive tennis. The pain begins in the elbow and at times radiates into the forearm. Splinting of the elbow decreases the intensity of the pain. During physical examination of the elbow mild swelling and tenderness are noted over the lateral epicondyle. Which one of the following wrist movements, if carried out by the patient with a closed fist and against resistance, will most likely exacerbate the pain? A. Radial deviation B. Ulnar deviation C. Flexion D. Extension E. Flexion and ulnar deviation

159 D. The lateral epicondyle is the common extensor origin. Most of the extensor muscles of the forearm originate from this area. Putting those muscles in action will exacerbate pain on the lateral epicondyle, a condition nicknamed "tennis elbow." Radial and lateral deviations have no effect because the movement is at the wrist joint. Flexion exacerbates pain on the medial epicondyle if the patient has "golfer's elbow." GAS 752, 785, 768; N 427; McM 145

16 A 55-year-old man is examined in a neighborhood clinic after receiving blunt trauma to his right axilla in a fall. He has difficulty elevating the right arm above the level of his shoulder. Physical examination shows that the inferior angle of his right scapula protrudes more than the lower part of the left scapula. The right scapula protrudes far more when the patient pushes against the examiner's hand with resistance. Which of the following neural structures has most likely been injured? A. The posterior cord of the brachial plexus B. The long thoracic nerve C. The upper trunk of the brachial plexus D. The site of origin of the middle and lower subscapular nerves E. Spinal nerve ventral rami C7, C8, and T1

16 B. The condition described in this patient is called "winging" of the scapula. "Winging" of the scapula occurs when the medial border of the scapula lifts off the chest wall when the patient pushes against resistance, such as a vertical wall. The serratus anterior muscle holds the medial border of the scapula against the chest wall and is innervated by the long thoracic nerve. The serratus anterior assists in abduction of the arm above the horizontal plane by rotating the scapula so that the glenoid fossa is directed more superiorly. GAS 727; N 413; McM 141

160 Following a difficult delivery, a 3-day-old infant girl showed limited movement of the right upper limb, with the arm adducted and internally/medially rotated, the forearm extended at the elbow and pronated, and the wrist slightly flexed. Tearing of fibers in which ventral rami of the brachial plexus best accounts for these symptoms? A. C5 and C6 B. C6 and C7 C. C7 and C8 D. C8 and T1 E. C5 to T1

160 A. Injuries to superior parts of the brachial plexus (C5-C6) usually result from an excessive increase in the angle between the neck and shoulder during a difficult delivery. Injury to the superior trunk of the plexus is apparent by the characteristic position of the limb ("waiter's position"), in which the limb hangs by the side in medial rotation. Injuries to the lower trunk of the brachial plexus (Klumpke paralysis) are much less common. These events injure the inferior trunk of the brachial plexus (C8 and T1) and may avulse the roots of the spinal nerves from the spinal cord. The short muscles of the hand are affected, and a claw hand results (GAS Fig. 7-52A). GAS 738; N 452; McM 159

161 A 48-year-old woman is seen in the orthopedic clinic with symptoms of carpal tunnel syndrome. This could result in weakening of which muscles? A. Dorsal and palmar interossei B. Lumbricals III and IV C. Thenar and lumbricals I and II D. Flexor digitorum superficialis and profundus E. Hypothenar

161 C. Carpal tunnel syndrome is a relatively common condition that causes pain, numbness, and a tingling sensation in the hand and fingers. Carpal tunnel syndrome is caused by compression of the median nerve, which supplies the thenar muscles and the first and second lumbricals. Dorsal and palmar interossei and the hypothenar muscles are supplied by the ulnar nerve. The flexor muscles of the forearm are supplied by the median nerve before it passes through the carpal tunnel. GAS 798; N 449; McM 159

162 A 24-year-old man complains of inability to button his shirt. Examination reveals that he can still grip a sheet of paper between his second and third fingers and there is no sensory deficit in the hand. Which nerve has been affected? A. Deep branch of ulnar B. Anterior interosseous C. Median D. Recurrent branch of median E. Deep branch of radial

162 D. Recurrent branch of the median is the correct answer. This nerve, which is a branch of the median nerve, is given off after the median nerve passes through the carpal tunnel. The nerve supplies the thenar muscles. The opponens pollicis muscle, which is part of the thenar muscle group, is used while buttoning a shirt, an action that requires thumb opposition. The deep branch of the ulnar nerve supplies motor innervations to all the intrinsic muscles of the hand except the lateral two lumbricals and sensation to the medial one and a half fingers on both the palmar and dorsal sides. The patient can still grip a paper between the second and third digits, a function largely performed by the interossei muscles, which are innervated by the deep branch of the ulnar nerve. The deep branch of the radial nerve is motor to the long extensors of the wrist and fingers. GAS 817; N 452; McM 157

163 A 22-year-old man accidentally smashes his hand through a window. He is cut across the entire length of the distal transverse crease on the anterior surface of the wrist. The cut is down to the surface of the flexor retinaculum but not into it. During physical examination which is one of the neuromuscular deficits that will be found? A. Weakened pronation of the forearm B. Inability to abduct the thumb C. Weakened flexion of thumb D. Weakened opposition of the thumb E. Inability to adduct the thumb

163 E. Inability to adduct the thumb is the correct answer because the ulnar nerve travels superficial to the flexor retinaculum and innervates the adductor pollicis muscle, which adducts the thumb. Pronation of the forearm is carried out by muscles innervated by the median nerve, and abduction of the thumb is performed by muscles innervated by the median and radial nerves. Flexion and opposition of the thumb are performed by muscles innervated by the median nerve and would not be injured, as the median nerve travels deep to the flexor retinaculum. GAS 814; N 452; McM 157

164 A 36-year-old woman is admitted to the emergency department after an athletic injury that has caused weakness in both lateral rotation and the initial 15 to 20 degrees of abduction of the arm. Which nerve was most probably injured? A. Lower subscapular B. Axillary C. Radial D. Suprascapular E. Upper subscapular

164 D. The supraspinatus is innervated by the suprascapular nerve (C5, C6) and the nerve continues through the spinoglenoid notch and innervates the infraspinatus. The supraspinatus initiates abduction of the arm up to the first 15 to 20 degrees. The subscapular nerve supplies the subscapularis and teres major muscles, which are medial rotators of the arm. The axillary nerve supplies the deltoid and teres minor muscles and also a patch of skin on the lateral side of the shoulder. The deltoid abducts the arm beyond 20 degrees, and the teres minor muscle, although a lateral rotator, does not abduct the arm. The radial nerve supplies muscles in the posterior compartments of the arm and forearm, which are extensors of the elbow, wrist, and fingers in that order. The upper subscapular nerve supplies the subscapularis, a medial rotator of the arm. GAS 717, 742; N 413; McM 138

165 A 35-year-old male carpenter suffered a deep cut to the tip of his thumb. Initially the redness, swelling, and pain were limited to the injured part of the thumb, but later the entire thumb and thenar eminence became inflamed. Which group of lymph nodes is the first to receive drainage from this injury? A. Posterior axillary B. Subclavian C. Lateral axillary D. Anterior axillary E. Central axillary

165 C. With the involvement of the thenar muscles, lymph drains initially to the epitrochlear nodes and then to the lateral (humeral) nodes. The posterior axillary nodes receive lymph from the upper back and shoulder. The subclavian nodes receive lymph from all the axillary nodes. The anterior axillary nodes (pectoral nodes) receive lymph from most of the breast and the upper side of the anterolateral chest wall. All anterior, lateral, posterior, and medial axillary nodes drain to the central axillary nodes. GAS 748; N 403; McM 364

166 A mother tugs violently on her child's arm to pull him out of the way of an oncoming car and the child screams in pain. The child is admitted to the emergency department and radiographic examination reveals a dislocated head of the radius resulting from the radial head slipping out past which ligament? A. Anular B. Joint capsule C. Interosseous membrane D. Radial collateral E. Ulnar collateral

166 A. "Nursemaid's elbow," a condition commonly found in children below 5 years of age, is caused by a sharp pull of the child's hand. In children, the anular ligament, which holds the head of the radius in place, is lax and allows the radial head to sublux when the hand is pulled. Also the radial head is small, so the anular ligament does not have a good "grip" on the hand. The joint capsule of the radioulnar joint is not attached to the radius; rather it passes around the neck of the radius inferiorly to attach to the coronoid process of the ulna. The interosseous membrane binds the radius and ulna together and does not maintain stability of the joint. The radial collateral ligament attaches the lateral side of the head of the radius to the lateral condyle of the humerus. The ulnar collateral ligament attaches the medial side of the ulnar head to the medial condyle of the humerus. GAS 766; N 424; McM 146

167 A 62-year-old woman is seen in the outpatient clinic. A photograph of her hand is shown in Figure 6-10. A radiograph reveals a hairline fracture of the hamate at Guyon's canal. Which of the following will also be present during physical examination? A. Numbness and weakness of the little and ring fingers B. Wrist drop C. Atrophy of the thenar muscles D. Positive Tinel's test E. Trouble turning her forearm outward

167 A. The patient has a classic claw hand due to the damage of the deep branch of the ulnar nerve by the fractured hamate at Guyon's canal. This nerve supplies the intrinsic muscles of the hand except the lateral two lumbricals and the thenar muscles. This nerve also supplies cutaneous innervations to the medial one and a half fingers (ring and little fingers) in the palmar and dorsal sides. The lumbricals and interossei insert at the back of the fingers via the dorsal (extensor) hood. This hood extends from the metacarpophalangeal joint to the distal phalanx. Through this mechanism, the muscles flex the metacarpophalangeal joint and extend the interphalangeal joint. With damage to the deep branch of the ulnar nerve, this function is lost. The result is that there will be flexion of the interphalangeal joints and extension of the metacarpophalangeal joint, giving the appearance as shown in the photograph. GAS 816; N 464; McM 159

168 A 25-year-old woman experiences numbness and tingling in her right arm and hand while carrying a piece of luggage. Physical examination showed no motor or sensory deficits in the upper limb. When asked to abduct her upper limb to 90 degrees and to maintain this position while repeatedly closing and opening her hands, the symptoms are reproduced along the medial border of the limb, from the axilla to the hand. Which nerve structure(s) is/are most likely compressed? A. Ulnar nerve at the medial epicondyle B. Radial nerve at the neck of the radius C. Median nerve in the carpal tunnel D. Inferior trunk of the brachial plexus E. Divisions of the brachial plexus

168 D. In thoracic outlet syndrome—sometimes caused by a cervical rib or a cervical band—ventral rami or trunks of the brachial plexus can be compressed by these structures as they travel from the neck to the axilla. In this case the inferior trunk of the brachial plexus is being compressed by a cervical rib. The anterior division of the inferior trunk continues as the medial cord of the brachial plexus. The medial brachial cutaneous nerve (medial cutaneous nerve of the arm) and medial antebrachial cutaneous nerve (medial cutaneous nerve of the forearm) are branches of the medial cord of the plexus, with the ulnar nerve as its terminal branch. Additionally, there is medial cord contribution to the median nerve. Compression of the inferior cord of the brachial plexus therefore presents with numbness and paraesthesia on the medial part of the arm, forearm, and hand. GAS 150; N 415; McM 140

169 A 50-year-old woman reports to the physician that she is no longer able to play her viola as she used to because of "locking" of her index finger. During physical examination a snap is heard during passive extension of the finger and the consequent flexion of it. What is the most likely diagnosis of this condition? A. Tenovaginitis stenosans (trigger finger) B. Dupuytren's contracture C. Mallet finger D. Boutonniere deformity E. Boxer's fracture

169 A. Tenovaginitis stenosans occurs after swelling or nodular growth of the flexor tendon, which interferes with it gliding through the pulley and producing a snap or click on active extension or flexion. Mallet finger presents with permanent flexion of the distal phalanx from the lateral band of the extensor digitorum avulsion. Boutonnière deformity is due to avulsion of the central band of the extensor digitorum tendon, which presents as abnormal flexion of the middle phalanx and hyperextension of the distal phalanx. Boxer's fracture affects the metacarpals of the second and third digits commonly. Dupuytren's contracture is progressive fibrosis of the palmar aponeurosis and fascia leading to progressive shortening and thickening, eventually leading to permanent partial flexion of the metacarpophalangeal and proximal interphalangeal joints. GAS 802; N 448; McM 157

17 A mother tugs violently on her 4-year-old boy's hand to pull him out of the way of an oncoming car and the child screams in pain. Thereafter, it becomes obvious that the child cannot straighten his forearm at the elbow. When the child is seen in the emergency department, radiographic examination reveals a dislocation of the head of the radius. Which of the following ligaments is most likely directly associated with this injury? A. Anular B. Joint capsular C. Interosseous D. Radial collateral E. Ulnar collateral

17 A. The anular ligament is a fibrous band that encircles the head of the radius, forming a collar that fuses with the radial collateral ligament and articular capsule of the elbow. The anular ligament functions to prevent displacement of the head of the radius from its socket. In a child of this age the head of the radius is almost the same diameter as the shaft of the bone, so the head is relatively easy to dislocate. The joint capsule functions to allow free rotation of the joint and does not function in its stabilization. The interosseous membrane is a fibrous layer between the radius and ulna helping to hold these two bones together. The radial collateral ligament extends from the lateral epicondyle to the margins of the radial notch of the ulnar and the anular ligament of the radius. The ulnar collateral ligament is triangular ligament and extends from the medial epicondyle to the olecranon of the ulna (GAS Figs. 7-73A and 7-72). GAS 764-766; N 424; McM 146

17. A 27-year-old pianist with a known carpal tunnel syndrome experiences diffi culty in fi nger movements. Which of the following intrinsic muscles of her hand is paralyzed? (A) Palmar interossei and adductor pollicis (B) Dorsal interossei and lateral two lumbricals (C) Lateral two lumbricals and opponens pollicis (D) Abductor pollicis brevis and palmar interossei (E) Medial two and lateral two lumbricals

17. The answer is C. The median nerve innervates the abductor pollicis brevis, opponens pollicis, and two lateral lumbricals. The ulnar nerve innervates all interossei (palmar and dorsal), the adductor pollicis, and the two medial lumbricals.

170 A 25-year-old woman fashion model has an unsightly lump on her wrist that is causing her great distress. She also reports a tingling sensation on the lateral three and a half digits of the palmar aspect of her hand. Her doctor uses his pen torch to illuminate the lump and then uses a syringe to drain its contents. What is the most likely diagnosis of this condition? A. Neurofibroma B. Ganglion cyst C. Chondroma D. Osteoma E. Osteophyte

170 B. Ganglion cysts are outpouchings of the joint capsule or tendons and may occur anywhere in the hand or feet. They contain synovial-like fluid and are pliable to touch. They commonly occur on the dorsum of the hand and may be surgically treated if necessary. The others are all solid tumors and cannot be drained. GAS 790, 800-801; N 440; McM 158

171 An 18-year-old man who is a professional cyclist complains of sensory loss to the medial one and a half fingers on the dorsal aspect of his hand. The orthopedic surgeon diagnoses "Handlebar neuropathy." What other signs may be elicited during physical examination? A. Sensory loss of the medial one and a half digits on the palmar aspect of the hand B. Weakness in abduction of the thumb C. Weakness in extension of the thumb D. Thenar muscle atrophy E. Tinel's sign at the scaphoid

171 A. Handlebar or ulnar neuropathy causes sensory loss of both palmar and dorsal aspects of the medial one and a half digits. Abduction of the thumb is by the abductor pollicis longus supplied by the radial nerve and the abductor pollicis brevis supplied by the median nerve. Extensors of the thumb are supplied by the radial nerve. Median nerve palsy can result in thenar muscle atrophy. A Tinel's sign might be observed near the hamate at Guyon's canal but not laterally at the scaphoid. GAS 814-816; N 464; McM 159

172 A 20-year-old man who is a racquetball player reports to the physician's office complaining that he is not able to grip his racquet during practice. During physical examination the physician notes that the patient has atrophy of the thenar eminence, inability to oppose the thumb, and difficulty in flexing the middle interphalangeal joints of the digits. What is the most likely diagnosis of this condition? A. Hypertrophy of the supinator B. Pronator syndrome C. Medial supracondylar fracture D. Tennis elbow E. Golfer's elbow

172 B. Pronator syndrome is due to damage of the median nerve as it passes between the two heads of a hypertrophied pronator teres muscle. It will present with loss of opposition, atrophy of the thenar muscles, and flexion difficulty of the digits and sensory loss of the lateral three and a half digits. Hypertrophy of the supinator muscle will affect the deep branch of the radial nerve that continues distally as the posterior interosseous nerve. A medial supracondylar fracture might affect the ulnar nerve. Tennis elbow affects only the common extensor muscle origin and will not cause flexor or opposition difficulties of the digits and thumb, respectively (GAS Fig. 7-83). GAS 777; N 463; McM 151

173 A 54-year-old man presents to his primary care physician complaining of weakness in his fingers. His attempt to make a ring between his thumb and index finger by bringing the tips together is shown in Figure 6-11. He is able to successfully hold a piece of paper between his thumb and index finger. Pronation and wrist flexion are weakened. Which of the following nerves is most likely affected? A. Ulnar nerve at Guyon's canal B. Median nerve in the carpal tunnel C. Anterior interosseous nerve beneath the ulnar head of pronator teres D. Posterior interosseous nerve beneath the supinator E. Median nerve beneath the bicipital aponeurosis

173 C. The anterior interosseous nerve runs distally and anterior to the interosseous membrane supplying the deep forearm flexors (except the ulnar part of the flexor digitorum profundus muscle, which sends tendons to the fourth and fifth fingers), it passes deep to and supplies the pronator quadratus muscle, hence the weakness in pronation and wrist flexion (GAS Fig. 7-87). GAS 784; N 463; McM 151

174 A 24-year-old man sustained multiple injuries including complex fractures in the right wrist as the result of a motor vehicle collision. After his injuries were stabilized and surgical repairs planned, neurological testing reveals decreased sensation along the medial border of the hand, as well as the little and ring fingers, and decreased strength of thumb adduction and finger adduction/abduction but with intact flexion of the distal interphalangeal joints of the ring and little fingers. If the nerve injury is not repaired, which of the following will become apparent in the affected hand over the next few weeks to months? A. Flattening of the thenar eminence B. Wrist drop C. Radial deviation at the wrist D. Ulnar deviation at the metacarpophalangeal joints E. Prominent metacarpal bones with "guttering" between adjacent metacarpals

174 E. Loss of ulnar nerve innervations eventually leads to atrophy of the interossei muscles, which presents as guttering between the metacarpals. Since the median nerve is intact there is no thenar atrophy. Similarly, if the wrist extensors, which are supplied by radial nerve, are intact, then no wrist drop is observed. Radial deviation is not seen due to action of the extensor carpi ulnaris supplied by the radial nerve. GAS 814-816; N 464; McM 145

175 A 30-year-old intoxicated man stumbled while descending stairs and fell on his outstretched and hyperextended hand. Since the fall (2 to 3 hours ago) he has had constant pain in his wrist, but over the past 30 minutes he has developed tingling and burning pain in his hand as well. Radiographs reveal fractures of both the radial and ulnar styloid processes, as well as dislocation of a carpal bone. Which of the following abnormal sensory and motor findings are most likely to be found on examination? A. Dysesthesia (tingling in response to light touch) along the medial border of the hand and little finger and weakness in adduction of the thumb B. Dysesthesia over the palm and palmar aspect of the thumb, index, and middle fingers and weakness in thumb opposition C. Numbness along the medial border of the hand and little finger and weakness in wrist extension D. Numbness over the dorsum of the hand laterally including the dorsal aspect of the thumb, index, and middle fingers and weakness in grip strength E. Numbness over the palm and palmar aspect of the thumb, index and middle fingers and weakness in adduction of the thumb

175 B. The lunate is the most commonly dislocated carpal bone. It helps to form the floor of the carpal arch. When it is dislocated, it is displaced into the carpal tunnel compressing the median nerve. The patient will then present with dysesthesia over the palm and palmar aspect of the thumb, index and middle fingers, and weakness in thumb opposition. The other options are not symptoms of injury solely to the median nerve. GAS 792-795; N 439; McM 122

176 A 25-year-old man falls on a slippery trail and injures his elbow and hand. Inspection reveals abrasions over the olecranon, medial epicondyle, and palm of the hand. Physical examination reveals decreased sensation with "pins and needles" (paraesthesia) along the ulnar border of the hand and medial one and a half digits. There is also weakness of finger abduction/ adduction, thumb adduction, and flexion at the DIP of the ring and little fingers. Which structure was most likely injured? A. Ulnar nerve at the medial epicondyle B. Ulnar nerve at Guyon's canal C. Median nerve in the cubital fossa D. Median nerve in the carpal tunnel E. Medial cord of brachial plexus in the axillary inlet

176 A. The deficits describe ulnar nerve damage close to its entry into the forearm. The ulnar nerve passes behind the medial epicondyle and is relatively unprotected, making this area prone to nerve injury. In the forearm, via its muscular branches, it innervates the flexor carpi ulnaris muscle and the medial half of the flexor digitorum profundus muscle. In the hand the deep branch of the ulnar nerve innervates the hypothenar muscles, adductor pollicis, abductor digiti minimi, flexor digiti minimi brevis, third and fourth lumbricals, opponens digiti minimi, and palmaris brevis muscles. The sensory innervation is to the fifth and medial half of the fourth digit and corresponding part of the hand, which can explain the deficits experienced by the patient. GAS 784; N 463; McM 151

177 A 29-year-old woman injures her wrist in a fall on an outstretched hand. Examination reveals pain on movement of the wrist associated with numbness and tingling on the radial side of the palm and palmar aspect of the thumb, index, and middle fingers. A radiograph of the wrist reveals, anterior dislocation of a carpal bone. Which dislocated carpal bone is compressing which structure? A. Pisiform compressing ulnar nerve B. Hook of hamate compressing ulnar artery C. Scaphoid compressing radial artery D. Lunate compressing median nerve E. Trapezoid bone compressing superficial radial nerve

177 D. The lunate is compressing the median nerve. The pisiform compressing the ulnar nerve is incorrect as the ulnar nerve innervates the skin on the medial one and a half digits. Hook of hamate compressing the ulnar artery is also incorrect. The hook of the hamate forms part of Guyon's canal; compression of the ulnar artery will not produce the deficits described because of the collateral circulation and anastomoses that exist with the radial artery. The scaphoid compressing the radial artery is also incorrect because there is collateral circulation from the palmar arches to compensate for radial artery occlusion. The trapezoid is not compressing the superficial radial nerve because the superficial branch of the radial nerve supplies the radial side and ball of the thumb and radial side of the index finger via its lateral and medial branches. GAS 792, 817; N 452; McM 168

178 A 54-year-old woman is admitted to the emergency department after a serious motor vehicle accident. Physical examination shows soft tissue edema and bruising around the neck. A radiograph of the humeroscapular region reveals a fracture of the midhumerus. Which of the following areas will most likely have impaired or absent sensation? A. Lateral aspect of the forearm B. Medial aspect of the arm C. Medial aspect of the arm and forearm D. Posterior aspect of the forearm E. Lateral and posterior aspect of the forearm

178 D. In the midshaft region of the humerus the radial nerve runs in the radial groove; fracture of the humerus at this point will likely impinge directly on the radial nerve, producing a sensory deficit along the posterior aspect of the forearm. The lateral aspect of the forearm is innervated by the lateral antebrachial cutaneous nerve of the forearm, which comes from the musculocutaneous nerve. These nerves may not be affected by a midshaft fracture of the humerus because they are well separated from the bone by muscle. The medial aspect of the arm and forearm is supplied by the intercostobrachial nerve and the medial antebrachial cutaneous nerve that takes its origin from the medial cord of the brachial plexus where it runs superficially, making it extremely difficult to injure both nerves during a midshaft fracture of the humerus. The lateral and posterior aspect of the forearm is an unlikely choice because the displaced bone not only has to impinge on the radial nerve but must also affect the very superficially located lateral antebrachial cutaneous nerve as well. GAS 763; N 465; McM 144

179 A 52-year-old man is admitted to the emergency department after a fall. Imaging studies show a fracture at the neck of the radius and a hematoma at the fracture site. Examination reveals weakness of wrist extension, abduction and extension of the thumb, and extension of the metacarpophalangeal and interphalangeal joints of the fingers. However, there was no sensory deficit. Which nerve is most likely affected? A. Anterior interosseous B. Posterior interosseous C. Radial D. Ulnar E. Superficial radial

179 B. The posterior interosseus nerve innervates the extensors of the wrist, abductor pollicis longus, extensor indicis, digiti minimi, and extensor pollicis longus muscles. The posterior interosseus nerve does not have any cutaneous branches, making it the best answer. The anterior interosseous nerve innervates flexors of the forearm. Although the radial nerve does give rise to the posterior interosseous nerve, there are no sensory deficits mentioned, so the radial nerve proper was not affected. The ulnar nerve also innervates flexors in the hand but since no sensory deficits were noted ulnar nerve injury can be ruled out. The superficial radial nerve is a purely cutaneous nerve. GAS 784, 787; N 466; McM 152

18 After a forceps delivery of an infant boy, the baby presents with his left upper limb adducted, internally rotated, and flexed at the wrist. The startle reflex is not seen on the ipsilateral side. Which part of the brachial plexus was most likely injured during this difficult delivery? A. Lateral cord B. Medial cord C. Ventral rami of the lower trunk D. Ventral ramus of the middle trunk E. Ventral rami of the upper trunk

18 E. The injury being described is also known as Erb-Duchenne paralysis or "waiter's tip hand" and is relatively common in children after a difficult delivery. This usually results from an injury to the upper trunk of the brachial plexus, presenting with loss of abduction, flexion, and lateral rotation of the arm. The superior trunk of the brachial plexus consists of spinal nerve ventral rami C5-6. GAS 738, 747; N 416; McM 139

18. A 31-year-old roofer walks in with tenosynovitis resulting from a deep penetrated wound in the palm by a big nail. Examination indicates that he has an infection in the ulnar bursa. This infection most likely resulted in necrosis of which of the following tendons? (A) Tendon of the fl exor carpi ulnaris (B) Tendon of the fl exor pollicis longus (C) Tendon of the fl exor digitorum profundus (D) Tendon of the fl exor carpi radialis (E) Tendon of the palmaris longus

18. The answer is C. The ulnar bursa, or common synovial fl exor sheath, contains the tendons of both the fl exor digitorum superfi cialis and profundus muscles. The radial bursa envelops the tendon of the fl exor pollicis longus. The tendons of the fl exor carpi ulnaris and the palmaris longus are not contained in the ulnar bursa.

180 After falling down concrete steps, a 42-year-old woman complains of tingling and numbness along the medial border of her left hand. Neurological examination reveals several abnormalities including Froment's sign. Weakness of which of the following muscles explains the presence of Froment's sign? A. First dorsal interosseous muscle B. Opponens pollicis C. Adductor pollicis D. Flexor pollicis longus E. Flexor pollicis brevis

180 C. Froment's test is a special test of the wrist to aid in the diagnosis of ulnar nerve palsy. The test evaluates the function of adductor pollicis muscle. The first dorsal interosseous is tested by adducting the index middle and ring fingers against resistance. The opponens pollicis muscle is evaluated using pulp-topulp opposition and the squeeze test. The flexor pollicis longus muscle is a flexor of the thumb and is tested by instructing the patient to flex the tip of the thumb against resistance while the proximal phalanx is held in extension. The flexor pollicis brevis muscle flexes the thumb at the metacarpophalangeal joint and is tested by asking the individual to flex the proximal phalanx of the thumb against resistance. GAS 814-816; N 452; McM 159

181 A 43-year-old woman visits the outpatient clinic with a painful hand. During physical examination, percussion over the flexor retinaculum causes a sharp pain in the lateral three and a half digits. This sign is indicative of which of the following conditions? A. Carpal tunnel syndrome B. De Quervain's tenosynovitis C. Thoracic outlet syndrome D. Mallet finger E. Radial nerve damage

181 A. Tinel's sign is used to aid in the diagnosis of carpal tunnel syndrome. It is performed by lightly percussing above the carpal tunnel where the median nerve is located. De Quervain's tenosynovitis describes tenosynovitis of the sheath or tunnel that surrounds tendons that control the thumb. It is tested using Finkelstein's test, where the examiner grips the thumb of the individual being tested and ulnar deviates the hand sharply. Thoracic outlet syndrome is tested using Adson's test. Mallet finger describes a finger deformity due to extensor digitorum tendons. Radial nerve damage is tested by evaluating the cutaneous distribution of the radial nerve or by testing the muscles innervated by the radial nerve. GAS 798; N 449; McM 157

182 A 25-year-old woman experiences numbness and tingling in her right arm and hand while carrying a piece of luggage. Physical examination showed no motor or sensory deficits in the upper limb. When asked to abduct her upper limb to 90 degrees and to maintain this position while repeatedly closing and opening her hands, the symptoms are reproduced along the medial border of the limb, from the axilla to the hand. Which nerve structure(s) is/are most likely compressed? A. Ulnar nerve at the medial epicondyle B. Radial nerve at the neck of the radius C. Median nerve in the carpal tunnel D. Inferior trunk of the brachial plexus E. Divisions of the brachial plexus

182 D. Compression on the inferior trunk of the brachial plexus compresses nerves C8 and T1. These nerves contribute to the medial cutaneous nerve of the arm (C8, T1) and the medial cutaneous nerve of the forearm (C8, T1). They also contribute to the median, medial pectoral, ulnar, and radial nerves. This patient has thoracic outlet syndrome, which causes compression of the inferior trunk of the brachial plexus usually by the presence of a cervical rib. Compression of the ulnar nerve at the medial epicondyle, radial nerve at the neck of the radius, or median nerve in the carpal tunnel would cause motor deficits not present in this patient. GAS 738, 747; N 416; McM 129

183 A 55-year-old woman is admitted to the emergency department after a car crash. Radiographic examination of her hand reveals a fractured carpal bone, which lies in the floor of the anatomical snuffbox. Which bone is fractured? A. Triquetrum B. Scaphoid C. Capitate D. Hamate E. Trapezoid

183 B. The scaphoid is the most commonly fractured carpal bone as a result of the relationship with the styloid process of the radius in the distal forearm. When a person falls as described in this question, the scaphoid gets pushed against the styloid process, usually at the narrowest ("waist") part of the scaphoid and fractures as a result of the forces transmitted through the bones. GAS 797; N 454; McM 122

184 An 18-year-old man presents to the emergency department with a painful right shoulder after a fall while diving for a soccer ball. A radiograph of the shoulder is shown in Figure 6-12. Examination revealed pain on passive adduction of the right arm across the chest. Which ligamentous structures must have been stretched/torn resulting in this injury? A. Acromioclavicular joint capsule and coracoclavicular ligament B. Acromioclavicular joint capsule and coracoacromial ligament C. Sternoclavicular joint capsule and coracoacromial ligament D. Coracoclavicular ligament and transverse scapular ligament E. Coracoclavicular ligament and coracoacromial ligament

184 A. The acromioclavicular and coracoclavicular ligaments are critical to the stability of the shoulder. In particular, the coracoclavicular ligament provides much of the weight-bearing support for the upper limb on the clavicle. The acromioclavicular joint ligament attaches the acromion (of the scapula) to the clavicle and the coracoclavicular ligament attaches the coracoid process to the clavicle. Interruption of these ligaments would cause dislocation of the acromioclavicular joint as seen in the radiograph. The sternoclavicular joint exists between the manubrium and the proximal end of the clavicle and is unrelated to either the injury or the radiograph. The coracoacromial ligament extends between the acromion and the coracoid process of the scapula. The transverse scapular ligament lies above the suprascapular notch and converts it into a foramen through which the suprascapular nerve runs (GAS Fig. 7-24). GAS 706; N 408; McM 136

185 A 67-year-old woman with osteoporosis injured her left shoulder/arm in a fall. Examination reveals bruising and dimpling of the upper part of the arm with exquisite tenderness over the affected area. The shoulder radiograph is shown in Figure 6-13. Which nerve is most likely to be injured? A. Radial B. Axillary C. Ulnar D. Median E. Musculocutaneous

185 B. The radiograph shows a fracture of the humerus at the surgical neck. The bruising and dimpling of the upper arm would result from this injury. The axillary nerve leaves the brachial plexus as a terminal branch of the posterior cord. It passes through the quadrangular space and wraps around the head of the humerus on its way to provide innervation to the teres minor, the deltoid, and the portion of skin over the lower aspect of the deltoid that is known as the "sergeant's patch." The radial nerve travels in the radial groove along the shaft of the humerus and would be injured in a fracture of the shaft of the humerus. The ulnar nerve would be injured in a fracture of the medial epicondyle. The median nerve travels too deep to be injured here and could be compressed at the carpal tunnel or at the cubital fossa. The musculocutaneous nerve is likewise within the tissue and will not be affected by this injury. GAS 704-705; N 418; McM 136

186 A 14-year-old boy falls on his outstretched hand. Examination reveals point tenderness above the humeral epicondyles and a pulsatile mass just above the cubital fossa. Neurological examination of the upper limb reveals weakness of pronation, wrist flexion, and grip strength. Only flexion at the distal interphalangeal joints of the ring and little fingers is intact. Thumb flexion and opposition are also impaired. A radiograph reveals a supracondylar fracture of the humerus. Which structures injured by the fracture best account for the findings? A. Axillary nerve and posterior circumflex humeral artery B. Radial nerve and deep brachial artery C. Median nerve and brachial artery D. Superficial radial nerve and radial artery E. Ulnar nerve and ulnar artery

186 C. The median nerve and brachial artery were injured. Injury to the median nerve is indicated by the weakness of pronation, wrist flexion, and grip strength. The median nerve innervates the muscles that govern or carry out these movements. The brachial artery is near the median nerve and can also be injured. Flexion at the distal interphalangeal joints of the ring and little fingers indicates that the ulnar nerve is intact. The radiograph indicates a supracondylar fracture of the humerus, which is the region in which the median nerve passes. A fracture at the surgical neck would injure the axillary nerve and posterior circumflex humeral artery. A fracture of the shaft of the humerus would injure the radial nerve and deep brachial artery. The superficial radial nerve and artery would be damaged by injury over or in the anatomical snuffbox. GAS 756, 761, 768; N 434; McM 149

19 A 35-year-old man has a small but painful tumor under the nail of his little finger. Which of the following nerves would have to be anesthetized for a painless removal of the tumor? A. Superficial radial B. Common palmar digital of median C. Common palmar digital of ulnar D. Deep radial E. Recurrent branch of median

19 C. The common palmar digital branch comes off the superficial branch of the ulnar nerve and supplies the skin of the little finger and the medial side of the ring finger. The superficial branch of the radial nerve provides cutaneous innervation to the radial (lateral) dorsum of the hand and the radial two and a half digits over the proximal phalanx. The common palmar digital branch of the median nerve innervates most of the lateral aspect of the palmar hand and the dorsal aspect of the second and third finger as well as the lateral part of the fourth digit. The deep radial nerve supplies the extensor carpi radialis brevis and supinator muscles and continues as the posterior interosseous nerve. The recurrent branch of the median nerve supplies the abductor pollicis brevis, flexor pollicis brevis, and opponens pollicis muscles. GAS 695-699, 814-815; N 460; McM 160

19. An 18-year-old boy involved in an automobile accident presents with arm that cannot abduct. His paralysis is caused by damage to which of the following nerves? (A) Suprascapular and axillary (B) Thoracodorsal and upper subscapular (C) Axillary and musculocutaneous (D) Radial and lower subscapular (E) Suprascapular and dorsal scapular

19. The answer is A. The abductors of the arm are the deltoid and supraspinatus muscles, which are innervated by the axillary and suprascapular nerves, respectively. The thoracodorsal nerve supplies the latissimus dorsi, which can adduct, extend, and rotate the arm medially. The upper and lower subscapular nerves supply the subscapularis, and the lower subscapular nerve also supplies the teres major; both of these structures can adduct and rotate the arm medially. The musculocutaneous nerve supplies the fl exors of the arm, and the radial nerve supplies the extensors of the arm. The dorsal scapular nerve supplies the levator scapulae and rhomboid muscles; these muscles elevate and adduct the scapula, respectively.

2 A 27-year-old man was admitted to the emergency department after an automobile collision in which he suffered a fracture of the lateral border of the scapula. Six weeks after the accident, physical examination reveals weakness in medial rotation and adduction of the humerus. Which nerve was most likely injured? A. Lower subscapular B. Axillary C. Radial D. Spinal accessory E. Ulnar

2 A. The lower subscapular nerve arises from the cervical spinal nerves 5 and 6. It innervates the subscapularis and teres major muscles. The subscapularis and teres major muscles are both responsible for adducting and medially rotating the arm. A lesion of this nerve would result in weakness in these motions. The axillary nerve also arises from cervical spinal nerves 5 and 6 and innervates the deltoid and teres minor muscles. The deltoid muscle is large and covers the entire surface of the shoulder, and contributes to arm movement in any plane. The teres minor muscle is a lateral rotator and a member of the rotator cuff group of muscles. The radial nerve arises from the posterior cord of the brachial plexus. It is the largest branch, and it innervates the triceps brachii and anconeus muscles in the arm. The spinal accessory nerve is cranial nerve XI, and innervates the trapezius muscle, which elevates and depresses the scapula. The ulnar nerve arises from the medial cord of the brachial plexus and runs down the medial aspect of the arm. It innervates muscles of the forearm and hand. GAS 714-717; N 413; McM 142

2. A patient comes in with a gunshot wound and requires surgery in which his thoracoacromial trunk needs to be ligated. Which of the following arterial branches would maintain normal blood flow? (A) Acromial (B) Pectoral (C) Clavicular (D) Deltoid (E) Superior thoracic

2. The answer is E. The superior thoracic artery is a direct branch of the axillary artery. The thoracoacromial trunk has four branches: the pectoral, clavicular, acromial, and deltoid.

20 A 25-year-old male athlete is admitted to the emergency department after a bad landing while performing the pole vault. Radiographic examination of his hand reveals a fractured carpal bone in the floor of the anatomic snuffbox (Fig. 6-2). Which bone has most likely been fractured? A. Triquetral B. Scaphoid C. Capitate D. Hamate E. Trapezoid

20 B. The anatomic snuffbox is formed by the tendons of the extensor pollicis brevis, the abductor pollicis longus, and the extensor pollicis longus. The floor is formed by the scaphoid bone, and it is here that one can palpate for a possible fractured scaphoid. GAS 800; N 430; McM 164

20. A 17-year-old boy with a stab wound received multiple injuries on the upper part of the arm and required surgery. If the brachial artery were ligated at its origin, which of the following arteries would supply blood to the profunda brachii artery? (A) Lateral thoracic (B) Subscapular (C) Posterior humeral circumfl ex (D) Superior ulnar collateral (E) Radial recurrent

20. The answer is C. The posterior humeral circumfl ex artery anastomoses with an ascending branch of the profunda brachii artery, whereas the lateral thoracic and subscapular arteries do not. The superior ulnar collateral and radial recurrent arteries arise inferior to the origin of the profunda brachii artery.

21 A 36-year-old man is brought to the emergency department because of a deep knife wound on the medial side of his distal forearm. He is unable to hold a piece of paper between his fingers and has sensory loss on the medial side of his hand and little finger. Which nerve is most likely injured? A. Axillary B. Median C. Musculocutaneous D. Radial E. Ulnar

21 E. The ulnar nerve innervates the palmar interossei, which adduct the fingers. This is the movement that would maintain the paper between the fingers. The axillary nerve does not innervate muscles of the hand. The median nerve supplies the first and second lumbricals, the opponens pollicis, abductor pollicis brevis, and the flexor pollicis brevis muscles. None of these muscles would affect the ability to hold a piece of paper between the fingers. The musculocutaneous and radial nerves do not supply muscles of the hand. GAS 808, 814; N 452; McM 162

21. A 23-year-old woman who receives a deep cut to her ring fi nger by a kitchen knife is unable to move the metacarpophalangeal joint. Which of the following pairs of nerves was damaged? (A) Median and ulnar (B) Radial and median (C) Musculocutaneous and ulnar (D) Ulnar and radial (E) Radial and axillary

21. The answer is D. The metacarpophalangeal joint of the ring figer is fl exed by the lumbrical, palmar, and dorsal interosseous muscles, which are innervated by the ulnar nerve. The extensor digitorum, which is innervated by the radial nerve, extends this joint. The musculocutaneous and axillary nerves do not supply muscles of the hand. The median nerve supplies the lateral two lumbricals, which can fl ex metacarpophalangeal joints of the index and middle fi ngers.

22 A 19-year-old man is brought to the emergency department after dislocating his shoulder while playing soccer. Following reduction of the dislocation, he has pain over the dorsal region of the shoulder and cannot abduct the arm normally. An MRI of the shoulder shows a torn muscle. Which of the following muscles is most likely to have been damaged by this injury? A. Coracobrachialis B. Long head of the triceps brachii C. Pectoralis minor D. Supraspinatus E. Teres major

22 D. The supraspinatus muscle is one of the four rotator cuff muscles—the other three being the infraspinatus, teres minor, and subscapularis muscles. The tendon of the supraspinatus muscle is relatively avascular and is often injured when the shoulder is dislocated. This muscle initiates abduction of the arm, and damage would impair this movement. The coracobrachialis muscle, which runs from the coracoid process to the humerus, functions in adduction and flexion of the arm. The main function of the triceps brachii muscle is to extend the elbow, and damage to its long head would not affect abduction. The pectoralis minor muscle functions as an accessory respiratory muscle and to stabilize the scapula and is not involved in abduction. The teres major muscle functions to adduct and medially rotate the arm. GAS 717; N 411; McM 137

22. A 27-year-old baseball player is hit on his forearm by a high-speed ball during the World Series, and the muscles that form the fl oor of the cubital fossa appear to be torn. Which of the following groups of muscles have lost their functions? (A) Brachioradialis and supinator (B) Brachialis and supinator (C) Pronator teres and supinator (D) Supinator and pronator quadratus (E) Brachialis and pronator teres

22. The answer is B. The brachialis and supinator muscles form the fl oor of the cubital fossa. The brachioradialis and pronator teres muscles form the lateral and medial boundaries, respectively. The pronator quadratus is attached to the distal ends of the radius and the ulna.

23 A 47-year-old female tennis professional is informed by her physician that she has a rotator cuff injury that will require surgery. Her physician explains that over the years of play, a shoulder ligament has gradually caused severe damage to the underlying muscle. To which of the following ligaments is the physician most likely referring? A. Acromioclavicular ligament B. Coracohumeral ligament C. Transverse scapular ligament D. Glenohumeral ligament E. Coracoacromial ligament

23 E. The coracoacromial ligament contributes to the coracoacromial arch, preventing superior displacement of the head of the humerus. Because this ligament is very strong, it will rarely be damaged; instead, the ligament can cause inflammation or erosion of the tendon of the supraspinatus muscle as the tendon passes back and forth under the ligament. The acromioclavicular ligament, connecting the acromion with the lateral end of the clavicle, is not in contact with the supraspinatus tendon. The coracohumeral ligament is located too far anteriorly to impinge upon the supraspinatus tendon. The glenohumeral ligament is located deep to the rotator cuff muscles and would not contribute to injury of the supraspinatus muscle. The transverse scapular ligament crosses the scapular notch and is not in contact with the supraspinatus tendon. GAS 705-706; N 411; McM 136

23. A 23-year-old man complains of numbness on the medial side of the arm following a stab wound in the axilla. On examination, he is diagnosed with an injury of his medial brachial cutaneous nerve. In which of the following structures are the cell bodies of the damaged nerve involved in numbness located? (A) Sympathetic chain ganglion (B) Dorsal root ganglion (C) Anterior horn of the spinal cord (D) Lateral horn of the spinal cord (E) Posterior horn of the spinal cord

23. The answer is B. The medial brachial cutaneous nerve contains sensory (general somatic afferent [GSA]) fi bers that have cell bodies in the dorsal root ganglia, and an injury of these GSA fi bers causes numbness of the medial side of the arm. It also contains sympathetic postganglionic fi bers that have cell bodies in the sympathetic chain ganglia. The anterior horn of the spinal cord contains cell bodies of skeletal motor (general somatic efferent [GSE]) fi bers, and the lateral horn contains cell bodies of sympathetic preganglionic fi bers. The posterior horn contains cell bodies of interneurons.

24 A 79-year-old man has numbness in the middle three digits of his right hand and finds it difficult to grasp objects with that hand. He states that he retired 9 years earlier, after working as a carpenter for 50 years. He has atrophy of the thenar eminence (Fig. 6-3). Which of the following conditions is the most likely cause of the problems in his hand? A. Compression of the median nerve in the carpal tunnel B. Formation of the osteophytes that compress the ulnar nerve at the medial epicondyle C. Hypertrophy of the triceps brachii muscle compressing the brachial plexus D. Osteoarthritis of the cervical spine E. Repeated trauma to the ulnar nerve

24 A. The median nerve supplies sensory innervation to the thumb, index, and middle fingers and also to the lateral half of the ring finger. The median nerve also provides motor innervation to muscles of the thenar eminence. Compression of the median nerve in the carpal tunnel explains these deficits in conjunction with normal functioning of the flexor compartment of the forearm because these muscles are innervated by the median nerve proximal to the carpal tunnel. Also, sensory innervation in the proximal palm will be normal because the palmar branch of the radial nerve usually branches off proximal to the flexor retinaculum. The ulnar nerve is not implicated in these symptoms. It does not provide sensation to digits 1 to 3. Compression of the brachial plexus could not be attributed to pressure from the triceps brachii because this muscle is located distal to the plexus. In addition, brachial plexus symptoms would include other upper limb deficits, rather than the focal symptoms described in this case. Osteoarthritis of the cervical spine would also lead to increasing complexity of symptoms. GAS 798; N 449; McM 160

24. A 38-year-old homebuilder was involved in an accident and is unable to supinate his forearm. Which of the following nerves are most likely damaged? (A) Suprascapular and axillary (B) Musculocutaneous and median (C) Axillary and radial (D) Radial and musculocutaneous (E) Median and ulnar

24. The answer is D. The supinator and biceps brachii muscles, which are innervated by the radial and musculocutaneous nerves, respectively, produce supination of the forearm. This is a question of two muscles that can supinate the forearm.

25 A 13-year-old boy is brought to the emergency department after losing control during a motorbike race in which he was hit by several of the other racers. Physical examination reveals several cuts and bruises. He is unable to extend the left wrist, fingers, and thumb, although he can extend the elbow. Sensation is lost in the lateral half of the dorsum of the left hand. Which of the following nerves has most likely been injured to result in these signs, and in what part of the arm is the injury located? A. Median nerve, anterior wrist B. Median nerve, arm C. Radial nerve, midhumerus D. Ulnar nerve, midlateral forearm E. Ulnar nerve, midpalmar region

25 C. The radial nerve innervates the extensor compartments of the arm and the forearm. It supplies the triceps brachii proximal to the spiral groove, so elbow extension is intact here. It also provides sensory innervation to much of the posterior arm and forearm as well as the dorsal thumb, index, and middle fingers up to the level of the fingernails. Symptoms are described only in the distal limb due to the midhumeral location of the lesion. The median nerve innervates flexors of the forearm and thenar muscles and provides sensory innervation to the lateral palmar hand. The ulnar nerve supplies only the flexor carpi ulnaris and the medial half of the flexor digitorum profundus in the forearm. Additionally, its sensory distribution is to both the palmar and dorsal aspects of the medial hand. It does not supply extensor muscles. GAS 746, 761; N 418; McM 144

25. A 31-year-old patient complains of sensory loss over the anterior and posterior surfaces of the medial third of the hand and the medial one and one-half fi ngers. He is diagnosed by a physician as having "funny bone" symptoms. Which of the following nerves is injured? (A) Axillary (B) Radial (C) Median (D) Ulnar (E) Musculocutaneous

25. The answer is D. The ulnar nerve supplies sensory fi bers to the skin over the palmar and dorsal surfaces of the medial third of the hand and the medial one and one-half fi ngers. The median nerve innervates the skin of the lateral side of the palm; the palmar side of the lateral three and one-half fi ngers; and the dorsal side of the index fi nger, the middle fi nger, and one-half of the ring fi nger. The radial nerve innervates the skin of the radial side of the hand and the radial two and one-half digits over the proximal phalanx.

26 A 17-year-old boy has weakness of elbow flexion and supination of the left hand after sustaining a knife wound in that arm in a street fight. Examination in the emergency department indicates that a nerve has been severed. Which of the following conditions will also most likely be seen during physical examination? A. Inability to adduct and abduct his fingers B. Inability to flex his fingers C. Inability to flex his thumb D. Sensory loss over the lateral surface of his forearm E. Sensory loss over the medial surface of his forearm

26 D. The musculocutaneous nerve innervates the brachialis and biceps brachii muscles, which are the main flexors at the elbow. The biceps brachii inserts on the radius and is an important supinator. Because the musculocutaneous nerve is damaged in this case, it leads to loss of sensory perception to the lateral forearm, which is supplied by the distal continuation of the musculocutaneous nerve (known as the lateral antebrachial cutaneous nerve). The name "musculocutaneuous" indicates it is "muscular" in the arm and "cutaneous" in the forearm. Adduction and abduction of the fingers are mediated by the ulnar nerve and would not be affected in this instance. The flexor pollicis brevis muscle flexes the thumb and is mainly innervated by the recurrent branch of the median nerve. Flexion of the fingers is performed by the long flexors of the fingers and lumbrical muscles, innervated by the median and ulnar nerves. Sensory innervation of the medial forearm is provided by the medial antebrachial cutaneous nerve, usually a direct branch of the medial cord of the brachial plexus. GAS 746, 760; N 417; McM 143

26. A patient with a deep stab wound in the middle of the forearm has impaired movement of the thumb. Examination indicates a lesion of the anterior interosseous nerve. Which of the following muscles is paralyzed? (A) Flexor pollicis longus and brevis (B) Flexor pollicis longus and opponens pollicis (C) Flexor digitorum profundus and pronator quadratus (D) Flexor digitorum profundus and superfi -cialis (E) Flexor pollicis brevis and pronator quadratus

26. The answer is C. The anterior interosseous nerve is a branch of the median nerve and supplies the fl exor pollicis longus, half of the fl exor digitorum profundus, and the pronator quadratus. The median nerve supplies the pronator teres, fl exor digitorum superfi cialis, palmaris longus, and fl exor carpi radialis muscles. A muscular branch (the recurrent branch) of the median nerve innervates the thenar muscles.

27 Following several days of 12-hour daily rehearsals of the symphony orchestra for a performance of a Wagnerian opera, the 52-year-old male conductor experienced such excruciating pain in the posterior aspect of his right forearm that he could no longer direct the musicians. When the maestro's forearm was palpated 2 cm distal and posteromedial to the lateral epicondyle, the resulting excruciating pain caused him to grimace. Injections of steroids and rest were recommended to ease the pain. Which of the following injuries is most likely? A. Compression of the ulnar nerve by the flexor carpi ulnaris B. Compression of the median nerve by the pronator teres C. Compression of the median nerve by the flexor digitorum superficialis D. Compression of the superficial radial nerve by the brachioradialis E. Compression of the deep radial nerve by the supinator

27 E. The deep branch of the radial nerve courses between the two heads of the supinator muscle and is located just medial and distal to the lateral epicondyle. After the nerve emerges from the supinator it is called the posterior interosseous nerve. It can be irritated by hypertrophy of the supinator, which compresses the nerve, causing pain and weakness. The ulnar nerve courses laterally behind the medial epicondyle and continues anterior to the flexor carpi ulnaris muscle. The median nerve passes into the forearm flexor compartment; the superficial radial nerve courses down the lateral aspect of the posterior forearm and would not cause pain due to pressure applied to the posterior forearm. GAS 792; N 466; McM 152

27. A 29-year-old patient comes in; he cannot fl ex the distal interphalangeal (DIP) joint of the index fi nger. His physician determines that he has nerve damage from a supracondylar fracture. Which of the following conditions is also a symptom of this nerve damage? (A) Inability to fl ex the DIP joint of the ring fi nger (B) Atrophy of the hypothenar eminence (C) Loss of sensation over the distal part of the second digit (D) Paralysis of all the thumb muscles (E) Loss of supination

27. The answer is C. The fl exor digitorum profundus muscle fl exes the DIP joints of the index and middle fi ngers and is innervated by the median nerve, which also supplies sensation over the distal part of the second digit. The same muscle fl exes the DIP joints of the ring and little fi ngers but receives innervation from the ulnar nerve, which also innervates the hypothenar muscles. The median nerve innervates the thenar muscles. The radial nerve innervates the supinator, abductor pollicis longus, and extensor pollicis longus and brevis muscles. The ulnar nerve innervates the adductor pollicis. The musculocutaneous nerve supplies the biceps brachii that can supinate the arm.

28 A 54-year-old female marathon runner presents with pain in her right wrist that resulted when she fell with force on her outstretched hand. Radiographic studies indicate an anterior dislocation of a carpal bone (Fig. 6-4). Which of the following bones is most likely dislocated? A. Capitate B. Lunate C. Scaphoid D. Trapezoid E. Triquetrum

28 B. The lunate is the most commonly dislocated carpal bone because of its shape and relatively weak ligaments anteriorly. Dislocations of the scaphoid and triquetrum are relatively rare. The trapezoid and capitate bones are located in the distal row of the carpal bones. GAS 793; N 439; McM 167

28. A 27-year-old man with cubital tunnel syndrome complains of numbness and tingling in the ring and little fi nger and back and sides of his hand because of damage to a nerve in the tunnel at the elbow. Which of the following muscles is most likely to be paralyzed? (A) Flexor digitorum superfi cialis (B) Opponens pollicis (C) Two medial lumbricals (D) Pronator teres (E) Supinator

28. The answer is C. The ulnar nerve innervates the two medial lumbricals. However, the median nerve innervates the two lateral lumbricals, the fl exor digitorum superfi cialis, the opponens pollicis, and the pronator teres muscles.

29 A 45-year-old man is admitted to the hospital after a car crash. Radiographic examination reveals mild disc herniations of C7, C8, and T1. The patient presents with a sensory deficit of the C8 and T1 spinal nerve dermatomes. The dorsal root ganglia of C8 and T1 would contain cell bodies of sensory fibers carried by which of the following nerves? A. Medial antebrachial cutaneous nerve B. Long thoracic nerve C. Lateral antebrachial cutaneous nerve D. Deep branch of ulnar nerve E. Anterior interosseous nerve

29 A. The medial antebrachial cutaneous nerve carries sensory fibers derived from the C8 and T1 levels. The lateral antebrachial cutaneous nerve is the distal continuation of the musculocutaneous nerve, carrying fibers from the C5, C6, and C7 levels. The deep branch of the ulnar nerve and the anterior interosseous nerves carry predominantly motor fibers. The sensory fibers coursing in the radial nerve are derived from the C5 to C8 levels. GAS 695-699, 742; N 460; McM 148

29. A secretary comes in to your offi ce complaining of pain in her wrists from typing all day. You determine that she likely has carpal tunnel syndrome. Which of the following conditions would help you determine the diagnosis? (A) Inability to adduct the little fi nger (B) Inability to fl ex the DIP joint of the ring fi nger (C) Flattened thenar eminence (D) Loss of skin sensation of the medial one and one-half fi ngers (E) Atrophied adductor pollicis muscle

29. The answer is C. The carpal tunnel contains the median nerve and the tendons of fl exor pollicis longus, fl exor digitorum profundus, and fl exor digitorum superfi cialis muscles. Carpal tunnel syndrome results from injury to the median nerve, which supplies the thenar muscle. Thus, injury to this nerve causes the fl attened thenar eminence. The middle fi nger has no attachment for the adductors. The ulnar nerve innervates the medial half of the fl exor digitorum profundus muscle, which allows fl exion of the DIP joints of the ring and little fi ngers. The ulnar nerve supplies the skin over the medial one and one-half fi ngers and adductor pollicis muscle.

3 A 48-year-old female court stenographer is admitted to the orthopedic clinic with symptoms of carpal tunnel syndrome, with which she has suffered with for almost a year. Which muscles most typically become weakened in this condition? A. Dorsal interossei B. Lumbricals III and IV C. Thenar D. Palmar interossei E. Hypothenar

3 C. The thenar muscles (and lumbricals I and II) are innervated by the median nerve, which runs through the carpal tunnel. The carpal tunnel is formed anteriorly by the flexor retinaculum and posteriorly by the carpal bones. Carpal tunnel syndrome is caused by a compression of the median nerve, due to reduced space in the carpal tunnel. The carpal tunnel contains the tendons of flexor pollicis longus, flexor digitorum profundus, and flexor digitorum superficialis muscles and their synovial sheaths. The dorsal interossei, lumbricals III and IV , palmar interossei, and hypothenar muscles are all innervated by the ulnar nerve. GAS 798, 808; N 452; McM 159

3. A 29-year-old man comes in with a stab wound, cannot raise his arm above horizontal, and exhibits a condition known as "winged scapula." Which of the following structures of the brachial plexus would most likely be damaged? (A) Medial cord (B) Posterior cord (C) Lower trunk (D) Roots (E) Upper trunk

3. The answer is D. Winged scapula is caused by paralysis of the serratus anterior muscle that results from damage to the long thoracic nerve, which arises from the roots of the brachial plexus (C5-C7).

30 A 23-year-old female maid was making a bed in a hotel bedroom. As she straightened the sheet by running her right hand over the surface with her fingers extended, she caught the end of the index finger in a fold. She experienced a sudden, severe pain over the base of the terminal phalanx. Several hours later when the pain had diminished, she noted that the end of her right index finger was swollen and she could not completely extend the terminal interphalangeal joint. Which one of the following structures within the digit was most likely injured? A. The proper palmar digital branch of the median nerve B. The vinculum longa C. The insertion of the tendon of the extensor digitorum onto the base of the distal phalanx D. The insertion of the flexor digitorum profundus tendon E. The insertion of the flexor digitorum superficialis tendon

30 C. The contraction of the extensor mechanism produces extension of the distal interphalangeal joint. When it is torn from the distal phalanx, the digit is pulled into flexion by the flexor digitorum profundus muscle. If a piece of the distal phalanx is attached to the torn tendon it is an avulsion fracture. The proper palmar digital branches of the median nerve supply lumbrical muscles and carry sensation from their respective digits. Vincula longa are slender, bandlike connections from the deep flexor tendons to the phalanx that can carry blood supply to the tendons. The insertions of the flexor digitorum superficialis and profundus are on the flexor surface of the middle and distal phalanges, respectively, and act to flex the interphalangeal joints. GAS 787, 808; N 451; McM 165

30. A man is unable to hold typing paper between his index and middle fi ngers. Which of the following nerves was likely injured? (A) Radial nerve (B) Median nerve (C) Ulnar nerve (D) Musculocutaneous nerve (E) Axillary nerve

30. The answer is C. To hold a typing paper, the index fi nger is adducted by the palmar interosseous muscle, and the middle fi nger is abducted by the dorsal interosseous muscle. Both muscles are innervated by the ulnar nerve.

31 A 45-year-old man had fallen on his outstretched hand, resulting in Smith's fracture of the distal end of the radius. The fractured bone displaced a carpal bone in the palmar direction, resulting in nerve compression within the carpal tunnel. Which of the following carpal bones will most likely be dislocated? A. Scaphoid B. Trapezium C. Capitate D. Hamate E. Lunate

31 E. In Smith's fracture, the distal fragment of the radius deviates palmarward, often displacing the lunate bone. The other listed bones are unlikely to be displaced in a palmar direction by Smith's fracture. GAS 771-774; N 439; McM 167

31. The victim of an automobile accident has a destructive injury of the proximal row of carpal bones. Which of the following bones is most likely damaged? (A) Capitate (B) Hamate (C) Trapezium (D) Triquetrum (E) Trapezoid

31. The answer is D. The proximal row of carpal bones consists of the scaphoid, lunate, triquetrum, and pisiform bones, whereas the distal row consists of the trapezium, trapezoid, capitate, and hamate bones.

32 A 15-year-old girl was brought to the emergency department with a tear of the tendons in the first dorsal compartment of the wrist from a severe bite by a pit bull dog. The injured tendons in this compartment would include which of the following muscles? A. Extensor carpi radialis longus and brevis B. Abductor pollicis longus and extensor pollicis brevis C. Extensor digitorum D. Extensor indicis proprius E. Extensor carpi ulnaris

32 B. The abductor pollicis longus and extensor pollicis brevis muscles are the occupants of the first dorsal compartment of the wrist. The extensor carpi radialis longus and brevis are in the second compartment. The extensor digitorum is in the third compartment, as is the extensor indicis. The extensor carpi ulnaris is located in the sixth dorsal compartment. GAS 785-790; N 431; McM 165

32. A patient has a torn rotator cuff of the shoulder joint as the result of an automobile accident. Which of the following muscle tendons is intact and has normal function? (A) Supraspinatus (B) Subscapularis (C) Teres major (D) Teres minor (E) Infraspinatus

32. The answer is C. The rotator cuff consists of the tendons of the supraspinatus, infraspinatus, subscapularis, and teres minor muscles. It stabilizes the shoulder joint by holding the head of the humerus in the glenoid cavity during movement. The teres major inserts on the medial lip of the intertubercular groove of the humerus.

33 As she fell from the uneven parallel bars, a 17-year-old female gymnast grasped the lower bar briefly with one hand but then fell painfully to the floor. An MRI examination reveals an injury to the medial cord of the brachial plexus. Which of the following spinal nerve levels would most likely be affected? A. C5, C6 B. C6, C7 C. C7, C8 D. C7, C8, T1 E. C8, T1

33 E. The medial cord has been injured by traction on the lower trunk of the brachial plexus. The medial cord is the continuation of the inferior (lower) trunk of the brachial plexus, which is formed by C8 and T1. C5 and C6 are typically associated with the superior (upper) trunk level and thus the lateral cord. C7 forms the middle trunk. An injury to the posterior cord would usually involve the C7 spinal nerve. This is a typical Klumpke paralysis. GAS 738-747; N 416; McM 139

33. A patient complains of having pain with repeated movements of his thumb (claudication). His physician performs the Allen test and finds an insufficiency of the radial artery. Which of the following conditions would be a result of the radial artery stenosis? (A) A marked decrease in the blood flow in the superficial palmar arterial arch (B) Decreased pulsation in the artery passing superficial to the flexor retinaculum (C) Ischemia of the entire extensor muscles of the forearm (D) A marked decrease in the blood flow in the princeps pollicis artery (E) A low blood pressure in the anterior interosseous artery

33. The answer is D. The radial artery divides into the princeps pollicis artery and the deep palmar arterial arch. Thus, stenosis of the radial artery results in a decreased blood fl ow in the princeps pollicis artery. The superfi cial palmar arterial arch is formed primarily by the ulnar artery, which passes superfi cial to the fl exor retinaculum. The extensor compartment of the forearm receives blood from the posterior interosseous artery, which arises from the common interosseous branch of the ulnar artery. However, the radial and radial recurrent arteries supply the brachioradialis and the extensor carpi radialis longus and brevis.

34 A 21-year-old female softball pitcher is examined in the emergency department after she was struck in the arm by a line drive. Plain radiographic and MRI studies show soft tissue injury to the region of the spiral groove, with trauma to the radial nerve. Which of the following muscles would be intact after this injury? A. Flexor carpi ulnaris B. Extensor indicis C. Brachioradialis D. Extensor carpi radialis longus E. Supinator

34 A. The flexor carpi ulnaris muscle is not innervated by the radial nerve but rather by the ulnar nerve. The brachioradialis, extensor carpi radialis longus and brevis, and supinator muscles are all innervated by the radial nerve distal to the spiral groove. GAS 777, 787; N 418; McM 149

34. A patient bleeding from the shoulder secondary to a knife wound is in fair condition because there is vascular anastomosis around the shoulder. Which of the following arteries is most likely a direct branch of the subclavian artery that is involved in the anastomosis? (A) Dorsal scapular artery (B) Thoracoacromial artery (C) Circumflex scapular artery (D) Transverse cervical artery (E) Suprascapular artery

34. The answer is A. The dorsal scapular artery arises directly from the third part of the subclavian artery and replaces the deep (descending) branch of the transverse cervical artery. The suprascapular and transverse cervical arteries are branches of the thyrocervical trunk of the subclavian artery. The thoracoacromial artery is a short trunk from the fi rst or second part of

35 Examination of a 21-year-old female athlete with an injury of the radial nerve in the spiral groove would typically demonstrate which of the following physical signs? A. Weakness of thumb abduction and thumb extension B. Weakness of thumb opposition C. Inability to extend the elbow D. Paralysis of pronation of the hand E. Paralysis of abduction and adduction of the arm

35 A. Injury to the radial nerve in the spiral groove will paralyze the abductor pollicis longus muscle and both extensors of the thumb. This injury will also lead to wrist drop (inability to extend the wrist). Weakness of grip would also occur, although this is not mentioned in the question. If the wrist is flexed, finger flexion and grip strength are weakened because the long flexor tendons are not under tension. Note how much your strength of grip is increased when your wrist is extended versus when it is flexed. GAS 763, 818; N 427; McM 144

35. During a breast examination of a 56-year-old woman, the physician found a palpable mass in her breast. Which of the following characteristics of breast cancer and its diagnosis is correct? (A) Elevated nipple (B) Polymastia (C) Shortening of the clavipectoral fascia (D) Dimpling of the overlying skin (E) Enlargement of the breast

35. The answer is D. Breast cancer may cause dimpling of the overlying skin because of shortening of the suspensory (Cooper's) ligaments and inverted or retracted nipple because of pulling on the lactiferous ducts. Polymastia is a condition in which more than two breasts are present.

36 A 58-year-old convenience store operator had received a superficial bullet wound to the soft tissues on the medial side of the elbow in an attempted robbery. A major nerve was repaired at the site where it passed behind the medial epicondyle. Bleeding was stopped from an artery that accompanied the nerve in its path toward the epicondyle. Vascular repair was performed on this small artery because of its important role in supplying blood to the nerve. Which of the following arteries was most likely repaired? A. The profunda brachii artery B. The radial collateral artery C. The superior ulnar collateral artery D. The inferior ulnar collateral artery E. The anterior ulnar recurrent artery

36 C. The superior ulnar collateral branch of the brachial artery accompanies the ulnar nerve in its path posterior to the medial epicondyle and is important in the blood supply of the nerve. The profunda brachii artery passes down the arm with the radial nerve. The radial collateral artery arises from the profunda brachii artery and anastomoses with the radial recurrent branch of the radial artery proximal to the elbow laterally. The inferior ulnar collateral artery arises from the brachial artery and accompanies the median nerve into the forearm. The anterior ulnar recurrent artery arises from the ulnar artery and anastomoses with the inferior ulnar collateral artery anterior to the elbow. GAS 756; N 435; McM 149

36. A patient with a stab wound receives a laceration of the musculocutaneous nerve. Which of the following conditions is most likely to have occurred? (A) Lack of sweating on the lateral side of the forearm (B) Inability to extend the forearm (C) Paralysis of brachioradialis muscle (D) Loss of tactile sensation on the arm (E) Constriction of blood vessels on the hand

36. The answer is A. The musculocutaneous nerve contains sympathetic postganglionic fi bers that supply sweat glands and blood vessels on the lateral side of the forearm as the lateral antebrachial cutaneous nerve. The musculocutaneous nerve does not supply the extensors of the forearm and the brachioradialis. This nerve also supplies tactile sensation on the lateral side of the forearm but not the

37 A 60-year-old male butcher accidentally slashed his wrist with his butcher's knife, partially dividing the ulnar nerve. Which of the following actions would most likely be lost as a result of this injury? A. Flexion of the proximal interphalangeal joint of the fifth digit (little finger) B. Extension of the thumb C. Adduction of the fifth digit D. Abduction of the thumb E. Opposition of the thumb

37 C. Adduction of the fifth digit is produced by contraction of the third palmar interosseous muscle. All of the interossei are innervated by the deep branch of the ulnar nerve. Flexion of the proximal interphalangeal joint is a function of the flexor digitorum superficialis, supplied by the median nerve. Opposition of the thumb is a function of the opponens pollicis, supplied by the recurrent branch of the median nerve. GAS 808; N 452; McM 162

37. A 20-year-old man fell from the parallel bar during the Olympic trial. A neurologic examination reveals that he has a lesion of the lateral cord of the brachial plexus. Which of the following muscles is most likely weakened by this injury? (A) Subscapularis (B) Teres major (C) Latissimus dorsi (D) Teres minor (E) Pectoralis major

37. The answer is E. The pectoralis major is innervated by the lateral and medial pectoral nerves originating from the lateral and medial cords of the brachial plexus, respectively. The subscapularis, teres major, latissimus dorsi, and teres minor muscles are innervated by nerves originating from the posterior cord of the brachial plexus.

38 A 23-year-old male medical student fell asleep in his chair with Netter's Atlas wedged into his axilla. When he awoke in the morning, he was unable to extend his wrist or fingers. Movements of the ipsilateral shoulder joint appear to be normal. Which of the following nerves was most likely compressed, producing the symptoms described? A. Lateral cord of the brachial plexus B. Medial cord of the brachial plexus C. Radial nerve D. Median nerve E. Lateral and medial pectoral nerves

38 C. The radial nerve is the most likely nerve compressed to cause these symptoms. This type of nerve palsy is often called "Saturday night palsy." One reason for this nickname is that people would supposedly fall asleep after being intoxicated on a Saturday night with their arm over the back of a chair or bench, thereby compressing the nerve in the spiral groove. The radial nerve innervates all of the extensors of the elbow, wrist, and fingers. It innervates the triceps brachii muscle but the motor branch typically comes off proximal to the site of compression, so the patient can still extend the elbow. Paralysis of the lateral cord of the brachial plexus would result in loss of the musculocutaneous nerve and the pectoral nerves, which do not mediate extension of the forearm or hand. The medial cord of the brachial plexus branches into the median nerve and ulnar nerve. Neither of these nerves innervates muscles that control extension. The median nerve innervates flexors of the forearm and the thenar muscles. The lateral and median pectoral nerves do not extend into the arm and innervate the pectoralis major and minor muscles. GAS 761-763; N 415; McM 139

38. A 24-year-old carpenter suffers a crush injury of his entire little fi nger. Which of the following muscles is most likely to be spared? (A) Flexor digitorum profundus (B) Extensor digitorum (C) Palmar interossei (D) Dorsal interossei (E) Lumbricals

38. The answer is D. The dorsal interossei are abductors of the fi ngers. The little fi nger has no attachment for the dorsal interosseous muscle because it has its own abductor. Therefore, the dorsal interosseous muscle is not affected. Other muscles are attached to the little fi nger, thus they are injured.

39 The kidneys of a 32-year-old woman were failing, and she needed to be placed on dialysis. However, the search in her upper limb for a suitable vein was unexpectedly difficult. The major vein on the lateral side of the arm was too small; others were too delicate. Finally, a vein was found on the medial side of the arm that passed through the superficial and deep fascia to join veins beside the brachial artery. Which of the following veins was this? A. Basilic B. Lateral cubital C. Cephalic D. Medial cubital E. Medial antebrachial

39 A. The basilic vein can be used for dialysis, especially when the cephalic vein is judged to be too small, as in this case. The basilic vein can be elevated from its position as it passes through the fascia on the medial side of the arm (brachium). The cephalic vein passes more laterally up the limb. The lateral cubital vein is a tributary to the cephalic vein, and the medial cubital vein joins the basilic vein, both of which are rather superficial in position. The medial antebrachial vein courses up the midline of the forearm (antebrachium) ventrally. GAS 700, 759, 769-770; N 401; McM 148

39. A 7-year-old boy falls from a tree house and is brought to the emergency department of a local hospital. On examination, he has weakness in rotating his arm laterally because of an injury of a nerve. Which of the following conditions is most likely to cause a loss of this nerve function? (A) Injury to the lateral cord of the brachial plexus (B) Fracture of the anatomic neck of the humerus (C) Knife wound on the teres major muscle (D) Inferior dislocation of the head of the humerus (E) A tumor in the triangular space in the shoulder region

39. The answer is D. Inferior dislocation of the head of the humerus may damage the axillary nerve, which arises from the posterior cord of the brachial plexus, runs through the quadrangular space accompanied by the posterior humeral circumfl ex vessels around the surgical neck of the humerus, and supplies the deltoid and teres minor, which are lateral rotators of the arm.

4 A 45-year-old man arrived at the emergency department with injuries to his left elbow after he fell in a bicycle race. Plain radiographic and magnetic resonance imaging (MRI) examinations show a fracture of the medial epicondyle and an injured ulnar nerve. Which of the following muscles will most likely be paralyzed? A. Flexor digitorum superficialis B. Biceps brachii C. Brachioradialis D. Flexor carpi ulnaris E. Supinator

4 D. Fracture of the medial epicondyle often causes damage to the ulnar nerve due to its position in the groove behind the epicondyle. The ulnar nerve innervates one and a half muscles in the forearm, the flexor carpi ulnaris and the medial half of the flexor digitorum profundus muscles. The nerve continues on to innervate most of the muscles in the hand. The flexor digitorum superficialis is innervated by the median nerve and the biceps brachii muscle by the musculocutaneous. The radial nerve innervates both the brachioradialis and supinator muscles. GAS 752, 764-768; N 432; McM 145

4. A 16-year-old patient has weakness flxing the metacarpophalangeal joint of the ring finger and is unable to adduct the same finger. Which of the following muscles is most likely paralyzed? (A) Flexor digitorum profundus (B) Extensor digitorum (C) Lumbrical (D) Dorsal interosseous (E) Palmar interosseous

4. The answer is E. dorsal and palmar interosseous and lumbrical muscles=flex the metacarpophalangeal joints and extend the interphalangeal joints. palmar interosseous muscles adduct the fingers, while the dorsal interosseous muscles abduct the fingers. flexor digitorum profundus flexes the distal interphalangeal (DIP) joints.

40 A 29-year-old woman had sustained a deep laceration in the proximal part of the forearm. After the wound is closed, the following functional deficits are observed by the neurologist on service: the first three digits are in a position of extension and cannot be flexed; digits 4 and 5 are partially flexed at the metacarpophalangeal (MCP) joints and noticeably more flexed at the distal interphalangeal joints; sensation is absent in the lateral side of the palm and the palmar surfaces of digits 1 to 3 and half of digit 4. Which of the following nerve(s) has (have) most likely been injured? A. Median nerve B. Ulnar and median nerves C. Ulnar nerve D. Radial and ulnar nerves E. Radial nerve

40 A. The patient exhibits the classic "benediction attitude" of the thumb and fingers from injury to the median nerve proximally in the forearm. The thumb is somewhat extended (radial supplied abductor and extensors unopposed); digits 2 and 3 are extended (by intact interossei); digits 4 and 5 are partially flexed (by their intact flexor digitorum profundus). A lesion of the median nerve would result in weakened flexion of the proximal interphalangeal joints of all digits (flexor digitorum superficialis muscle), loss of flexion of the interphalangeal joint of the thumb, the distal interphalangeal joints of digits 2 and 3 (flexor digitorum profundus muscle), and weakened flexion of the metacarpophalangeal joints of the second and third digits (first and second lumbricals). A lesion of both the ulnar and median nerves would cause weakness or paralysis of flexion of all of the digits. A lesion of the ulnar nerve would mostly cause weakness in flexion of the DIP of the fourth and fifth digits and would affect all of the interosseous muscles and the lumbricals of the third and fourth digits. A lesion of the radial nerve would cause weakness in extension of the wrist, thumb, and metacarpophalangeal joints. GAS 784, 817; N 434; McM 149

40. A 49-year-old woman is diagnosed as having a large lump in her right breast. Lymph from the cancerous breast drains primarily into which of the following nodes? (A) Apical nodes (B) Anterior (pectoral) nodes (C) Parasternal (internal thoracic) nodes (D) Supraclavicular nodes (E) Nodes of the anterior abdominal wall

40. The answer is B. Lymph from the breast drains mainly (75%) to the axillary nodes, more specifi cally to the anterior (pectoral) nodes.

41 A 35-year-old male wrestler is admitted to the emergency department with excruciating pain in his right shoulder and proximal arm. During physical examination, the patient clutches the arm at the elbow with his contralateral hand and is unable to move the injured limb. Radiographic studies show that the patient has a dislocation of the humerus at the glenohumeral joint. Which of the following conditions is the most likely? A. The head of the humerus is displaced anteriorly B. The head of the humerus is displaced posteriorly C. The head of the humerus is displaced inferiorly D. The head of the humerus is displaced superiorly E. The head of the humerus is displaced medially

41 C. The head of the humerus is displaced inferiorly because in that location it is not supported by rotator cuff muscle tendons or the coracoacromial arch. It is also pulled anteriorly (relative to the tendon of the triceps brachii) beneath the coracoid process by pectoralis and subscapularis muscles. It would not be displaced posteriorly because it is supported by the teres minor and infraspinatus muscle tendons. It would not be displaced superiorly because the coracoacromial ligament and supraspinatus reinforce in that direction. A medial dislocation is blocked by the subscapularis tendon. GAS 712; N 408; McM 136

42 The 35-year-old woman has a hard breast nodule about 1 cm in diameter slightly above and lateral to the right areola of her right breast. A specific dye is injected into the tissue around the tumor, and an incision is made to expose the lymphatic vessels draining the area, for the lymphatic vessels to take up the dye, which is visible to the eye. The vessels can then be traced to surgically expose the lymph nodes receiving the lymph from the tumor. Which of the following nodes will most likely first encounter the lymph from the tumor? A. Anterior axillary (pectoral) nodes B. Rotter's interpectoral nodes C. Parasternal nodes along the internal thoracic artery and vein D. Central axillary nodes E. Apical or infraclavicular nodes

42 A. The anterior axillary (or anterior pectoral) nodes are the first lymph nodes to receive most of the lymph from the breast parenchyma, areola, and nipple. From there, lymph flows through central axillary, apical, and supraclavicular nodes in sequence. The interpectoral Rotter's nodes lie between the pectoral muscles and are, unfortunately, an alternate route in some patients, speeding the rate of metastasis. The parasternal nodes receive lymph from the medial part of the breast and lie along the internal thoracic artery and vein. GAS 748; N 403; McM 179

42. A construction worker suffers a destructive injury of the structures related to the anatomic snuffbox. Which of the following structures would most likely be damaged? (A) Triquetral bone (B) Trapezoid bone (C) Extensor indicis tendon (D) Abductor pollicis brevis tendon (E) Radial artery

42. The answer is E. The radial artery lies on the fl oor of the anatomic snuffbox. Other structures are not related to the snuffbox. The tendons of the extensor pollicis longus, extensor pollicis brevis, and abductor pollicis longus muscles form the boundaries of the anatomic snuffbox. The scaphoid and trapezium bones form its fl oor.

43 During a fight in a tavern, a 45-year-old male construction worker received a shallow stab wound from a broken beer bottle at a point near the middle of the left posterior triangle of his neck. Upon physical examination, it is observed that the left shoulder is drooping lower than the right shoulder, and the superior angle of the scapula juts out slightly. Strength in turning the head to the right or left appears to be symmetric. Which of the following nerves is most likely injured? A. Suprascapular nerve in the supraspinous fossa B. The terminal segment of the dorsal scapular nerve C. The upper trunk of the brachial plexus D. The spinal accessory nerve in the posterior cervical triangle E. The thoracodorsal nerve in the axilla

43 D. The left spinal accessory nerve (CN XI) has been injured distal to the sternocleidomastoid muscle, resulting in paralysis of the trapezius, allowing the shoulder to droop and the superior angle to push out posteriorly. The sternocleidomastoid muscles are intact, as demonstrated by symmetry in strength in turning the head to the right and left. There is no indication of paralysis of the lateral rotators of the shoulder or elbow flexors (suprascapular nerve or upper trunk). Thoracodorsal nerve injury would result in paralysis of the latissimus dorsi muscle, an extensor, and medial rotator of the humerus. GAS 714; N 413; McM 132

43. A rock climber falls on his shoulder, resulting in a chipping off of the lesser tubercle of the humerus. Which of the following structures would most likely have structural and functional damage? (A) Supraspinatus muscle (B) Infraspinatus muscle (C) Subscapularis muscle (D) Teres minor muscle (E) Coracohumeral ligament

43. The answer is C. The subscapularis muscle inserts on the lesser tubercle of the humerus. The supraspinatus, infraspinatus, and teres minor muscles insert on the greater tubercle of the humerus. The coracohumeral ligament attaches to the greater tubercle.

44 A 44-year-old woman is diagnosed with radial nerve palsy. When muscle function is examined at the metacarpophalangeal (MCP), proximal interphalangeal (PIP), and distal interphalangeal (DIP) joints, what findings are most likely to be present? A. Inability to abduct the digits at the MCP joint B. Inability to adduct the digits at the MCP joint C. Inability to extend the MCP joints only D. Inability to extend the MCP, PIP, and DIP joints E. Inability to extend the PIP and DIP joints

44 C. Inability to extend the metacarpophalangeal (MCP) joints. The tendons of the extensor digitorum and extensor digiti minimi muscles, innervated by the radial nerve, are responsible for extension of the MCP and, to a much lesser degree, the proximal (PIP) and distal (DIP) interphalangeal joints. Abduction and adduction of the MCP joints are functions of the interossei, all of which are innervated by the deep ulnar nerve. Extension of the PIP and DIP joints is performed by the lumbricals and interossei. The first two lumbricals are supplied by the median nerve; the other lumbricals and the interossei, by the deep branch of the ulnar nerve. GAS 792, 814-818; N 451; McM 155

44. A 22-year-old female Macarena dancer fell from the stage and complains of elbow pain and inability to supinate her forearm. Which of the following nerves are most likely injured from this accident? (A) Median and ulnar nerves (B) Axillary and radial nerves (C) Radial and musculocutaneous nerves (D) Ulnar and axillary nerves (E) Musculocutaneous and median nerves

44. The answer is C. The supinator and biceps brachii muscles supinate the forearm. The supinator is innervated by the radial nerve, and the biceps brachii is innervated by the musculocutaneous nerve.

45 A 27-year-old male painter is admitted to the hospital after falling from a ladder. Physical examination reveals that the patient is unable to abduct his arm more than 15 degrees and he cannot rotate the arm laterally. A radiographic examination reveals an oblique fracture of the humerus. He has associated sensory loss over the shoulder area. Which of the following injuries will most likely correspond to the symptoms of the physical examination? A. Fracture of the medial epicondyle B. Fracture of the glenoid fossa C. Fracture of the surgical neck of the humerus D. Fracture of the anatomic neck of the humerus E. Fracture of the middle third of the humerus

45 C. Fracture of the surgical neck of the humerus often injures the axillary nerve, which innervates the deltoid and teres minor muscles. Abduction of the humerus between 15 degrees and the horizontal is performed by the deltoid muscle. Lateral rotation of the humerus is mainly performed by the deltoid muscle, teres minor, and the infraspinatus. The deltoid and teres minor are both lost in this case. Fracture of the glenoid fossa would lead to drooping of the shoulder. Fracture of the anatomic neck of the humerus will similarly lead to a drooping of the shoulder but would not necessarily affect abduction of the humerus. It is also quite unusual. Fracture of the middle third of the humerus would most likely injure the radial nerve. The ulnar nerve would be potentially compromised in a fracture of the medial epicondyle of the humerus. GAS 705; N 405; McM 140

46 A 47-year-old woman's right breast exhibited characteristics of peau d'orange, that is, the skin resembled an orange peel. This condition is primarily a result of which of the following? A. Shortening of the suspensory ligaments by cancer in the axillary tail (of Spence) of the breast B. Blockage of cutaneous lymphatic vessels C. Contraction of the retinacula cutis of the areola and nipple D. Invasion of the pectoralis major by the cancer E. Ipsilateral (same side) inversion of the periareolar skin from ductular cancer

46 B. When cutaneous lymphatics of the breast are blocked by cancer, the skin becomes edematous, except where hair follicles cause small indentations of the skin, giving an overall resemblance to orange peel. Shortening of the suspensory ligaments (of Cooper) or retinacula cutis leads to pitting of the overlying skin, pitting that is intensified if the patient raises her arm above her head. Invasion of the pectoralis major by cancer can result in fixation of the breast, seen upon elevation of the ipsilateral limb. Inversion of areolar skin with involvement of the ducts would also be due to involvement of the retinacula cutis. GAS 748; N 181; McM 179

47 A 29-year-old woman is examined in the emergency department after falling from her balcony. Radiographic examination reveals that she has suffered a broken clavicle, with associated internal bleeding. Which of the following vessels is most likely to be injured in clavicular fractures? A. Subclavian artery B. Cephalic vein C. Lateral thoracic artery D. Subclavian vein E. Internal thoracic artery

47 D. The subclavian vein traverses between the clavicle and first rib and is the most superficial structure to be damaged following a fracture of the clavicle. The subclavian artery runs posterior to the subclavian vein, and though it is in the appropriate location, it would likely not be damaged because of its deep anatomic position. The cephalic vein is a tributary to the axillary vein after ascending on the lateral side of the arm. Its location within the body is too superficial and lateral to the site of injury. The lateral thoracic artery is a branch from the axillary artery that runs lateral to the pectoralis minor. It courses inferior and medial from its point of origin from the axillary artery, and it does not maintain a position near the clavicle during its descent. The internal thoracic artery arises from the first part of the subclavian artery before descending deep to the costal cartilages. Its point of origin from the subclavian artery is lateral to clavicular injury. Furthermore, its course behind the costal cartilages is quite medial to the clavicular fracture. GAS 694, 736-737; N 415; McM 129

48 A 68-year-old woman is examined by the senior resident in emergency medicine after she fell on a wet bathroom floor in the shopping center. Physical examination reveals a posterior displacement of the left distal wrist and hand. Radiographic examination reveals an oblique fracture of the radius. Which of the following is the most likely fracture involved in this case? A. Colles' fracture B. Scaphoid fracture C. Bennett's fracture D. Volkmann's ischemic contracture E. Boxer's fracture

48 A. A Colles' fracture is a fracture of the distal end of the radius. The proximal portion of the radius is displaced anteriorly, with the distal bone fragment projecting posteriorly. The displacement of the radius from the wrist often gives the appearance of a dinner fork, thus a Colles' fracture is often referred to as a "dinner fork" deformity. A scaphoid fracture results from a fracture of the scaphoid bone and would thus not cause displacement of the radius. This fracture usually occurs at the narrow aspect ("waist") of the scaphoid bone. Bennett's and boxer's fractures both result from fractures of the metacarpals (first and fifth, respectively). Volkmann's ischemic contracture is a muscular deformity that can follow a supracondylar fracture of the humerus, with arterial laceration into the flexor compartment of the forearm. Ischemia and muscle contracture, with extreme pain, accompany this fracture. GAS 771-774; N 439; McM 124

49 A 34-year-old female skier was taken by ambulance to the hospital after she struck a tree on the ski slope. Imaging gives evidence of a shoulder separation. Which of the following typically occurs in this kind of injury? A. Displacement of the head of the humerus from the glenoid cavity B. Partial or complete tearing of the coracoclavicular ligament C. Partial or complete tearing of the coracoacromial ligament D. Rupture of the transverse scapular ligament E. Disruption of the glenoid labrum

49 B. In shoulder separation, either or both the acromioclavicular and coracoclavicular ligaments can be partially or completely torn through. The acromioclavicular joint can be interrupted and the distal end of the clavicle may deviate upward in a complete separation, while the upper limb droops away inferiorly, causing a "step off" that can be palpated and sometimes observed. Displacement of the head of the humerus is shoulder dislocation, not separation. The coracoacromial ligament is not torn in separation (but it is sometimes used in the repair of the torn coracoclavicular ligament). Disruption of the glenoid labrum often accompanies shoulder dislocation. GAS 711; N 411; McM 136

5 While walking to his classroom building, a firstyear medical student slipped on the wet pavement and fell against the curb, injuring his right arm. Radiographic images showed a midshaft fracture of the humerus. Which pair of structures was most likely injured at the fracture site? A. Median nerve and brachial artery B. Axillary nerve and posterior circumflex humeral artery C. Radial nerve and deep brachial artery D. Suprascapular nerve and artery E. Long thoracic nerve and lateral thoracic artery

5 C. A midshaft humeral fracture can result in injury to the radial nerve and deep brachial artery because they lie in the spiral groove located in the midshaft. Injury to the median nerve and brachial artery can be caused by a supracondylar fracture that occurs by falling on an outstretched hand and partially flexed elbow. A fracture of the surgical neck of the humerus can injure the axillary nerve and posterior circumflex humeral artery. The suprascapular artery and nerve can be injured in a shoulder dislocation. The long thoracic nerve and lateral thoracic artery may be damaged during a mastectomy procedure. GAS 751, 763; N 418; McM 144

5. A 27-year-old patient presents with an inability to draw the scapula forward and downward because of paralysis of the pectoralis minor. Which of the following would most likely be a cause of his condition? (A) Fracture of the clavicle (B) Injury to the posterior cord of the brachial plexus (C) Fracture of the coracoid process (D) Axillary nerve injury (E) Defects in the posterior wall of the axilla

5. The answer is C. pectoralis minor inserts on coracoid process -innervated by the medial and lateral pectoral nerves -depresses the shoulder

50 A 22-year-old male construction worker is admitted to the hospital after he suffers a penetrating injury to his upper limb from a nail gun. Upon physical examination, the patient is unable to flex the distal interphalangeal joints of digits 4 and 5. What is the most likely cause of his injury? A. Trauma to the ulnar nerve near the trochlea B. Trauma to the ulnar nerve at the wrist C. Median nerve damage proximal to the pronator teres D. Median nerve damage at the wrist E. Trauma to spinal nerve root C8

50 A. The nail was fired explosively from the nail gun and then pierced the ulnar nerve near the coronoid process of the ulna and the trochlea of the humerus. Paralysis of the medial half of the flexor digitorum profundus muscle would result (among other significant deficits), with loss of flexion of the distal interphalangeal joints of digits 4 and 5. Ulnar trauma at the wrist would not affect the interphalangeal joints, although it would cause paralysis of interossei, hypothenar muscles, and so on. Median nerve damage proximal to the pronator teres would affect proximal interphalangeal joint flexion and distal interphalangeal joint flexion of digits 2 and 3 as well as thumb flexion. Median nerve injury at the wrist would cause loss of thenar muscles but not long flexors of the fingers. Trauma to spinal nerve ventral ramus C8 would affect all long finger flexors. GAS 784, 814; N 434; McM 149

51 The shoulder of a 44-year-old deer hunter had been penetrated by a bolt released from a crossbow. The bolt had transected the axillary artery just beyond the origin of the subscapular artery. A compress is placed on the wound with deep pressure. After a clamp is placed on the bleeding artery, thought is given to the anatomy of the vessel. What collateral arterial pathways are available to bypass the site of injury? A. Suprascapular with circumflex scapular artery B. Dorsal scapular with thoracodorsal artery C. Posterior circumflex humeral artery with deep brachial artery D. Lateral thoracic with brachial artery E. Supreme thoracic artery with thoracoacromial artery

51 C. The injury has occurred just beyond the third part of the axillary artery. The only collateral arterial channel between the third part of the axillary artery and the brachial artery is between the posterior circumflex humeral and the ascending branch of the profunda brachii, and this anastomotic path is often inadequate to supply the arterial needs of the limb. The posterior circumflex humeral arises from the third part of the axillary artery. It typically anastomoses with a variably small, ascending branch of the profunda brachii branch of the brachial artery. The suprascapular artery anastomoses with the circumflex scapular deep to the infraspinatus. The dorsal scapular artery (passing beneath the medial border of the scapula) has no anastomosis with thoracodorsal within the scope of the injury. The lateral thoracic artery has no anastomoses with the brachial artery. The supreme thoracic artery (from first part of axillary) has no helpful anastomoses with the thoracoacromial (second part of axillary) (GAS Figs. 7-39 and 7-50). GAS 719-721; N 420; McM 139

52 A 17-year-old boy suffered the most common of fractures of the carpal bones when he fell on his outstretched hand. Which bone would this be? A. Trapezium B. Lunate C. Pisiform D. Hamate E. Scaphoid

52 E. The scaphoid (or the older term, navicular) bone is the most commonly fractured carpal bone. GAS 797; N 439; McM 167

53 A 54-year-old male cotton farmer visits the outpatient clinic because of a penetrating injury to his forearm from a baling hook. After the limb is anesthetized, the site of the wound is opened and flushed thoroughly to remove all debris. The patient is not able to oppose the tip of the thumb to the tip of the index finger, as in making the OK sign. He is able to touch the tips of the ring and little fingers to the pad of his thumb. What nerve has most likely been injured? A. Median B. Posterior interosseous C. Radial D. Recurrent median E. Anterior interosseous

53 E. The anterior interosseous nerve is a branch of the median nerve that supplies the flexor pollicis longus, the lateral half of the flexor digitorum profundus, and the pronator quadratus muscles. If it is injured, flexion of the interphalangeal joint of the thumb will be compromised. The median nerve gives rise to the anterior interosseous nerve but is not a direct enough answer as injury to it would result in more widespread effects. The posterior interosseous nerve supplies extensors in the forearm, not flexors. The radial nerve gives rise to the posterior interosseous nerve and is not associated with the anterior interosseous nerve; therefore, it would not have any effect on the flexors of the forearm. The recurrent median nerve is also a branch of the median nerve but supplies the thenar eminence muscles, and its injury would result in problems with opposable motion of the thumb (GAS Fig. 7-87). GAS 784; N 463; McM 151

54 Arthroscopic examination of the shoulder of a 62-year-old woman clearly demonstrated erosion of the tendon within the glenohumeral joint. What tendon was this? A. Glenohumeral B. Long head of triceps brachii C. Long head of biceps brachii D. Infraspinatus E. Coracobrachialis

54 C. The tendon of the long head of the biceps brachii muscle passes through the glenohumeral joint, surrounded by synovial membrane. The glenohumeral is a ligament that attaches to the glenoid labrum. The long head of the triceps brachii arises from the infraglenoid tubercle, beneath the glenoid fossa. The infraspinatus tendon passes posterior to the head of the humerus to insert on the greater tubercle. The coracobrachialis arises from the coracoid process and inserts on the humerus. GAS 707; N 417; McM 137

55 An orthopedic surgeon exposed a muscle in the supraspinous fossa so that she could move it laterally while repairing an injured rotator cuff. As she reflected the muscle from its bed, an artery was exposed crossing the ligament that bridges the notch in the superior border of the scapula. What artery was this? A. Subscapular B. Transverse cervical C. Dorsal scapular D. Posterior circumflex humeral E. Suprascapular

55 E. The suprascapular artery passes over, and the suprascapular nerve passes under, the superior transverse scapular ligament. This ligament bridges the suprascapular notch in the upper border of the scapula, converting the notch to foramen. The artery and nerve then pass deep to the supraspinatus muscle, thereafter supplying it and then passing through the spinoglenoid notch to supply the infraspinatus. The subscapular artery is a branch of the third part of the axillary artery; it divides into circumflex scapular and thoracodorsal branches. The transverse cervical artery courses anterior to this site. The dorsal scapular artery and nerve pass deep to the medial border of the scapula. The posterior circumflex humeral branch of the axillary artery passes through the quadrangular space with the axillary nerve. GAS 719-720; N 414; McM 133

56 A 61-year-old man was hit in the midhumeral region of his left arm by a cricket bat. Physical examination reveals an inability to extend the wrist and loss of sensation on a small area of skin on the dorsum of the hand proximal to the first two fingers. What nerve supplies this specific region of the hand? A. Radial B. Posterior interosseous C. Lateral antebrachial cutaneous D. Medial antebrachial cutaneous E. Dorsal cutaneous of ulnar

56 A. The patient has suffered injury to the radial nerve in the midhumeral region. The nerve that provides sensation to the dorsum of the hand proximal to the thumb and index finger is the superficial branch of the radial nerve. The posterior interosseous nerve supplies a strip of skin on the back of the forearm and wrist extensors. The lateral antebrachial cutaneous nerve is a continuation of the musculocutaneous nerve and supplies the lateral side of the forearm. The medial antebrachial cutaneous is a direct branch of the medial cord and supplies skin of the medial side of the forearm. The dorsal cutaneous branch of the ulnar nerve supplies the medial side of the dorsum of the hand. GAS 761, 792; N 418; McM 144

57 A 45-year-old woman is admitted to the hospital with neck pain. An MRI examination reveals a herniated disc in the cervical region. Physical examination reveals weakness in wrist extension and paraesthesia on the back of her arm and forearm. Which of the following spinal nerves is most likely injured? A. C5 B. C6 C. C7 D. C8 E. T1

57 C. The seventh cervical nerve makes a major contribution to the radial nerve, and this nerve is the prime mover in wrist extension. The dermatome of C7 is in the region described. GAS 745, 787, 790; N 416; McM 153

58 A 22-year-old male football player suffered a wrist injury while falling with force on his outstretched hand. When the anatomic snuffbox is exposed in surgery, which artery is visualized crossing the fractured bone that provides a floor for this space? A. Ulnar B. Radial C. Anterior interosseous D. Posterior interosseous E. Deep palmar arch

58 B. As the radial artery passes from the ventral surface of the wrist to the dorsum, it crosses through the anatomic snuffbox, passing over the scaphoid bone. The ulnar artery at the wrist is located on the medial side of the wrist, passing from beneath the flexor carpi ulnaris to reach Guyon's canal between the pisiform bone and the flexor retinaculum. Guyon's canal is adjacent to but not in communication with the carpal tunnel. The anterior interosseous and posterior interosseous arteries arise from the common interosseous branch of the ulnar artery and pass proximal to distal in the forearm between the radius and ulna, in the flexor and extensor compartments, respectively. The deep palmar branch of the ulnar artery passes between the two heads of the adductor pollicis to anastomose with the radial artery in the palm (GAS Fig. 7-86). GAS 810-814; N 454; McM 166

59 The right shoulder of a 78-year-old woman had become increasingly painful over the past year. Abduction of the right arm caused her to wince from the discomfort. Palpation of the deltoid muscle by the physician produced exquisite pain. Imaging studies reveal intermuscular inflammation extending over the head of the humerus. Which structure was inflamed? A. Subscapular bursa B. Infraspinatus muscle C. Glenohumeral joint cavity D. Subacromial bursa E. Teres minor muscle

59 D. The patient is suffering from subacromial or subdeltoid bursitis. (If the pain on palpation is less when the arm has been elevated to the horizontal, the bursitis may be thought of as being more subacromial, that is, associated more with the supraspinatus tendon perhaps, for such a bursa may be drawn back under the acromion when the limb is abducted.) The subscapular bursa, beneath the subscapularis muscle, would not present as superficial pain. It can communicate with the glenohumeral joint cavity. Inflammation or arthritic changes within the glenohumeral joint present as more generalized shoulder pain than that present here. The teres minor muscle and tendon are located inferior to the point of marked discomfort. GAS 708, 713; N 424; McM 136

6 An 18-year-old man is brought to the emergency department after an injury while playing rugby. Imaging reveals a transverse fracture of the humerus about 1 inch proximal to the epicondyles. Which nerve is most frequently injured by the jagged edges of the broken bone at this location? A. Axillary B. Median C. Musculocutaneous D. Radial E. Ulnar

6 B. A supracondylar fracture often results in injury to the median nerve. The course of the median nerve is anterolateral, and at the elbow it lies medial to the brachial artery on the brachialis muscle. The axillary nerve passes posteriorly through the quadrangular space, accompanied by the posterior circumflex humeral artery, and winds around the surgical neck of the humerus. Injury to the surgical neck may damage the axillary nerve. The musculocutaneous nerve pierces the coracobrachialis muscle and descends between the biceps brachii and brachialis muscle. It continues into the forearm as the lateral antebrachial cutaneous nerve. The ulnar nerve descends behind the medial epicondyle in its groove and is easily injured and produces "funny bone" symptoms. GAS 763, 764; N 434; McM 143

6. A 22-year-old patient received a stab wound in the chest that injured the intercostobrachial nerve. Which of the following conditions results from the described lesion of the nerve? (A) Inability to move the ribs (B) Loss of tactile sensation on the lateral aspect of the arm (C) Absence of sweating on the posterior aspect of the arm (D) Loss of sensory fibers from the second intercostal nerve (E) Damage to the sympathetic preganglionic fibers

6. The answer is D. The intercostobrachial nerve arises from the lateral cutaneous branch of the second intercostal nerve and pierces the intercostal and serratus anterior muscles. It may communicate with the medial brachial cutaneous nerve, and it supplies skin on the medial side of the arm. It contains no skeletal motor fi bers but does contain sympathetic postganglionic fi bers, which supply sweat glands.

60 A 55-year-old male metallurgist had been diagnosed with carpal tunnel syndrome. To begin the operation, an anesthetic injection into his axillary sheath was given instead of general anesthesia. From which of the following structures does the axillary sheath take origin? A. Superficial fascia of the neck B. Superficial cervical investing fascia C. Buccopharyngeal fascia D. Clavipectoral fascia E. Prevertebral fascia

60 E. The axillary sheath is a fascial continuation of the prevertebral layer of the deep cervical fascia extending into the axilla. It encloses the nerves of the neurovascular bundle of the upper limb. Superficial fascia is loose connective tissue between the dermis and the deep investing fascia and contains fat, cutaneous vessels, nerves, lymphatics, and glands. The buccopharyngeal fascia covers the buccinator muscles and the pharynx mingles with the pretracheal fascia. The clavipectoral muscle invests the clavicle and pectoralis minor muscle. The axillary fascia is continuous with the pectoral and latissimus dorsi fascia and forms the hollow of the armpit. GAS 721, 731; N 412; McM 361

61 A 45-year-old woman is admitted to the hospital with neck pain. A computed tomography (CT) scan reveals a tumor on the left side of her oral cavity. The tumor and related tissues are removed and a radical neck surgical procedure is performed. Two months postoperatively the patient's left shoulder droops quite noticeably. Physical examination reveals distinct weakness in turning her head to the right and impairment of abduction of her left upper limb to the level of the shoulder. Which of the following structures was most likely injured during the radical neck surgery? A. Suprascapular nerve B. Long thoracic nerve C. Spinal accessory nerve D. The junction of spinal nerves C5 and C6 of the brachial plexus E. Radial nerve

61 C. The spinal accessory nerve (CN XI) arises from the ventral rootlets of C1 to C4 that ascend through the foramen magnum to then exit the cranial cavity through the jugular foramen. It innervates the sternocleidomastoid and trapezius muscles, which function in head rotation and raising of the shoulders. The suprascapular nerve receives fibers from C5-6 (occasionally from C4 if the plexus is "prefixed") and innervates the supraspinatus muscle, which is responsible for the first 15 degrees of arm abduction. Erb's point of the brachial plexus is at the union of C5-6 spinal nerves. The long thoracic nerve arises from plexus routes C5, 6, and 7, and supplies the serratus anterior. GAS 714; N 33; McM 132

62 A 23-year-old male basketball player is admitted to the hospital after injuring his shoulder during a game. Physical and radiographic examinations reveal total separation of the shoulder (Fig. 6-5). Which of the following structures has most likely been torn? A. Glenohumeral ligament B. Coracoacromial ligament C. Tendon of long head of biceps brachii D. Acromioclavicular ligament E. Transverse scapular ligament

62 D. The acromioclavicular ligament connects the clavicle to the coracoid process of the scapula. Separation of the shoulder (dislocation of the acromioclavicular [AC] joint) is associated with damage to the acromioclavicular ligament (capsule of the AC joint) and, in more severe injuries, disruption of the coracoclavicular ligaments (conoid and trapezoid portions). The glenohumeral ligament may be injured by an anterior dislocation of the humerus but is not likely to be injured by a separated shoulder. The coracoacromial ligament, transverse scapular ligament, and tendon of the long head of triceps brachii are not likely to be injured by separation of the shoulder. GAS 706; N 408; McM 136

63 A 35-year-old male body builder has enlarged his shoulder muscles to such a degree that the size of the quadrangular space is greatly reduced. Which of the following structures would most likely be compressed in this condition? A. Axillary nerve B. Anterior circumflex humeral artery C. Cephalic vein D. Radial nerve E. Subscapular artery

63 A. The quadrangular space is bordered medially by the long head of the triceps brachii muscle, laterally by the surgical neck of the humerus, superiorly by the teres minor and subscapularis muscles, and inferiorly by the teres major muscle. Both the axillary nerve and posterior circumflex humeral vessels traverse this space. The other structures listed are not contained within the quadrangular space. The cephalic vein is located in the deltopectoral triangle, and the radial nerve is located in the triangular interval. GAS 718-720, 730; N 413; McM 139

64 A 43-year-old woman visits the outpatient clinic with a neurologic problem. Diagnostically, she cannot hold a piece of paper between her thumb and the lateral side of her index finger without flexing the distal joint of her thumb. This is a positive Froment's sign, which is consistent with ulnar neuropathy. Weakness of which specific muscle causes this sign to appear? A. Flexor pollicis longus B. Adductor pollicis C. Flexor digiti minimi D. Flexor carpi radialis E. Extensor indicis

64 B. Froment's sign is positive for ulnar nerve palsy. More specifically it tests the action of the adductor pollicis muscle. The patient is asked to hold a sheet of paper between the thumb and a flat palm. The flexor pollicis longus is innervated by the anterior interosseous branch of the median nerve. The flexor digiti minimi is innervated by the deep branch of the ulnar nerve and would not be used to hold a sheet of paper between the thumb and palm. The flexor carpi radialis is innervated by the median nerve, and the extensor indicis is innervated by the radial nerve (Fig. 6-14). GAS 814-816, 826; N 464; McM 157

65 A 48-year-old female piano player visited the outpatient clinic with numbness and tingling in her left hand. A diagnosis was made of nerve compression in the carpal tunnel, and the patient underwent an endoscopic nerve release. Two weeks postoperatively the patient complained of a profound weakness in the thumb, with loss of thumb opposition. The sensation to the hand, however, was unaffected. Which of the following nerves was injured during the operation? A. The first common digital branch of the median nerve B. The second common digital branch of the median nerve C. Recurrent branch of median nerve D. Deep branch of the ulnar nerve E. Anterior interosseus nerve

65 C. The recurrent branch of the median nerve innervates the thenar muscles (opponens pollicis, abductor pollicis brevis, and flexor pollicis brevis) and is not responsible for any cutaneous innervation. Damage to the palmar cutaneous branches of the median nerve or to the ulnar nerve would not cause weakness of opposition of the thumb for they are principally sensory in function. The deep branch of the ulnar nerve supplies the hypothenar muscles, adductor and abductor muscles of digits 2-5, and does not innervate the abductor pollicis brevis. GAS 817; N 463; McM 159

66 A 19-year-old man had suffered a deep laceration to an upper limb when he stumbled and fell on a broken bottle. On examination of hand function it is observed that he is able to extend the metacarpophalangeal joints of all his fingers in the affected limb. He cannot extend the interphalangeal (IP) joints of the fourth and fifth digits, and extension of the IP joints of the second and third digits is very weak. There is no apparent sensory deficit in the hand. Which of the following nerves has most likely been injured? A. Radial nerve at the elbow B. Median nerve at the wrist C. Ulnar nerve in midforearm D. Deep branch of ulnar nerve E. Recurrent branch of the median nerve

66 D. Injury to the deep branch of the ulnar nerve results in paralysis of all interosseous muscles and the lumbrical muscles of digits 4 and 5. Extension of the metacarpophalangeal joints is intact, a function of the radial nerve. Interphalangeal extension of digits 4 and 5 is absent, due to the loss of all interosseous muscle and the lumbricals of digits 4 and 5. Some weak interphalangeal joint extension is still present in digits 2 and 3 because the lumbricals of these two fingers are innervated by the median nerve. The radial nerve and the median nerve appear to be intact in this case. If the ulnar nerve were injured in the midforearm region, there would be sensory loss in the palm and digits 4 and 5 and on the dorsum of the hand. The recurrent branch of the median nerve supplies the thenar muscles; it does not supply lumbricals. Moreover, paralysis of this nerve would have no effect on the interphalangeal joints. GAS 814; N 464; McM 159

67 A 41-year-old woman is scheduled for a latissimus dorsi muscle flap to cosmetically augment the site of her absent left breast after mastectomy. Part of the latissimus dorsi muscle is advanced to the anterior thoracic wall, based upon arterial supply provided in part by the artery that passes through the triangular space of the axilla. Which artery forms the vascular base of this flap? A. Circumflex scapular artery B. Dorsal scapular artery C. Transverse cervical artery D. Lateral thoracic artery E. Thoracoacromial artery

67 A. The circumflex scapular artery passes through the triangular space after arising from the subscapular artery. It provides superficial branches to the overlying latissimus dorsi, whereas its deep portion passes into the infraspinous fossa to anastomose with the suprascapular artery. The dorsal scapular artery passes between the ventral rami of the brachial plexus and then deep to the medial border of the scapula. The transverse cervical artery arises from the thyrocervical trunk at the root of the neck and can provide origin for a dorsal scapular branch. The lateral thoracic and thoracoacromial arteries are branches of the second part of the axillary artery and provide no supply to the latissimus dorsi. GAS 721; N 414; McM 141

68 A 31-year-old male hockey player fell on his elbow and is admitted to the emergency department. Radiographic examination reveals a fracture of the surgical neck of the humerus, producing an elevation and adduction of the distal fragment. Which of the following muscles would most likely cause the adduction of the distal fragment? A. Brachialis B. Teres minor C. Pectoralis major D. Supraspinatus E. Pectoralis minor

68 C. The surgical neck of the humerus is a typical site of fractures. The fracture line lies above the insertions of the pectoralis major, teres major, and latissimus dorsi muscles. The supraspinatus muscle abducts the proximal fragment, whereas the distal fragment is elevated and adducted. The elevation results from contraction of the deltoid, biceps brachii, and coracobrachialis muscles. The adduction is due to the action of pectoralis major, teres major, and latissimus dorsi. GAS 705; N 413; McM 140

69 A 74-year-old woman is admitted to the emergency department after stumbling over her pet dog. Radiographic examination reveals a fracture of the upper third of the right radius, with the distal fragment of the radius and hand pronated. The proximal end of the fractured radius deviates laterally. Which of the following muscles is primarily responsible for the lateral deviation? A. Pronator teres B. Supinator C. Pronator quadratus D. Brachioradialis E. Brachialis

69 B. The fracture line of the upper third of the radius lies between the bony attachments of the supinator and the pronator teres muscles. The distal radial fragment and hand are pronated due to unopposed contraction of pronator teres and pronator quadratus muscles. The proximal fragment deviates laterally by the unopposed contraction of the supinator muscle. The brachioradialis inserts distally on the radius. The brachialis inserts on the coronoid process of the ulna and would not be involved in the lateral deviation of the radius. GAS 772-774; N 431; McM 152

7 A 52-year-old female band director suffered problems in her right arm several days after strenuous field exercises for a major athletic tournament. Examination in the orthopedic clinic reveals wrist drop and weakness of grasp but normal extension of the elbow joint. There is no loss of sensation in the affected limb. Which nerve was most likely affected? A. Ulnar B. Anterior interosseous C. Posterior interosseous D. Median E. Superficial radial

7 C. The radial nerve descends posteriorly between the long and lateral heads of the triceps brachii muscle and passes inferolaterally on the back of the humerus between the medial and lateral heads of the triceps brachii muscle. It eventually enters the anterior compartment and descends to enter the cubital fossa, where it divides into superficial and deep branches. The deep branch of the radial nerve winds laterally around the radius and runs between the two heads of the supinator muscle and continues as the posterior interosseous nerve, innervating extensor muscles of the forearm. Because this injury does not result in loss of sensation over the skin of the upper limb, it is likely that the superficial branch of the radial nerve is not injured. If the radial nerve were injured very proximally, the woman would not be able to extend her elbow. The branches of the radial nerve to the triceps brachii muscle arise proximal to where the nerve runs in the spiral groove. The anterior interosseous nerve arises from the median nerve and supplies the flexor digitorum profundus, flexor pollicis longus, and pronator quadratus muscles, none of which seem to be injured in this example. Injury to the median nerve causes a characteristic flattening (atrophy) of the thenar eminence. GAS 744-756, 761, 787, 792; N 431; McM 152

7. A 16 year-old boy fell from a motorcycle, and his radial nerve was severely damaged because of a fracture of the midshaft of the humerus. Which of the following conditions would most likely result from this accident? (A) Loss of wrist extension leading to wrist drop (B) Weakness in pronating the forearm (C) Sensory loss over the ventral aspect of the base of the thumb (D) Inability to oppose the thumb (E) Inability to abduct the fingers

7. The answer is A. Injury to the radial nerve results in loss of wrist extension, leading to wrist drop. The median nerve innervates the pronator teres, pronator quadratus, and opponens pollicis muscles and the skin over the ventral aspect of the thumb. The ulnar nerve innervates the dorsal interosseous muscles, which act to abduct the fi ngers.

70 A 12-year-old boy lacerated the palmar surface of the wrist while playing with a sharp knife. The cut ends of a tendon could be seen within the wound in the exact midline of the wrist. Which tendon lies in this position in most people? A. Palmaris longus B. Flexor carpi radialis C. Abductor pollicis longus D. Flexor carpi ulnaris E. Flexor pollicis longus

70 A. The palmaris longus passes along the midline of the flexor surface of the forearm. The flexor carpi radialis is seen in the lateral portion of the forearm superficially, passing over the trapezium to insert at the base of the second metacarpal. The abductor pollicis longus tendon is laterally located in the wrist, where it helps form the lateral border of the anatomic snuffbox. The flexor carpi ulnaris tendon can be seen and palpated on the medial side of the wrist ventrally. The flexor pollicis longus tendon passes deep through the carpal tunnel. GAS 777; N 432; McM 158

71 A 22-year-old male medical student was seen in the emergency department with a complaint of pain in his right hand. He confessed that he had hit a vending machine in the hospital when he did not receive his soft drink after inserting money twice. The medial side of the dorsum of the hand was quite swollen, and one of his knuckles could not be seen when he "made a fist." The physician made a diagnosis of a "boxer's fracture." What was the nature of the impatient student's injury? A. Fracture of the styloid process of the ulna B. Fracture of the neck of the fifth metacarpal C. Colles' fracture of the radius D. Smith's fracture of the radius E. Bennett's fracture of the thumb

71 B. The student had broken the neck of the fifth metacarpal when hitting the machine with his fist. This is the more common type of "boxer's fracture." Neither a fracture of the ulnar styloid nor a Colles' fracture nor Smith's fracture of the distal radius would present with the absence of a knuckle as observed here. Bennett's fracture involves dislocation of the carpometacarpal joint of the thumb. Indications are that the injury is on the medial side of the hand, not the wrist, nor the lateral side of the hand or wrist. GAS 793-794; N 440; McM 167

72 Fine motor function in the right hand of a 14-yearold girl with scoliosis since birth appeared to be quite reduced, including opposition of the thumb, abduction and adduction of the digits, and interphalangeal joint extension. Radiography confirmed that her severe scoliosis was causing marked elevation of the right first rib. Long flexor muscles of the hand and long extensors of the wrist appear to be functioning within normal limits. There is notable anesthesia of the skin on the medial side of the forearm; otherwise, sensory function in the limb is intact. Which of the following neural structures is most likely impaired? A. Median nerve B. Middle trunk of the brachial plexus C. Radial nerve D. Lower trunk of the brachial plexus E. T1 ventral ramus

72 E. Scoliosis (severe lateral curvature of the spine) in the patient is causing compression or stretching of the T1 spinal nerve ramus by the first rib as the nerve ascends to join C8 and form the lower trunk of the brachial plexus. T1 provides sensation for the medial side of the forearm, via the medial antebrachial cutaneous nerve from the medial cord of the brachial plexus. T1 is the principal source of motor supply to all of the intrinsic muscles in the palm. Its dysfunction affects all fine motor movements of the digits. Long flexors of the fingers are intact; therefore, the median nerve and ulnar nerve are not injured. The extensors of the wrist are functional; therefore, the radial nerve is not paralyzed. The only sensory disturbance is that of the T1 dermatome. GAS 695-700, 744-745; N 161; McM 94

73 A 23-year-old woman had a painful injury to her hand in a dry ski-slope competition, in which she fell and caught her thumb in the matting. Radiographic and physical examinations reveal rupture of the ulnar collateral ligament of the metacarpophalangeal joint of the thumb. Lidocaine is injected into the area to relieve the pain, and she is scheduled for a surgical repair. From which of the following clinical problems is she suffering? A. De Quervain's syndrome B. Navicular bone fracture C. Boxer's thumb D. Gamekeeper's thumb E. Bennett's thumb

73 D. Interestingly, "gamekeeper's thumb" was a term coined because this injury was most commonly associated with Scottish gamekeepers who, it is said, killed small animals such as rabbits by breaking their necks between the ground and the gamekeeper's thumb and index finger. The resulting valgus force on the abducted metacarpophalangeal (MCP) joint caused injury to the ulnar collateral ligament. These days this injury is more commonly seen in skiers who land awkwardly with their hand braced on a ski pole, causing the valgus force on the thumb as is seen in this patient. Whereas the term "skier's thumb" is sometimes used, "gamekeeper's thumb" is still in common usage. GAS 795-796; N 441; McM 163

74 A 26-year-old male power lifter visits the outpatient clinic with a painful shoulder. Radiographic examination reveals tendinopathy of the long head of the biceps brachii muscle. Which of the following conditions will most likely be present during physical examination? A. Pain is felt in the anterior shoulder during forced contraction B. Pain is felt in the lateral shoulder during forced contraction C. Pain is felt during abduction and flexion of the shoulder joint D. Pain is felt during extension and adduction of the shoulder joint E. Pain is felt in the lateral shoulder during flexion of the shoulder joint

74 A. The long head of the biceps brachii muscle assists in shoulder flexion and during a tendinopathy would cause pain in the anterior compartment of the shoulder, where it originates at the supraglenoid tubercle. Also, forced contraction would cause a greater tension force on the tendon. GAS 732; N 419; McM 136

75 A 43-year-old female tennis player visits the outpatient clinic with pain over the right lateral epicondyle of her elbow. Physical examination reveals that the patient has lateral epicondylitis. Which of the following tests should be performed during physical examination to confirm the diagnosis? A. Nerve conduction studies B. Evaluation of pain experienced during flexion and extension of the elbow joint C. Observing the presence of pain when the wrist is extended against resistance D. Observing the presence of numbness and tingling in the ring and little fingers when the wrist is flexed against resistance E. Evaluation of pain felt over the styloid process of radius during brachioradialis contraction

75 C. The common extensor tendon originates from the lateral epicondyle, and inflammation of this tendon is lateral epicondylitis, nicknamed "tennis elbow" because the tendon is often irritated during the backhand stroke in tennis. Because the extensors of the wrist originate as part of the common extensor tendon, extension of the wrist will exacerbate the pain of lateral epicondylitis. GAS 768, 785; N 427; McM 152

76 A male skier had a painful fall against a rocky ledge. Radiographic findings revealed a hairline fracture of the surgical neck of the humerus. The third-year medical student assigned to this patient was asked to determine whether there was injury to the nerve associated with the area of injury. Which of the following tests would be best for checking the status of the nerve? A. Have the patient abduct the limb while holding a 10 lb weight B. Have the patient shrug the shoulders C. Test for presence of skin sensation over the lateral side of the shoulder D. Test for normal sensation over the medial skin of the axilla E. Have the patient push against an immovable object like a wall and assess the position of the scapula

76 C. The axillary nerve passes dorsally around the surgical neck of the humerus (accompanied by the posterior circumflex humeral artery) and can be injured when the humerus is fractured at that location. The axillary nerve provides sensation to the skin over the upper, lateral aspect of the shoulder. Therefore, although the patient might not be able to abduct the arm because of the injury, a simple test of skin sensation can indicate whether there is associated nerve injury of the axillary nerve (CN XI). Shrugging the shoulders can help assess trapezius function, thereby testing the spinal accessory nerve. Intact sensation of the skin on the medial aspect of the axilla and arm is an indication that the radial and intercostobrachial nerves are functional. Pushing against an immovable object tests the serratus anterior muscle and the long thoracic nerve. GAS 718-720; N 465; McM 139

77 A 27-year-old man had lost much of the soft tissue on the dorsum of his left hand in a motorcycle crash. Imaging studies show no other upper limb injuries. Because function of the left extensor carpi radialis longus and brevis tendons was lost, it was decided to replace those tendons with the palmaris longus tendons from both forearms because of their convenient location and relative unimportance. Postoperatively, it is found that sensation is absent in both hands on the lateral palm and palmar surfaces of the first three digits; there is also paralysis of thumb opposition. What is the most likely cause of the sensory deficit and motor loss in both thumbs? A. Bilateral loss of spinal nerve T1 with fractures of first rib bilaterally B. Lower plexus (lower trunk) trauma C. Dupuytren contracture D. Left radial nerve injury in the posterior compartment of the forearm E. The palmaris longus was absent bilaterally; the nerve normally beneath it looked like a tendon and was cut

77 E. The surgeon took the distal segments of the median nerves from both forearms, mistakenly believing them to be palmaris longus tendons. Both of the structures lie in the midline of the ventral surface of the distal forearm and are often of similar appearance in color and diameter. The nerve is located deep to the tendon, when the tendon is present, but when the tendon is absent, the nerve appears to be where the tendon belongs. There is no evidence of rib fractures; even so, a fractured rib would not explain loss of sensation on the lateral portion of the palm. Lower plexus trauma (C8, T1) would result in paralysis of forearm flexor muscles and all intrinsic hand muscles and sensory loss over the medial dorsum of the hand, in addition to palmar sensory loss. Dupuytren's contracture is a flexion contracture of (usually) digits four and five from connective tissue disease in the palm. Radial nerve injury in the posterior forearm would affect metacarpophalangeal joint extension, thumb extension, and so on, not palmar disturbances. GAS 784-785; N 432; McM 150

78 A 15-year-old boy received a shotgun wound to the ventral surface of the upper limb. Three months after the injury the patient exhibits a complete claw hand but can extend his wrist. What is the nature of this patient's injury? A. The ulnar nerve has been severed at the wrist. B. The median nerve has been injured in the carpal tunnel. C. The median and ulnar nerves are damaged at the wrist. D. The median and ulnar nerves have been injured at the elbow region. E. The median, ulnar, and radial nerves have been injured at midhumerus.

78 C. Trauma both to the median and ulnar nerves at the wrist results in total clawing of the fingers. The metacarpophalangeal joints of all digits are extended by the unopposed extensors because the radial nerve is intact. All interossei and lumbricals are paralyzed because the deep branch of the ulnar nerve supplies all of the interossei; lumbricals I and II are paralyzed, for they are innervated by the median nerve; lumbricals III and IV are paralyzed, for they receive supply from the deep ulnar nerve. The interossei and lumbricals are responsible for extension of the interphalangeal joints. When they are paralyzed, the long flexor tendons pull the fingers into a position of flexion, completing the "claw" appearance. If the median nerve were intact, the clawing would be less noticeable in the index and long fingers because the two lumbricals would still be capable of some degree of extension of those interphalangeal joints. If the median nerve alone is injured in the carpal tunnel, there would be loss of thenar opposition but not clawing. If the median and ulnar nerves are both transected at the elbow, the hand appears totally flat because of the loss of long flexors, in addition to intrinsic paralysis. GAS 784, 814-818; N 434; McM 157

79 A 68-year-old woman fell when she missed the last step from her motor home. Radiographic examination at the local medical care center reveals a fracture of the distal radius. The distal fragment of the radius is angled forward. What name is commonly applied to this type of injury? A. Colles' fracture B. Scaphoid fracture C. Bennett's fracture D. Smith's fracture E. Boxer's fracture

79 D. Colles' fracture is a fracture of the distal radius with the distal fragment displaced dorsally. Smith's fracture involves the distal fragment displaced in a volar direction. Smith's fracture is sometimes referred to as a reverse Colles' fracture. GAS 771-774; N 440; McM 153

8 A 32-year-old woman is admitted to the emergency department after an automobile collision. Radiologic examination reveals multiple fractures of the humerus. Flexion and supination of the forearm are severely weakened. She also has loss of sensation on the lateral surface of the forearm. Which of the following nerves has most likely been injured? A. Radial B. Musculocutaneous C. Median D. Lateral cord of brachial plexus E. Lateral cutaneous nerve of the forearm

8 B. The musculocutaneous nerve supplies the biceps brachii and brachialis muscles, which are the flexors of the forearm at the elbow. The musculocutaneous nerve continues as the lateral antebrachial cutaneous nerve, which supplies sensation to the lateral side of the forearm (with the forearm in the anatomic position). The biceps brachii muscle is the most powerful supinator muscle. Injury to this nerve would result in weakness of supination and forearm flexion and lateral forearm sensory loss. Injury to the radial nerve would result in weakened extension and a characteristic wrist drop. Injury to the median nerve causes paralysis of flexor digitorum superficialis muscle and other flexors in the forearm and results in a characteristic flattening of the thenar eminence. The lateral cord of the brachial plexus gives origin both to the musculocutaneous and lateral pectoral nerves. There is no indication of pectoral paralysis or weakness. Injury to the lateral cord can result in weakened flexion and supination in the forearm, and weakened adduction and medial rotation of the arm. The lateral cutaneous nerve of the forearm is a branch of the musculocutaneous nerve and does not supply any motor innervation. Injury to the musculocutaneous nerve alone is unusual but can follow penetrating injuries. GAS 760; N 462; McM 139

8. A patient comes in complaining that she cannot flex her proximal interphalangeal joints. Which of the following muscles appear(s) to be paralyzed on further examination of her fi nger? (A) Palmar interossei (B) Dorsal interossei (C) Flexor digitorum profundus (D) Flexor digitorum superfi cialis (E) Lumbricals

8. The answer is D. The flexor digitorum superficialis muscle flexes the proximal interphalangeal joints. The flexor digitorum profundus muscle flexes the DIP joints. The palmar and dorsal interossei and lumbricals can flex metacarpophalangeal joints and extend the interphalangeal joints. The palmar interossei adduct the fingers, and the dorsal interossei abduct the fi ngers.

80 It was reported by the sports media that the outstanding 27-year-old shortstop for a New York team would miss a number of baseball games. He was hit on a fingertip while attempting to catch a ball barehanded. A tendon had been torn. The team doctor commented that the ballplayer could not straighten the last joint of the long finger of his right hand, and the finger would require surgery. From what injury did the ballplayer suffer? A. Claw hand deformity B. Boutonnière deformity C. Swan-neck deformity D. Dupuytren's contracture E. Mallet finger

80 E. The extensor tendons of the fingers insert distally on the distal phalanx of each digit. If the tendon is avulsed, or the proximal part of the distal phalanx is detached, the distal interphalangeal joint (DIP) is pulled into total flexion by the unopposed flexor digitorum profundus muscle. This result gives the digit the appearance of a mallet. In boutonnière deformity, the central portion of the extensor tendon expansion is torn over the proximal interphalangeal (PIP) joint, allowing the tendon to move palmarward, causing the tendon to act as a flexor of the PIP joint. This causes the DIP joint to be hyperextended. Swanneck deformity involves slight flexion of the metacarpophalangeal (MCP) joints, hyperextension of PIP joints, and slight flexion of DIP joints. This condition results most often from shortening of the tendons of intrinsic muscles, as in rheumatoid arthritis. Dupuytren's contracture results from connective tissue disorder in the palm, usually causing irreversible flexion of digits 4 and 5. Claw hand occurs with lesions to the median and ulnar nerves at the wrist. In this clinical problem all intrinsic muscles are paralyzed, including the extensors of the interphalangeal joints. The MCP joint extensors, supplied by the radial nerve, and the long flexors of the fingers, supplied more proximally in the forearm by the median and ulnar nerves, are intact and are unopposed, pulling the fingers into the "claw" appearance. GAS 787-790, 802; N 451; McM 153

81 A 31-year-old female figure skater is examined in the emergency department following an injury that forced her to withdraw from competition. When her male partner missed catching her properly from an overhead position, he grasped her powerfully, but awkwardly, by the forearm. Clinical examination demonstrated a positive Ochsner test, inability to flex the distal interphalangeal joint of the index finger on clasping the hands. In addition, she is unable to flex the terminal phalanx of the thumb and has loss of sensation over the thenar half of the hand. What is the most likely nature of her injury? A. Median nerve injured within the cubital fossa B. Anterior interosseous nerve injury at the pronator teres C. Radial nerve injury at its entrance into the posterior forearm compartment D. Median nerve injury at the proximal skin crease of the wrist E. Ulnar nerve trauma halfway along the forearm

81 A. Because the median nerve is injured within the cubital fossa, the long flexors are paralyzed, including the flexor pollicis longus muscle. The flexor pollicis longus would not be paralyzed if the median nerve were injured at the wrist. Lateral palm sensory loss confirms median nerve injury. If only the anterior interosseous nerve were damaged, there would be no cutaneous sensory deficit. The radial nerve supplies wrist extensors, long thumb abductor, and metacarpophalangeal joint extensors. The ulnar nerve does not supply sensation to the lateral palm. GAS 768, 804; N 434; McM 149

82 A 19-year-old man fell from a cliff when he was hiking in the mountains. He broke his fall by grasping a tree branch, but he suffered injury to the C8 to T1 spinal nerve ventral rami. Sensory tests would thereafter confirm the nature of his neurologic injury by the sensory loss in the part of the limb supplied by which of the following? A. Lower lateral brachial cutaneous nerve B. Musculocutaneous nerve C. Intercostobrachial nerve D. Medial antebrachial cutaneous nerve E. Median nerve

82 D. In a lesion of the lower trunk of the brachial plexus, or the C8 and T1 ventral rami, there is sensory loss on the medial forearm and the medial side of hand (dorsal and ventral). The medial cord is an extension of the lower trunk. The medial cord gives origin to the medial antebrachial cutaneous nerve, which supplies the T1 dermatome of the medial side of the antebrachium. The lower lateral brachial cutaneous nerve arises from the radial nerve, C5 and C6. The musculocutaneous nerve arises from the lateral cord, ending in the lateral antebrachial cutaneous nerve, with C5 and C6 dermatome fibers. The intercostobrachial nerve is the lateral cutaneous branch of the T2 ventral primary ramus and supplies skin on the medial side of the arm. The median nerve distributes C6 and C7 sensory fibers to the lateral part of the palm, thumb, index, long finger, and half of the ring finger. GAS 738-745; N 416; McM 138

83 The mastectomy procedure on a 52-year-old woman involved excision of the tumor and removal of lymph nodes, including the pectoral, central axillary, and infraclavicular groups. Six months after her mastectomy, the patient complains to her personal physician of an unsightly deep hollow area inferior to the medial half of the clavicle, indicating a significant area of muscle atrophy and loss. She states that the disfigurement has taken place quite gradually since her mastectomy. Physical examination reveals no obvious motor or sensory deficits. What was the most likely cause of the patient's cosmetic problem? A. Part of the pectoralis major muscle was cut and removed in the mastectomy B. The pectoralis minor muscle was removed entirely in the surgery C. A branch of the lateral pectoral nerve was cut D. The medial pectoral nerve was cut E. The lateral cord of the brachial plexus was injured

83 C. The first branch of the lateral pectoral nerve is typically the only source of motor supply to the clavicular head of the pectoralis major muscle. If it is injured (as in this case of an iatrogenic injury when the infraclavicular nodes were removed), this part of the muscle undergoes atrophy, leaving an infraclavicular cosmetic deficit. The remainder of the lateral pectoral nerve joins the medial pectoral nerve in a neural arch that provides motor supply to the remaining parts of the pectoralis major and the pectoralis minor. Physical examination reveals no obvious motor or sensory deficits. Loss of the medial pectoral nerve would have no effect on the clavicular head of pectoralis major and might not be discernible. Injury to the lateral cord would lead to loss not only of all of the lateral pectoral nerve but also the musculocutaneous nerve, resulting in biceps brachii and brachialis paralysis and lateral antebrachial sensory loss. GAS 724, 742; N 415; McM 141

84 A 54-year-old woman was found unconscious on the floor, apparently after a fall. She was admitted to the hospital, and during physical examination it was observed that she had unilateral absence of her brachioradialis reflex. Which spinal nerve is primarily responsible for this reflex in the majority of cases? A. C5 B. C6 C. C7 D. C8 E. T1

84 B. The C6 spinal nerve is primarily responsible for the brachioradialis reflex. C5 and C6 are both involved in the biceps brachii reflex; C5 for motor, C6 for the sensory part of the reflex arc; C7 is the key spinal nerve in the triceps reflex. GAS 755; N 432; McM 143

85 A 43-year-old man is admitted to the hospital, having suffered a whiplash injury when his compact automobile was struck from behind by a sports utility vehicle. MRI examination reveals some herniation of a disc in the cervical region. Physical examination reveals that the patient has lost elbow extension; there is absence of his triceps reflex and loss of extension of the metacarpophalangeal joints on the ipsilateral side. Which of the following spinal nerves is most likely affected? A. C5 B. C6 C. C7 D. C8 E. T1

85 C. C7 is the main spinal nerve that contributes to the radial nerve and innervates the triceps brachii. Absence of the triceps reflex is usually indicative of a C7 radiculopathy or injury. GAS 745-746, 756; N 416; McM 144

86 A 29-year-old patient has a dislocated elbow in which the ulna and medial part of the distal humerus have become separated. What classification of joint is normally formed between these two bones? A. Trochoid B. Ginglymus C. Enarthrodial D. Synarthrosis E. Sellar

86 B. Ginglymus joint is the correct technical term to describe a hinge joint. It allows motion in one axis (flexion and extension in the case of the humeroulnar joint) and is therefore a uniaxial joint. The other types of joints listed allow motion in more than one axis. GAS 764; N 442; McM 120

87 A 45-year-old female motorcyclist, propelled over the handlebars of her bike by an encounter with a rut in the road, lands on the point of one shoulder. She is taken by ambulance to the emergency department. During physical examination, the arm appears swollen, pale, and cool. Any movement of the arm causes severe pain. Radiographic examination reveals a fracture and a large hematoma, leading to diagnosis of Volkmann's ischemic contracture. At which of the following locations has the fracture most likely occurred? A. Surgical neck of humerus B. Radial groove of humerus C. Supracondylar line of humerus D. Olecranon E. Lateral epicondyle

87 C. A fracture of the humerus just proximal to the epicondyles is called a supracondylar fracture. This is the most common cause of a Volkmann ischemic fracture. The sharp bony fragment often lacerates the brachial (or other) artery, with bleeding into the flexor compartment. Diminution of arterial supply to the compartment results in the ischemia. Bleeding into the compartment causes greatly increased pressure, first blocking venous outflow from the compartment, then reducing the arterial flow into the compartment as the pressure rises to arterial levels. The ischemic muscles then undergo unrelieved contracture. A humeral fracture is sometimes placed in a cast from shoulder to wrist, often concealing the ischemia until major tissue loss occurs. Cold, insensate digits, and great pain are warnings of this compartmental syndrome, demanding that the cast be removed and the compartment opened ("released") for pressure reduction and vascular repair. Fracture of the surgical neck endangers the axillary nerve and posterior circumflex humeral artery, although not ischemic contracture. Fracture of the humerus in the spiral groove can injure the radial nerve and profunda brachii artery. Fracture of the olecranon does not result in Volkmann's contracture, although the triceps brachii can displace the distal fractured fragment of the ulna. GAS 766; N 420; McM 149

88 A 55-year-old female choreographer had been treated in the emergency department after she fell from the stage into the orchestra pit. Radiographs revealed fracture of the styloid process of the ulna. Disruption of the triangular fibrocartilage complex is suspected. With which of the following bones does the ulna normally articulate at the wrist? A. Triquetrum B. Hamate C. Radius and lunate D. Radius E. Pisiform and triquetrum

88 D. Normally the distal part of the ulna articulates only with the radius at the distal radioulnar joint at the wrist, a joint that participates in pronation and supination. The head of the ulna does not articulate with any of the carpal bones; instead, it is separated from the triquetrum and lunate bones by the triangular fibrocartilage complex between it and the radius. The pisiform articulates with the triquetrum. The carpal articulation of the radius is primarily that of the scaphoid (the old name is navicular) bone. GAS 764-765; N 439; McM 123

89 A 67-year-old woman had a bad fall while walking her dog the evening before. She states that she fell on her outstretched hand. Radiographs do not demonstrate any bony fractures. The clinician observes the following signs of neurologic injury: weakness of flexion of her wrist in a medial direction, a loss of sensation on the medial side of the hand, and clawing of the fingers. Where is the most likely place of nerve trauma? A. Behind the medial epicondyle B. Between the pisiform bone and the flexor retinaculum C. Within the carpal tunnel D. At the cubital fossa, between the ulnar and radial heads of origin of flexor digitorum superficialis E. At the radial neck, 1 cm distal to the humerocapitellar joint

89 A. The force of the woman's fall on the outstretched hand was transmitted up through the forearm, in this case resulting in dislocation of the olecranon at the elbow, putting traction on the ulnar nerve as it passes around the medial epicondyle of the humerus. Ulnar trauma at the elbow can cause weakness in medial flexion (adduction) at the wrist, from loss of the flexor carpi ulnaris. Ulnar nerve injury also results in sensory loss in the medial hand and paralysis of the interossei and medial two lumbricals, with clawing especially of digits 4 and 5. Injury of the ulnar nerve at the pisiform bone would not affect the flexor carpi ulnaris, nor would it produce sensory loss on the dorsum of the hand because the dorsal cutaneous branch of the ulnar branches off proximal to the wrist. Carpal tunnel problems affect median nerve function, which is not indicated here. The ulnar nerve passes medial to the cubital fossa between the heads of the flexor carpi ulnaris, not between the heads of the flexor digitorum superficialis. Injuries at the radial neck affect the site of division of the radial nerve, and its paralysis would not result in the clinical problems seen in this patient. GAS 768, 784; N 464; McM 145

9 A 24-year-old female medical student was bitten at the base of her thumb by her dog. The wound became infected and the infection spread into the radial bursa. The tendon(s) of which muscle will most likely be affected? A. Flexor digitorum profundus B. Flexor digitorum superficialis C. Flexor pollicis longus D. Flexor carpi radialis E. Flexor pollicis brevis

9 C. Tenosynovitis can be due to an infection of the synovial sheaths of the digits. Tenosynovitis in the thumb may spread through the synovial sheath of the flexor pollicis longus tendon, also known as the radial bursa. The tendons of the flexor digitorum superficialis and profundus muscles are enveloped in the common synovial flexor sheath or ulnar bursa. Neither the flexor carpi radialis nor flexor pollicis brevis tendons are contained in synovial flexor sheaths. GAS 802; N 448; McM 158

9. A 21-year-old woman walks in with a shoulder and arm injury after falling during horseback riding. Examination indicates that she cannot adduct her arm because of paralysis of which of the following muscles? (A) Teres minor (B) Supraspinatus (C) Latissimus dorsi (D) Infraspinatus (E) Serratus anterior

9. The answer is C. The latissimus dorsi adducts the arm, and the supraspinatus muscle abducts the arm. The infraspinatus and the teres minor rotate the arm laterally. The serratus anterior rotates the glenoid cavity of the scapula upward, abducts the arm, and elevates it above a horizontal position.

90 An 18-year-old man suffered a significant laceration through the skin and underlying tissues at the distal crease of the wrist. The medical student rotating through the emergency department suspected (correctly) that the ulnar nerve was cut completely through at this location. Which of the following would most likely occur? A. The patient could not touch the tip of the thumb to the tips of the other digits B. There would be loss of sensation on the dorsum of the medial side of the hand C. The patient would be unable to flex the interphalangeal joints D. There would be decreased ability to extend the interphalangeal joints E. There would be no serious functional problem at all to the patient

90 D. The interossei are the most important muscles in extension of the interphalangeal (IP) joints because of the manner of their insertion into the extensor expansion of the fingers, which passes dorsal to the transverse axes of these joints. The lumbrical muscles assist in IP extension, in addition to flexing the metacarpophalangeal joints. Ulnar nerve injury at the wrist results in paralysis of all the interossei and the medial two lumbricals. Extensors of the metacarpophalangeal (MCP) joints are innervated by the deep radial nerve. Unopposed extension of the MCP joints causes them to be held in extension whereas unopposed long flexors of the fingers (supplied by median and ulnar nerves proximally in the forearm) cause them to be flexed into the "claw" position. The lumbricals of digits two and three are still intact because they are supplied by the median nerve, so clawing is not seen as much on these digits. Loss of opposition would result from median or recurrent nerve paralysis. If the ulnar nerve is cut at the wrist, its dorsal cutaneous branch to the dorsum of the hand is unaffected. GAS 808-809; N 451; McM 159

91 A 45-year-old man visits the outpatient clinic after a digit of his left hand was injured when a door was slammed on his hand. A superficial cut on his middle finger has been sutured, but functional deficits are observed in the finger. The proximal interphalangeal joint is pulled into constant flexion, whereas the distal interphalangeal joint is held in a position of hyperextension. What is the most likely diagnosis? A. Mallet finger B. Boutonnière deformity C. Dupuytren's contracture D. Swan-neck deformity E. Silver fork wrist deformity

91 B. In boutonnière deformity, the central portion of the extensor tendon expansion is torn over the proximal interphalangeal (PIP) joint, allowing the tendon to move toward the palm, causing the tendon to act as a flexor of the PIP joint. This causes the distal interphalangeal (DIP) joint to be hyperextended. The tear in the extensor tendon is said to resemble a buttonhole (boutonnière in French), and the head of the proximal phalanx may stick through the hole. GAS 795-796; N 451; McM 166

92 A 67-year-old housepainter visits the outpatient clinic complaining that his hands are getting progressively worse, becoming more and more painful and losing their function. On physical examination of the hands, there is flexion of the metacarpophalangeal joints, extension of the proximal interphalangeal joints, and slight flexion of the distal interphalangeal joints. What is the most likely diagnosis? A. Mallet finger B. Boutonnière deformity C. Dupuytren's contracture D. Swan-neck deformity E. Silver fork wrist deformity

92 D. Swan-neck deformity involves slight flexion of the metacarpophalangeal (MCP) joints, hyperextension of the proximal interphalangeal (PIP) joints, and slight flexion of the distal interphalangeal (DIP) joints. This condition results most often from shortening of the tendons of intrinsic muscles, as in rheumatoid arthritis. When asked to straighten the injured finger, the patient is unable to do so and the curvature of the finger somewhat resembles the neck of a swan. GAS 795-796; N 451; McM 159

93 Several weeks after surgical dissection of her left axilla for the removal of lymph nodes for staging and treatment of her breast cancer, a 32-year-old woman was told by her general physician that she had "winging" of her left scapula when she pushed against resistance during her physical examination. She told the physician that she had also experienced difficulty lately in raising her left arm above her head when she was combing her hair. In a subsequent consult visit with her surgeon, she was told that a nerve was accidentally injured during the diagnostic surgical procedure and that this produced her scapular abnormality and inability to raise her arm normally. What was the origin of this nerve? A. The upper trunk of her brachial plexus B. The posterior division of the middle trunk C. Ventral rami of the brachial plexus D. The posterior cord of the brachial plexus E. The lateral cord of the brachial plexus

93 C. The long thoracic nerve was injured during the axillary dissection, resulting in paralysis of the serratus anterior muscle. The serratus anterior is important in rotation of the scapula in raising the arm above the level of the shoulder. Its loss results in protrusion of the medial border ("winging" of the scapula), which is more obvious when one pushes against resistance. The long thoracic nerve arises from the ventral rami of C5, C6, and C7. The upper trunk (C5, C6) supplies rotator and abductor muscles of the shoulder and elbow flexors. The posterior division of the middle trunk contains C7 fibers for distribution to extensor muscles; likewise, the posterior cord supplies extensors of the arm, forearm, and hand. The lateral cord (C5, C6, and C7) gives origin to the lateral pectoral nerve, the musculocutaneous nerve, and the lateral root of the median nerve. There is no sensory loss in the limb in this patient; injury to any of the other nerve elements listed here would be associated with specific dermatome losses. GAS 726-727; N 413; McM 138

94 A 72-year-old man consulted his physician because he had noticed a thickening of the skin at the base of his left ring finger during the preceding 3 months. As he described it, "There appears to be some hard tissue that is pulling my little and ring fingers into my palm." On examination of the palms of both hands, localized and firm ridges are observed in the palmar skin that extend from the middle part of the palm to the base of the ring and little fingers. What is the medical term for this sign? A. Ape hand B. Dupuytren's contracture C. Claw hand D. Wrist drop E. Mallet finger

94 B. Dupuytren's contracture or deformity is a result of fibromatosis of palmar fascia, resulting in irregular thickening of the fascial attachments to the skin, which causes gradual contraction of the digits, especially digits 4 and 5. In 50% of cases, it is bilateral in occurrence. Ape hand, or flat hand, is a result of loss of the median and ulnar nerves at the elbow, with paralysis of all long flexors of the fingers and all intrinsic hand muscles. The term can also be specific for just median nerve injury and a flattened thenar eminence. Claw hand results from paralysis of interphalangeal joint extension by interossei and lumbricals, innervated primarily by the ulnar nerve. Wrist drop occurs with radial nerve paralysis and loss of the extensors carpi radialis longus and brevis. Mallet finger results from detachment of the extensor mechanism from the distal phalanx of a finger and unopposed flexion of that distal interphalangeal joint. GAS 800; N 446; McM 157

95 A 24-year-old female basketball player is admitted to the emergency department after an injury to her shoulder. Radiographic examination reveals a shoulder dislocation. What is the most commonly injured nerve in shoulder dislocations? A. Axillary B. Radial C. Median D. Ulnar E. Musculocutaneous

95 A. The axillary nerve is a direct branch of the posterior cord and wraps around the surgical neck of the humerus to innervate the teres minor and the deltoid muscles. With this anatomic arrangement, the axillary nerve is tightly "tethered" to the proximal humerus. When the head of the humerus is dislocated, it often puts traction on the axillary nerve. GAS 718-719; N 413; McM 142

96 An 85-year-old man is admitted to the hospital with a painful arm after lifting a case of wine. Physical examination gives evidence of a rupture of the long tendon of the biceps brachii (Fig. 6-6). Which of the following is the most likely location of the rupture? A. Intertubercular groove B. Midportion of the biceps brachii muscle C. Junction with the short head of the biceps brachii muscle D. Proximal end of the combined biceps brachii muscle E. Bony insertion of the muscle

96 A. The tendon of the long head of the biceps brachii muscles runs in the intertubercular groove on the proximal humerus as it changes direction and turns medially to attach to the supraglenoid tubercle of the scapula. This change in direction within an osseous structure predisposes the tendon to wear and tear, particularly in people who overuse the biceps brachii muscle. This type of injury presents with a characteristic sign called the "Popeye sign" after the cartoon character. GAS 731-732; N 417; McM 114

97 After an orthopedic surgeon examined the MRI of the shoulder of a 42-year-old woman he informed her that the supraspinatus tendon was injured and needed to be repaired surgically. Which of the following is true of the supraspinatus muscle? A. It inserts onto the lesser tubercle of the humerus. B. It initiates adduction of the shoulder. C. It is innervated chiefly by the C5 spinal nerve. D. It is supplied by the upper subscapular nerve. E. It originates from the lateral border of the scapula.

97 C. The supraspinatus muscle inserts on the greater tubercle of the humerus and is said to initiate abduction of the arm at the shoulder. It is supplied principally by spinal nerve C5. The subscapularis muscle is the only muscle that inserts on the lesser tubercle. The subscapularis muscle is innervated by the upper and lower subscapular nerves. The teres minor muscle takes origin from the lateral border of the scapula; the teres major muscle takes origin from the region of the inferior angle and the lateral border of the scapula. GAS 717; N 411; McM 115

98 A 5-year-old boy is admitted to the emergency department after falling from a tree. The parents are informed by the radiologist that their son's fracture is the most common fracture that occurs in children. Which of the following bones was broken? A. Humerus B. Radius C. Ulna D. Scaphoid E. Clavicle

98 E. During a fall on an outstretched upper limb, the forces are conducted through the hand on up through the bones of the limb in succession. Often these bones do not fracture but rather pass the compressive forces proximally. The appendicular skeleton joins with the axial skeleton at the sternoclavicular joint. The forces are not sufficiently transferred to the sternum, causing the clavicle to absorb the force, resulting in common pediatric fracture of this sigmoidal-shaped bone. GAS 711; N 461; McM 112

99 A 22-year-old woman visits the outpatient clinic with pain in her left upper limb. She has a long history of pain in this limb and difficulty with fine motor tasks of the hand. Physical examination reveals paraesthesia along the medial surface of the forearm and palm and weakness and atrophy of gripping muscles ("long flexors") and the intrinsic muscles of the hand. The radial pulse is diminished when her neck is rotated to the ipsilateral side (positive Adson's test). What is the most likely diagnosis? A. Erb-Duchenne paralysis B. Aneurysm of the brachiocephalic artery, with plexus compression C. Thoracic outlet syndrome D. Carpal tunnel syndrome E. Injury to the medial cord of the brachial plexus

99 C. The patient is suffering from thoracic outlet syndrome, involving neural and vascular elements. This results from any condition that decreases the dimensions of the superior thoracic aperture (the formal name of the thoracic outlet). It could be a result of a cervical rib, accessory muscles, and/or atypical connective tissue bands at the root of the neck. In this case, symptoms involve the arm, forearm, and hand. Paraesthesia along the medial forearm and hand and atrophy of long flexors and intrinsic muscles point to a possible compression or traction problem of the lower trunk (C8, T1) rather than a lesion of either the median or ulnar nerve. The lateral palm has no sensory problem, which tends to rule out median nerve involvement. Changes in the radial artery pulse point to vascular compression. Erb-Duchenne paralysis of the upper trunk would affect proximal limb functions, such as arm rotation, abduction, and so on. This lesion is on the left side, so the brachiocephalic artery could not be involved because it arises from the right side of the aortic arch; moreover, it would not compress the brachial plexus. Carpal tunnel syndrome would not explain the problems of the forearm and medial hand, or the long flexor atrophy. An isolated medial cord lesion would not explain the atrophy of all long flexors and intrinsic muscles and does not explain the radial pulse characteristics. The ischemic pain in the arm is due to vascular compression. GAS 150; N 183; McM 138

1. A 21-year-old patient has a lesion of the upper trunk of the brachial plexus (Erb- Duchenne paralysis). Which of the following is the most likely diagnosis? (A) Paralysis of the rhomboid major (B) Inability to elevate the arm above the horizontal (C) Arm tending to lie in medial rotation (D) Loss of sensation on the medial side of the arm (E) Inability to adduct the thumb

C upper trunk leision= waiters tip hand arm= -medial rotates long thoracic innervates serratus anterior = elevates arm above horizontal Dorsal scapular innervates rhomboid major The adductor pollicis is innervated by the ulnar nerve.

11. The police bring in a murder suspect who has been in a gunfi ght with a police offi cer. The suspect was struck by a bullet in the arm; his median nerve has been damaged. Which of the following symptoms is likely produced by this nerve damage? (A) Waiter's tip hand (B) Claw hand (C) Wrist drop (D) Ape hand (E) Flattening of the hypothenar eminence

The answer is D. Injury to the median nerve produces the ape hand (a hand with the thumb permanently extended). Injury to the radial nerve results in loss of wrist extension, leading to wrist drop. Damage to the upper trunk of the brachial plexus produces waiter's tip hand. A claw hand and fl attening of the hypothenar eminence


Conjuntos de estudio relacionados

Prep U's - Chapter 16 - Outcome Identification and Planning (TF)

View Set

mammals test What best describes what this creature does during the coldest parts of winter

View Set

Unit 11 Market Grades and Classes

View Set

ATI-RN Metabolism 3.0 Case Study Test Part 1 (Type 1 DM)

View Set

AP Psych Unit 7 Key Terms, AP Psych Unit 6 Key Terms, AP Psych Unit 2 Key Terms, AP Psych Unit 1 Key Terms, AP Psych Unit 5 Key Terms, AP Psych Unit 4 Key Terms, AP Psych Unit 3 Key Terms

View Set